Вы находитесь на странице: 1из 296

6 th

Edition

GMAT Number Properties Guide


Joern Meissner

TURBOCHARGE
GMAT and GMAT CAT are registered trademarks
YOUR PREP
of the Graduate Management Admission Council (GMAC).
GMAC does not endorse nor is it affiliated in any way with
the owner of this product or any content herein.

+1 (212) 316-2000 www.manhattanreview.com


Turbocharge Your GMAT:
Number Properties Guide
part of the 6th Edition Series
April 20th, 2016

 Coverage of all Number Properties topics


relevant for takers of the GMAT
 Intuitive and graphical explanations of
concepts
 250+ GMAT-like practice questions
Great collection of 700+ level questions
Ample questions with Alternate Ap-
proaches

 Mapped according to the scope of the


GMAT

www.manhattanreview.com
19992016 Manhattan Review. All Rights Reserved.
ii Number Properties Guide

Copyright and Terms of Use

Copyright and Trademark


All materials herein (including names, terms, trademarks, designs, images, and graphics) are
the property of Manhattan Review, except where otherwise noted. Except as permitted herein,
no such material may be copied, reproduced, displayed or transmitted or otherwise used with-
out the prior written permission of Manhattan Review. You are permitted to use material
herein for your personal, noncommercial use, provided that you do not combine such material
into a combination, collection, or compilation of material. If you have any questions regarding
the use of the material, please contact Manhattan Review at info@manhattanreview.com.

This material may make reference to countries and persons. The use of such references is for
hypothetical and demonstrative purposes only.

Terms of Use
By using this material, you acknowledge and agree to the terms of use contained herein.

No Warranties
This material is provided without warranty, either express or implied, including the implied
warranties of merchantability, of fitness for a particular purpose and noninfringement. Man-
hattan Review does not warrant or make any representations regarding the use, accuracy or
results of the use of this material. This material may make reference to other source materials.
Manhattan Review is not responsible in any respect for the content of such other source ma-
terials, and disclaims all warranties and liabilities with respect to the other source materials.

Limitation on Liability
Manhattan Review shall not be responsible under any circumstances for any direct, indirect,
special, punitive, or consequential damages (Damages) that may arise from the use of this
material. In addition, Manhattan Review does not guarantee the accuracy or completeness of
its course materials, which are provided as is with no warranty, express or implied. Man-
hattan Review assumes no liability for any Damages from errors or omissions in the material,
whether arising in contract, tort or otherwise.

GMAT is a registered trademark of the Graduate Management Admission Council.


GMAC does not endorse, nor is it affiliated in any way with, the owner of this product or any
content herein.
10-Digit International Standard Book Number: (ISBN: 1-62926-058-4)
13-Digit International Standard Book Number: (ISBN: 978-1-62926-058-7)

Last updated on April 20th, 2016.

Manhattan Review, 275 Madison Avenue, Suite 1429, New York, NY 10016.
Phone: +1 (212) 316-2000. E-Mail: info@manhattanreview.com. Web: www.manhattanreview.com

www.manhattanreview.com 19992016 Manhattan Review


Number Properties Guide iii

About the Turbocharge your GMAT Series

The Turbocharge Your GMAT Series is carefully designed to be clear, comprehensive, and
content-driven. Long regarded as the gold standard in GMAT prep worldwide, Manhattan Re-
views GMAT prep books offer professional GMAT instruction for dramatic score improvement.
Now in its updated 6th edition, the full series is designed to provide GMAT test-takers with
complete guidance for highly successful outcomes. As many students have discovered, Man-
hattan Reviews GMAT books break down the different test sections in a coherent, concise,
and accessible manner. We delve deeply into the content of every single testing area and zero
in on exactly what you need to know to raise your score. The full series is comprised of 16
guides that cover concepts in mathematics and grammar from the most basic through the
most advanced levels, making them a great study resource for all stages of GMAT preparation.
Students who work through all of our books benefit from a substantial boost to their GMAT
knowledge and develop a thorough and strategic approach to taking the GMAT.

 GMAT Math Essentials (ISBN: 978-1-62926-057-0)


 GMAT Number Properties Guide (ISBN: 978-1-62926-058-7)
 GMAT Arithmetic Guide (ISBN: 978-1-62926-059-4)
 GMAT Algebra Guide (ISBN: 978-1-62926-060-0)
 GMAT Geometry Guide (ISBN: 978-1-62926-061-7)
 GMAT Word Problems Guide (ISBN: 978-1-62926-062-4)
 GMAT Sets & Statistics Guide (ISBN: 978-1-62926-063-1)
 GMAT Combinatorics & Probability Guide (ISBN: 978-1-62926-064-8)
 GMAT Data Sufficiency Guide (ISBN: 978-1-62926-065-5)
 GMAT Quantitative Question Bank (ISBN: 978-1-62926-066-2)
 GMAT Sentence Correction Guide (ISBN: 978-1-62926-067-9)
 GMAT Critical Reasoning Guide (ISBN: 978-1-62926-068-6)
 GMAT Reading Comprehension Guide (ISBN: 978-1-62926-069-3)
 GMAT Integrated Reasoning Guide (ISBN: 978-1-62926-070-9)
 GMAT Analytical Writing Guide (ISBN: 978-1-62926-071-6)
 GMAT Vocabulary Builder (ISBN: 978-1-62926-072-3)

19992016 Manhattan Review www.manhattanreview.com


iv Number Properties Guide

About the Company


Manhattan Reviews origin can be traced directly back to an Ivy League MBA classroom in 1999.
While teaching advanced quantitative subjects to MBAs at Columbia Business School in New
York City, Professor Dr. Joern Meissner developed a reputation for explaining complicated
concepts in an understandable way. Remembering their own less-than-optimal experiences
preparing for the GMAT, Prof. Meissners students challenged him to assist their friends, who
were frustrated with conventional GMAT preparation options. In response, Prof. Meissner
created original lectures that focused on presenting GMAT content in a simplified and intel-
ligible manner, a method vastly different from the voluminous memorization and so-called
tricks commonly offered by others. The new approach immediately proved highly popular
with GMAT students, inspiring the birth of Manhattan Review.

Since its founding, Manhattan Review has grown into a multi-national educational services
firm, focusing on GMAT preparation, MBA admissions consulting, and application advisory
services, with thousands of highly satisfied students all over the world. The original lectures
have been continuously expanded and updated by the Manhattan Review team, an enthusiastic
group of master GMAT professionals and senior academics. Our team ensures that Manhattan
Review offers the most time-efficient and cost-effective preparation available for the GMAT.
Please visit www.ManhattanReview.com for further details.

About the Founder


Professor Dr. Joern Meissner has more than 25 years of teaching experience at the graduate
and undergraduate levels. He is the founder of Manhattan Review, a worldwide leader in test
prep services, and he created the original lectures for its first GMAT preparation class. Prof.
Meissner is a graduate of Columbia Business School in New York City, where he received a PhD
in Management Science. He has since served on the faculties of prestigious business schools
in the United Kingdom and Germany. He is a recognized authority in the areas of supply chain
management, logistics, and pricing strategy. Prof. Meissner thoroughly enjoys his research,
but he believes that grasping an idea is only half of the fun. Conveying knowledge to others
is even more fulfilling. This philosophy was crucial to the establishment of Manhattan Review,
and remains its most cherished principle.

www.manhattanreview.com 19992016 Manhattan Review


Number Properties Guide v

The Advantages of Using Manhattan Review


I Time efficiency and cost effectiveness.
For most people, the most limiting factor of test preparation is time.
It takes significantly more teaching experience to prepare a student in less time.
Our test preparation approach is tailored for busy professionals. We will teach you
what you need to know in the least amount of time.

I Our high-quality and dedicated instructors are committed to helping every student
reach her/his goals.

19992016 Manhattan Review www.manhattanreview.com


vi Number Properties Guide

International Phone Numbers and Official Manhattan Review Websites


Manhattan Headquarters +1-212-316-2000 www.manhattanreview.com
USA & Canada +1-800-246-4600 www.manhattanreview.com
Argentina +1-212-316-2000 www.review.com.ar
Australia +61-3-9001-6618 www.manhattanreview.com
Austria +43-720-115-549 www.review.at
Belgium +32-2-808-5163 www.manhattanreview.be
Brazil +1-212-316-2000 www.manhattanreview.com.br
Chile +1-212-316-2000 www.manhattanreview.cl
China +86-20-2910-1913 www.manhattanreview.cn
Czech Republic +1-212-316-2000 www.review.cz
France +33-1-8488-4204 www.review.fr
Germany +49-89-3803-8856 www.review.de
Greece +1-212-316-2000 www.review.com.gr
Hong Kong +852-5808-2704 www.review.hk
Hungary +1-212-316-2000 www.review.co.hu
India +1-212-316-2000 www.review.in
Indonesia +1-212-316-2000 www.manhattanreview.id
Ireland +1-212-316-2000 www.gmat.ie
Italy +39-06-9338-7617 www.manhattanreview.it
Japan +81-3-4589-5125 www.manhattanreview.jp
Malaysia +1-212-316-2000 www.review.my
Mexico +1-212-316-2000 www.manhattanreview.mx
Netherlands +31-20-808-4399 www.manhattanreview.nl
New Zealand +1-212-316-2000 www.review.co.nz
Philippines +1-212-316-2000 www.review.ph
Poland +1-212-316-2000 www.review.pl
Portugal +1-212-316-2000 www.review.pt
Qatar +1-212-316-2000 www.review.qa
Russia +1-212-316-2000 www.manhattanreview.ru
Singapore +65-3158-2571 www.gmat.sg
South Africa +1-212-316-2000 www.manhattanreview.co.za
South Korea +1-212-316-2000 www.manhattanreview.kr
Sweden +1-212-316-2000 www.gmat.se
Spain +34-911-876-504 www.review.es
Switzerland +41-435-080-991 www.review.ch
Taiwan +1-212-316-2000 www.gmat.tw
Thailand +66-6-0003-5529 www.manhattanreview.com
Turkey +1-212-316-2000 www.review.com.tr
United Arab Emirates +1-212-316-2000 www.manhattanreview.ae
United Kingdom +44-20-7060-9800 www.manhattanreview.co.uk
Rest of World +1-212-316-2000 www.manhattanreview.com

www.manhattanreview.com 19992016 Manhattan Review


Contents

1 Welcome 1

2 Number Concepts 3
2.1 Understanding numbers . . . . . . . . . . . . . . . . . . . . . . . . . . . . . . . . . . . . 4
2.1.1 Classification of numbers . . . . . . . . . . . . . . . . . . . . . . . . . . . . . . . 4
2.1.2 Number line . . . . . . . . . . . . . . . . . . . . . . . . . . . . . . . . . . . . . . . . 6
2.1.3 Basic operations on numbers . . . . . . . . . . . . . . . . . . . . . . . . . . . . . 7
2.1.4 Face value and place value . . . . . . . . . . . . . . . . . . . . . . . . . . . . . . . 9
2.1.5 Even and Odd numbers . . . . . . . . . . . . . . . . . . . . . . . . . . . . . . . . 10
2.1.6 Consecutive numbers . . . . . . . . . . . . . . . . . . . . . . . . . . . . . . . . . . 11
2.1.7 Prime numbers . . . . . . . . . . . . . . . . . . . . . . . . . . . . . . . . . . . . . . 11
2.1.8 Co-Prime numbers . . . . . . . . . . . . . . . . . . . . . . . . . . . . . . . . . . . . 12
2.1.9 Rounding off . . . . . . . . . . . . . . . . . . . . . . . . . . . . . . . . . . . . . . . 12
2.1.10 Order of operations PEMDAS . . . . . . . . . . . . . . . . . . . . . . . . . . . . 15
2.1.11 Divisibility . . . . . . . . . . . . . . . . . . . . . . . . . . . . . . . . . . . . . . . . . 16
2.1.12 Factors . . . . . . . . . . . . . . . . . . . . . . . . . . . . . . . . . . . . . . . . . . . 18
2.1.13 Perfect Square numbers . . . . . . . . . . . . . . . . . . . . . . . . . . . . . . . . 19
2.1.14 Multiples . . . . . . . . . . . . . . . . . . . . . . . . . . . . . . . . . . . . . . . . . 20
2.1.15 Lowest Common MultipleLCM . . . . . . . . . . . . . . . . . . . . . . . . . . . 20
2.1.16 Highest Common FactorHCF OR Greatest Common DivisorGCD . . . . . 22
2.2 Exponents . . . . . . . . . . . . . . . . . . . . . . . . . . . . . . . . . . . . . . . . . . . . . 24
2.2.1 Unit digits of numbers with exponents . . . . . . . . . . . . . . . . . . . . . . . 25
2.2.2 Deduction of unit digit . . . . . . . . . . . . . . . . . . . . . . . . . . . . . . . . . 26
2.2.3 Roots and Surds . . . . . . . . . . . . . . . . . . . . . . . . . . . . . . . . . . . . . 27
2.2.3.1 Surds . . . . . . . . . . . . . . . . . . . . . . . . . . . . . . . . . . . . . . 28
2.2.3.2 Rationalization of surds . . . . . . . . . . . . . . . . . . . . . . . . . . . 28
2.3 Multiplication and Division of decimals with 10x . . . . . . . . . . . . . . . . . . . . . 29
2.4 Common Binomial expressions . . . . . . . . . . . . . . . . . . . . . . . . . . . . . . . . 31
2.5 Some important summation of series . . . . . . . . . . . . . . . . . . . . . . . . . . . . 32
2.6 Fractions . . . . . . . . . . . . . . . . . . . . . . . . . . . . . . . . . . . . . . . . . . . . . . 33
2.6.1 Concept of fraction . . . . . . . . . . . . . . . . . . . . . . . . . . . . . . . . . . . 34
2.6.2 Mathematical operations on fractions . . . . . . . . . . . . . . . . . . . . . . . . 35
2.6.3 Comparing fractions . . . . . . . . . . . . . . . . . . . . . . . . . . . . . . . . . . 36
2.6.4 More on fractions . . . . . . . . . . . . . . . . . . . . . . . . . . . . . . . . . . . . 38
2.6.5 Fractions and their percent equivalents . . . . . . . . . . . . . . . . . . . . . . 39
2.6.6 See-Saw Approach . . . . . . . . . . . . . . . . . . . . . . . . . . . . . . . . . . . . 40
2.7 Absolute numbers . . . . . . . . . . . . . . . . . . . . . . . . . . . . . . . . . . . . . . . . 43

vii
viii Number Properties Guide

2.7.1 Inequality . . . . . . . . . . . . . . . . . . . . . . . . . . . . . . . . . . . . . . . . . 45
2.7.2 Inequalities with absolute numbers . . . . . . . . . . . . . . . . . . . . . . . . . 47
2.7.3 Compound Inequality . . . . . . . . . . . . . . . . . . . . . . . . . . . . . . . . . . 48
2.7.4 Mathematical operations on inequalities . . . . . . . . . . . . . . . . . . . . . . 49

3 Practice Questions 53
3.1 Problem Solving . . . . . . . . . . . . . . . . . . . . . . . . . . . . . . . . . . . . . . . . . 54
3.1.1 Numbers & Digits . . . . . . . . . . . . . . . . . . . . . . . . . . . . . . . . . . . . 54
3.1.2 Even/Odd/Consecutive/Prime numbers . . . . . . . . . . . . . . . . . . . . . . 60
3.1.3 Divisibility . . . . . . . . . . . . . . . . . . . . . . . . . . . . . . . . . . . . . . . . . 64
3.1.4 Exponents . . . . . . . . . . . . . . . . . . . . . . . . . . . . . . . . . . . . . . . . . 67
3.1.5 Factors/Multiplicands/LCM/HCF . . . . . . . . . . . . . . . . . . . . . . . . . . . 70
3.1.6 Inequality . . . . . . . . . . . . . . . . . . . . . . . . . . . . . . . . . . . . . . . . . 73
3.2 Data Sufficiency . . . . . . . . . . . . . . . . . . . . . . . . . . . . . . . . . . . . . . . . . 76
3.2.1 Numbers & Digits . . . . . . . . . . . . . . . . . . . . . . . . . . . . . . . . . . . . 77
3.2.2 Even/Odd/Consecutive/Prime numbers . . . . . . . . . . . . . . . . . . . . . . 79
3.2.3 Divisibility . . . . . . . . . . . . . . . . . . . . . . . . . . . . . . . . . . . . . . . . . 81
3.2.4 Exponents . . . . . . . . . . . . . . . . . . . . . . . . . . . . . . . . . . . . . . . . . 84
3.2.5 Factors/Multiplicands/LCM/HCF . . . . . . . . . . . . . . . . . . . . . . . . . . . 87
3.2.6 Inequality . . . . . . . . . . . . . . . . . . . . . . . . . . . . . . . . . . . . . . . . . 88
3.3 Assorted Questions . . . . . . . . . . . . . . . . . . . . . . . . . . . . . . . . . . . . . . . 93
3.3.1 Problem Solving . . . . . . . . . . . . . . . . . . . . . . . . . . . . . . . . . . . . . 93
3.3.2 Data Sufficiency . . . . . . . . . . . . . . . . . . . . . . . . . . . . . . . . . . . . . 107

4 Answer-key 125
4.1 Problem Solving . . . . . . . . . . . . . . . . . . . . . . . . . . . . . . . . . . . . . . . . . 126
4.2 Data Sufficiency . . . . . . . . . . . . . . . . . . . . . . . . . . . . . . . . . . . . . . . . . 127
4.3 Assorted Questions . . . . . . . . . . . . . . . . . . . . . . . . . . . . . . . . . . . . . . . 128
4.3.1 Problem Solving . . . . . . . . . . . . . . . . . . . . . . . . . . . . . . . . . . . . . 128
4.3.2 Data Sufficiency . . . . . . . . . . . . . . . . . . . . . . . . . . . . . . . . . . . . . 129

5 Solution 131
5.1 Problem Solving . . . . . . . . . . . . . . . . . . . . . . . . . . . . . . . . . . . . . . . . . 132
5.1.1 Numbers & Digits . . . . . . . . . . . . . . . . . . . . . . . . . . . . . . . . . . . . 132
5.1.2 Even/Odd/Consecutive/Prime numbers . . . . . . . . . . . . . . . . . . . . . . 139
5.1.3 Divisibility . . . . . . . . . . . . . . . . . . . . . . . . . . . . . . . . . . . . . . . . . 145
5.1.4 Exponents . . . . . . . . . . . . . . . . . . . . . . . . . . . . . . . . . . . . . . . . . 150
5.1.5 Factors/Multiplicands/LCM/HCF . . . . . . . . . . . . . . . . . . . . . . . . . . . 155
5.1.6 Inequality . . . . . . . . . . . . . . . . . . . . . . . . . . . . . . . . . . . . . . . . . 159
5.2 Data Sufficiency . . . . . . . . . . . . . . . . . . . . . . . . . . . . . . . . . . . . . . . . . 166
5.2.1 Numbers & Digits . . . . . . . . . . . . . . . . . . . . . . . . . . . . . . . . . . . . 167
5.2.2 Even/Odd/Consecutive/Prime numbers . . . . . . . . . . . . . . . . . . . . . . 171
5.2.3 Divisibility . . . . . . . . . . . . . . . . . . . . . . . . . . . . . . . . . . . . . . . . . 177
5.2.4 Exponents . . . . . . . . . . . . . . . . . . . . . . . . . . . . . . . . . . . . . . . . . 184
5.2.5 Factors/Multiplicands/LCM/HCF . . . . . . . . . . . . . . . . . . . . . . . . . . . 191
5.2.6 Inequality . . . . . . . . . . . . . . . . . . . . . . . . . . . . . . . . . . . . . . . . . 194
5.3 Assorted Questions . . . . . . . . . . . . . . . . . . . . . . . . . . . . . . . . . . . . . . . 212
5.3.1 Problem Solving . . . . . . . . . . . . . . . . . . . . . . . . . . . . . . . . . . . . . 212

www.manhattanreview.com 19992016 Manhattan Review


Number Properties Guide ix

5.3.2 Data Sufficiency . . . . . . . . . . . . . . . . . . . . . . . . . . . . . . . . . . . . . 231

6 Talk to Us 281

19992016 Manhattan Review www.manhattanreview.com


x Number Properties Guide

www.manhattanreview.com 19992016 Manhattan Review


Chapter 1

Welcome

Dear Students,
Here at Manhattan Review, we constantly strive to provide you the best educational content
for standardized test preparation. We make a tremendous effort to keep making things better
and better for you. This is especially important with respect to an examination such as the
GMAT. A typical GMAT aspirant is confused with so many test-prep options available. Your
challenge is to choose a book or a tutor that prepares you for attaining your goal. We cannot
say that we are one of the best, it is you who has to be the judge.

There are umpteen numbers of books on Quantitative Ability for GMAT preparation. What is
so different about this book? The answer lies in its approach to deal with the questions. The
book is meant to develop your fundamentals on one of the most scared and widely tested topic
on GMATNumber Properties.

The concepts are explained with the help of text-cum-graphic aid. It is a treat to read the book
along with relevant graphics. Pictures speak louder than words!

You will find a lot of variety in the problems discussed. Alternate approaches to few tricky
questions are worth appreciating. We have tried to craft the options, a typical GMAT test-
maker prepares to trap you and duly explained how to get rid of those. You will find many
700+ level of questions in the book.

The Manhattan Reviews Number Properties book is holistic and comprehensive in all re-
spects. Should you have any queries, please feel free to write to me at info@mahattanreview.com.

Happy Learning!

Professor Dr. Joern Meissner


& The Manhattan Review Team

1
2 Number Properties Guide Concepts

www.manhattanreview.com 19992016 Manhattan Review


Chapter 2

Number Concepts

3
4 Number Properties Guide Concepts

The concepts of numbers are discussed in the book GMAT Math Essentials; we reproduce
the same for you.

2.1 Understanding numbers

Numbersthe backbone of GMAT quants; we all are familiar with num-


bers, but the devil lies in the detail!

Let us understand numbers from the GMAT perspective.

2.1.1 Classification of numbers

Numbers

Real Complex
Numbers (R) Numbers (a+ib)

Rational Irrational
Numbers (Q) Numbers (I)

Integers (Z) Fractions (F) Decimals (D)

(A) Real numbers: Almost all the numbers you can imagine are Real numbers. Any number
that can be represented on a number line is a real number. These include 0, 1, 3, 4/3, 0.3457, 48.87, 3, ,
and many more!

If most numbers are real numbers, then what are not real numbers?


The numbers such as 1, (2 + 5), etc. are not real numbers. They are Imaginary
number and Complex numbers respectively. Fortunately, these are beyond the scope of
the GMAT.

Real numbers can further be classified into Rational and Irrational numbers.

www.manhattanreview.com 19992016 Manhattan Review


Number Properties Guide Concepts 5

p
(a) Rational numbers: Numbers that can be expressed in the form of where p and q
q
are integers, and q 6= 0. Remember that p may be 0.

The above definition may look a little scary to those who are not very familiar with
the concepts of numbers; however the good news for them is that you need not
know the above definition to master the GMAT quants.

So what do rational numbers include? Simply put, these include all integers, deci-
mal, and fractions.

So what do rational numbers exclude? These exclude roots of non-perfect square


numbers, roots of non-perfect cube numbers, special numbers such as = 3.14, e =
2.718, and many others, basically, non terminating & non recurring decimals.

Rational numbers can be broadly classified into three categoriesIntegers, Frac-


tions, and Decimals.

i. Integers: All counting numbers are integers whether negative, positive or zero.
Example: {1, 2, 3, 4, 5,...} are called positive integers; {1, 2, 3, 4, 5, ....}
are called negative integers. Note that 0 is also an integer.

In Data Sufficiency questions, you may come across terms like non-negative in-
tegers or non-positive integers. A non-negative integer would be one among the
set of {1, 2, 3, 4, 5,...}; note that 0 is included; whereas a non-positive integer
would be one among the set of {1, 2, 3, 4, 5, ....}.

ii. Fractions: Any number that can be expressed with an integer numerator and
an integer denominator is called fraction. In other words, a number that can
be expressed in the form of x/y is a fraction such that y 6= 0. Example:
1/3, 5/4, 3/8 etc.

iii. Decimal numbers: Decimal numbers are another way of expressing fractions.
The decimal numbers are written with the use of a decimal (.). The left of the
decimal point (.) has place values of digits for units, tens, hundreds, thousands,
and more; whereas the right of the decimal point (.) has place values of digits
for tenths, hundredths, thousandths, and more.

Only those decimals which are Terminating, for example 2.4, 4.35, 2.1, etc. or
Recurring, for example 2.3333, 4.151515...2.3404040, etc. are Rational.

This is because only these can be expressed in the form p/q, as discussed ear-
lier. For example: A terminating decimal. 2.35 is simply 235/100 = 47/20, and
is recurring. A recurring decimal, say 2.343434... can be analyzed as follows:

Say x = 2.343434...
=> 100x = 234.343434

19992016 Manhattan Review www.manhattanreview.com


6 Number Properties Guide Concepts

Subtracting: 99x = 234 2 = 232


232
=> x = , and hence is a rational number.
99
We will discuss place value in the following pages.
p
(b) Irrational numbers: Numbers that cannot be expressed in the form of , where
q
p and q are integers, and q 6= 0 are irrational numbers or in other words, all real
numbers that are not rational are irrational numbers. As stated earlier that these
include roots of non-perfect squared numbers: 2, 5 and others; roots of non-perfect
cube numbers, special numbers such as = 3.14, e = 2.718, and many others.

By default any number mentioned in the QA section is a real number; so, you must NOT
assume that it is an integer unless stated as such.

(B) Complex numbers: As stated earlier that he numbers such as 1, (2 + 5), etc. are
not real numbers. They are Imaginary number and Complex numbers, respectively. For-
tunately, these are beyond the scope of the GMAT.

2.1.2 Number line


A number line is a line with 0 as its center. Numbers on the right side of 0 are positive and
those on the left side are negative. Number line helps define the direction of measurement.

Numbers keep on increasing

5<1 1/2 3.25

5 4 3 2 1 0 1 2 3 4 5 +
5>6 2.75 1/2

Numbers keep on decreasing

www.manhattanreview.com 19992016 Manhattan Review


Number Properties Guide Concepts 7

2.1.3 Basic operations on numbers

Symbol Mathematical Application Examples


operation

Sum, Add, 2 + 3 = 5; (Two positives will be added)


Plus,
+ Addition Total, (2) + (3) = 5; (Two negatives will be
Increase added)

2 + (3) = 1; (A positive & a negative will


be subtracted)

(2) + 3 = 1; (A negative and a positive will


be subtracted)

Deduct, 23 = 1; (Two positives will be subtracted)


Minus,
Subtraction Decrease, (2) (3) = 1; (Two negatives will
Less, Reduce, be subtracted)
Difference
2 (3) = 5; (A positive & a negative
will be added)

(2) 3 = 5; (A negative & a posi-


tive will be added)

Multiply, 2 3 = 6; (Positive Positive = Positive)


Multiplication Product,
Times, , By (2) (3) = 6; (Negative Negative
= Positive)

2 (3) = 6; (Positive Negative =


Negative)

(2) 3 = 6; (Negative Positive =


Negative)

19992016 Manhattan Review www.manhattanreview.com


8 Number Properties Guide Concepts

Division Divide, 6 3 = 2; (Positive Positive = Positive)


Quotient, Part
of, Per unit (6) (3) = 2; (Negative Negative
calculation = Positive)

6 (3) = 2; (Positive Negative =


Negative)

(6) 3 = 2; (Negative Positive = Nega-


tive)

www.manhattanreview.com 19992016 Manhattan Review


Number Properties Guide Concepts 9

2.1.4 Face value and place value


The number system we use is set up from ten symbols only {0, 1, 2, 3, 4, 5, 6, 7, 8, 9}. Each
symbol is called a digit and each has its value called Face value. So, the face value of 9 is
nine times the face value of the digit 1 or three times the face value of the digit 3. For
example, for numbers 243, & 234, the face values of digits 2, 3, and 4 do not vary upon
their placement.

Place value of a digit varies depending on where it is placed in a number. For example, for
numbers 234, and 243, the place values of digits 3, and 4 are different.

If a digit placed on the left of another digit, the place value of the first digit is 10 times of the
other digit. For a number 234, the place value of digit 2 is 200, that of digit 3 is 30, and
that of 4 is 4. Similarly, the place values of digits placed after the decimal point () reduces
in its value by 1/10 times. This can be better understood this way.

243 = 2 100 + 4 10 + 3 1;

234 = 2 100 + 3 10 + 4 1..

Tenths
Units Decimal
(1/10 times)

Tens Hundredths
(10 times) (1/100 times)

Hundreds Thousandsths
(100 times) (1/1000 times)

87934.125
Thousands
(1000 times)

Ten thousands
(10000 times)

10 times larger 10 times smaller

19992016 Manhattan Review www.manhattanreview.com


10 Number Properties Guide Concepts

2.1.5 Even and Odd numbers


Even number: An integer completely divisible by 2, without leaving a remainder is called an
Even number. In other words, an even number can be expressed as 2m, where m is an inte-
ger. As per this definition, 0 is an even integer. Example: {0, 2, 4, 6, 8, ...}, {8, 4, 10, 18, ...}.

Odd number: An integer when divided by 2 leaves a remainder 1 is called Odd number.
In other words, an odd number can be expressed as (2m + 1), where m is an integer. All the
integers are either even or odd integers. Example: {1, 3, 5, 7, ...}, {3, 1, 9, 11...}.

Basic operations on Even and Odd numbers:

Mathematical Application Examples


operation

Even Even = Even; 4 + 2 = 6; 42 = 2; Even


Odd Odd = Even; 7 + 3 = 10; 37 =4; Even
Addition/ Even Odd = Odd 5 + 2 = 7; 52 = 3; Odd
Subtraction
Any number of even numbers can
make an even number; however
only an even number of odd num-
bers can make an even number;
e.g.: Two odd numbers on addi-
tion give even, four odd numbers
on addition also give even; how-
ever three odd numbers on addi-
tion give odd number.

Even Even = Even; 4 2 = 8; Even


Odd Odd = Odd; 3 3 = 9; Odd
Even Odd = Even 2 3 = 6; Even

Multiplication Multiplication of any number of


odd numbers always results in an-
other odd number.

If the product of two integers is


Even and one of them is an Even
integer, we cannot comment on
the nature of the other number,
i.e. whether it is Even or Odd.

www.manhattanreview.com 19992016 Manhattan Review


Number Properties Guide Concepts 11

Not every operation of Division is


completely divisible. We show you
the results if the division leaves no
remainder.

Division Odd Odd = Odd; 9 3 = 3; Odd


Even Odd = Even; 10 5 = 2; Even
Odd Even = Not divisible; 11 2 = ? Not divisible
Even Even = Even or Odd 4 2 = 2; Even
10 2 = 5; Odd

2.1.6 Consecutive numbers

An ordered set of continuous integers written in ascending order are called Consecutive num-
bers.

Example: {23, 24, 25}, {2, 1, 0, 1, 2, 3}

Example of consecutive Even numbers: {12, 14, 16}

Example of consecutive Odd numbers: {11, 13, 15}

2.1.7 Prime numbers

A number greater than 1 which has no factors other than 1 and the
2 number itself, is called Prime number. A number with exactly two distinct
factors is called prime. Hence, 1 cannot be considered as prime since it
has only one factor, 1 itself.

Example: {2, 3, 5, 7, 11, 13, 17, 19, 23, 29, .....}. There are infinitely many Prime numbers.

First 25 prime numbers are:

{2, 3, 5, 7, 11, 13, 17, 19, 23, 29, 31, 37, 41, 43, 47, 53, 59, 61, 67, 71, 73, 79, 83, 89, 97}

2 is the only even prime number or all the prime numbers are odd except 2.

Also, the difference between any two primes greater than 2 is always even.

Another point to note is that any prime, other than 2 or 3, if divided by 6, will always
leave a remainder of 1 or 5 (== 1). Thus, any prime can be expressed as 6k 1, where
k is some positive positive integer.

However, you should note that not all numbers which leave a remainder of 1 or 5 when divided
by 6 are prime, for example, 25, 35, etc.

19992016 Manhattan Review www.manhattanreview.com


12 Number Properties Guide Concepts

How to verify whether a number n is a prime number:


Step1: Take square root of the number; if square root is an integer the number is not prime
number, else follow the next step.

Step2: Say the largest integer closest to the square root of the number n is m; where

m < n;

Step3: Divide n by all the prime numbers from 2 to m, inclusive: if n is divisible by any
number among them, the number n is not prime, else prime

=> Let us take a number n = 101.



Step1: 10 < 101 < 11; not an integer;

Step2: 101 = 10 = m;

Step3: Divide 101 by prime numbers from 2 to 10 i.e. 2, 3, 5, & 7. Since 101 is not divisible by
any prime number among these, hence 101 is a prime number.

=> Let us take a number n = 87.



Step1: 9 < 87, 10; not an integer;

Step2: 87 = 9 = m;

Step3: Divide 87 by prime numbers from 2 to 9 i.e. 2, 3, 5, & 7. Since 87 is divisible by 3, hence
87 is not a prime number

2.1.8 Co-Prime numbers


Two numbers are co-prime to each other if they have no factor common between them except
1.

Example: 14 & 15: The factors of 14 are {1, 2, 7, 14}, and the factors of 15 are {1, 3, 5, 15}.
There is no factor common between 14 & 15 except 1, so 14 & 15 are co-prime to each
other.

It is important to know that the LCM of co-prime numbers is their product, and GCD or HCF
is 1. Say two numbers a and b are co-prime to each other, then LCM = ab, and GCD = 1.

Note that any two consecutive numbers are always co-prime to each other. Also 1 is co-prime
to any number.

2.1.9 Rounding off


Rounding off a number means to get a significant and practically usable number. Say, the
distance between Manhattan to JFK airport is 15.24 miles; we can truncate the hundredth

www.manhattanreview.com 19992016 Manhattan Review


Number Properties Guide Concepts 13

place digit to get a number which is rounded off to tenth place equalling 15.2 miles. However,
by doing this, we do get a less accurate number. The degree of rounding off a number depends
on how much accurate number you want. In this case, even the additional 0.2 miles may seem
insignificant to a regular commuter. So, 15.2 can further be rounded off to the nearest tens as
15 miles which is further less accurate.

CASE I:

If the dropped digit is less than 5 (i.e. 0, 1, 2, 3, 4), the digit to the left of the dropped digit
should be unchanged. For example, 5.312 rounded to the tenths digit is 5.3.

CASE II:

If the dropped digit is greater than 5 (i.e. 6, 7, 8 or 9), the digit to the left of the dropped
digit should be increased by 1. For example, 5.326 rounded to the hundredths digit is 5.33,
but, rounded to the tenths digit is 5.3; similarly, 4.961 becomes 5.00, if rounded to the nearest
integer.

CASE III:

If the dropped digit is 5, and all the figures following the 5 are 0, or if there is no digit after
the 5:

The digit to the left of the dropped digit should be unchanged, if that last figure is even,
and

The digit to the left of the dropped digit should be increased by 1 if that last figure is
odd. For example, 3.195 becomes 3.20, whereas 2.365 becomes 2.36, both when rounded
to the hundredths digit.

CASE IV:

If the dropped digit is 5, and all the figures following the 5 are not all 0, the digit to the
left of the dropped digit should be increased by 1. For example, 4.7501 rounded to the hun-
dredths digit is 4.8.

The same rules for rounding can be applied in the following cases as well:

2134 rounded to the nearest tens is 2130


2139 rounded to the nearest tens is 2140
2135 rounded to the nearest tens is 2140
2145 rounded to the nearest tens is 2140
2145.1 rounded to the nearest tens is 2150
2145 rounded to the nearest hundreds is 2100
2155 rounded to the nearest hundreds is 2200
2168 rounded to the nearest hundreds is 2200
2134 rounded to the nearest hundreds is 2100

19992016 Manhattan Review www.manhattanreview.com


14 Number Properties Guide Concepts

Note that the number, rounded off, can be more than or less than the original number. 15.8454
rounded off to 15.8 < 15.8454, whereas another number 15.8554 rounded off to 15.9 > 15.8554.

www.manhattanreview.com 19992016 Manhattan Review


Number Properties Guide Concepts 15

2.1.10 Order of operations PEMDAS

12 2(1+2)?
What is the value of 2{32 2(6 52 50 + 25 5)}?
2 or 18

Follow the order of operation given as below:

P: Parentheses first

E: Exponents (i.e. Powers and Square Roots, etc.)

MD: Multiplication and Division (left-to-right)

AS: Addition and Subtraction (left-to-right)

Expression Comments

2{102 2(6 52 50 + 25 5)} Solve the parenthesis first. If there are more than two
parentheses, follow the order: [ ], { }, & ( ). Here, we will
solve { } first, and then ( ). (P)

= 2{100 2(6 25 50 25)} Among the operators exponent, multiplication, division,


addition, subtraction, solve the exponent first. (E)

= 2{100 2(150 50 + 5)} Multiplication and division can be done simultaneously.


(MD)

= 2{100 2(105)} Addition and subtraction can be done simultaneously.


(AS)
= 2{100 210}
= 2{110}
= 220

19992016 Manhattan Review www.manhattanreview.com


16 Number Properties Guide Concepts

2.1.11 Divisibility
A number is said to be divisible by another number if the division does not leave any remain-
der. Following are quick reckoner rules for divisibility test.

Divisor Condition: Example


A number is divisible

2 if the unit digit is Even: {0, 2, 4, 6, 268; 126596


8}

21681; Sum of digits = 2+1+6+8+1 = 18;


18 div. by 3 implies that 21681 is divisible
3 if the Sum of all the digits is by 3.
divisible by 3
89896; Sum of digits = 8+9+8+9+6 = 40;
40 not div. by 3 implies that 89896 is not
divisible by 3.

1224 and 2356 are div. by 4 as 24 and 56


4 if the number formed out of last are divisible by 4.
two digits is divisible by 4
1254 and 2569866 are not div. by 4
as 54 and 66 are not divisible by 4.

1220 and 2355 are div. by 5.


5 if the unit digit is 0 or 5.
1254 and 2569866 are not div. by
5.

if the number is divisible by both 1254, and 89892 are div. by 6.


6 2 and 3; follow the divisibility
rules of 2 and 3.

if the number formed out of the 891224 and 2542352 are div. by 8.
8 last three digits is divisible by 8
561354 and 2569866 are not div. by
8.

21681; Sum of digits = 2+1+6+8+1 = 18;


18 div. by 9 implies that 21681 is divisible
9 if the Sum of all digits of the by 9.
number is divisible by 9
89896; Sum of digits = 8+9+8+9+6 = 40;
40 not div. by 9 implies that 89896 is not
divisible by 9.

www.manhattanreview.com 19992016 Manhattan Review


Number Properties Guide Concepts 17

10 if the unit digit is 0. 12340 and 5245870.

251384;

S.O. = Sum of digits at odd places =


4 + 3 + 5 = 11;

if the difference of the sum S.E. = Sum of digits at even places =


11 of digits at odd places, and that 8 + 1 + 2 = 11;
at even places is either 0 or
divisible by 11. S.O.S.E. = 1111 = 0 implies that 251384
is div. by 11.

81927092;

S.O.S.E. = 527 = 22 div. by 11 implies


that 81927092 is div. by 11.

19992016 Manhattan Review www.manhattanreview.com


18 Number Properties Guide Concepts

2.1.12 Factors

All the possible divisors of an integer which leave no reminder are called factors.
Example: The factors of 6 are 1, 2, 3, & 6. We can write 6 as 1 6 or 2 3.

Factors of 60:

60 = 1 60;
= 2 30;
= 3 20;
= 4 15;
= 5 12;
= 6 10.

So, the factors of 60 are {1, 2, 3, 4, 5, 6, 10, 12, 15, 20, 30, 60}. Remember that 1 and the num-
ber itself are also the factors of a number.

Number of factors

y y
Say, a number N can be written as N = a x b c z , where a, b, and c are prime factors of the
number N, and x, y, and z are positive integers, then

=> Number of factors (including 1 and the number itself) = (x + 1)(y + 1)(z + 1);

=> Number of factors (excluding 1 and the number itself) = (x + 1)(y + 1)(z + 1) 2.

Number of factors of 24:

24 = 2 2 2 3 = 23 31 ; here a = 2, & b = 3 are prime numbers, and x = 2, & y = 1.

Hence the number of factors of 24 including 1 and the number itself


= (3 + 1)(1 + 1) = 4 2 = 8.

Let us verify this.

Factors of 24 are: {1, 2, 3, 4, 6, 8, 12, 28}; total number of factors equals to 8.

Let us take one more example.

Number of factors of 2520:

2520 = 2 2 2 3 3 5 7 = 23 32 5 7; here 2, 3, 5, & 7 are prime numbers.

Hence the number of factors of 2520 including 1 and the number itself
= (3 + 1)(2 + 1)(1 + 1)(1 + 1) = 4.3.2.2 = 48.

www.manhattanreview.com 19992016 Manhattan Review


Number Properties Guide Concepts 19

Sum of all the factors:

Say, a number N can be written as N = ax by c z , where a, b, and c are prime factors of number
N, x, y, and z are positive integers, then
" #" #" #
ax+1 1 by+1 1 c z+1 1
Sum of all the factors (including 1 and the number itself) =
a1 b1 c1
Sum of all the factors of 60:

60 = 22 .31 .51 ;
" #" #" #
22+1 1 31+1 1 51+1 1
Sum of all the factors (including 1 and the number itself) =
21 31 51
= 7.(8/2).(24/4) = 168.

Let us cross check this. Sum of all the factors of 60 = 1 + 2 + 3 + 4 + 5 + 6 + 10 + 12 + 15 +


20 + 30 + 60 = 168.

2.1.13 Perfect Square numbers


A perfect square number is a non-negative integer that can be expressed as the product of an
integer with itself. In other words, a number which is a square of any integer is called Perfect
square number. Perfect square numbers are always non-negative integers.

Example: {0, 1, 4, 9, 16, 25, 36, 49, ...}. Some mathematicians do not consider that 0 is a per-
fect square number. Moreover the GMAT does not assume that you know the meaning of
perfect square number. The phrase used typically is square of an integer.

We can see that 1 = 1 1 = 1 1; and 4 = 2 2 = 2 2.

Some properties of perfect square:

Perfect square number ALWAYS has even number of powers of prime factors.

The number of distinct factors of a perfect square number is ALWAYS Odd. (By factors,
we refer to only the positive numbers that divide the number without remainder.)

The sum of distinct factors of a perfect square number is ALWAYS Odd.

A perfect square number ALWAYS has an Odd number of Odd-factors, and Even number
of Even-factors.

19992016 Manhattan Review www.manhattanreview.com


20 Number Properties Guide Concepts

2.1.14 Multiples
While the factors divide the number completely, multiples are completely divisible by the num-
ber itself.

Example: The factors of 6 are 1,2,3 & 6; and the multiples of 6 are 6,12,18,24,30,...

=> A number is both a factor and a multiple of itself.

=> There are a finite number of factors of a number, but there are an infinite number of mul-
tiples.

In other words, we can write that the multiples of a number N = k . N , where k is a positive
integer. So the multiples of a number, say, xy 2 would be xy 2 , 2xy 2 , 3xy 2 , 4xy 2 , ... and the
factors of xy 2 would be {1, x, xy, y 2 , xy 2 }.

2.1.15 Lowest Common MultipleLCM


We all know what the LCM is; but occasionally we may get confused with HCF.

The LCM has three properties:

(1) It is a multiple; for numbers 6 & 10, the multiples of 6 are {6, 12, 18, 24, 30, 36, 42, 48, 54, 60, ....},
and the multiples of 10 are {10, 20, 30, 40, 50, 60, 70, 80, 90, ...}, so LCM is one among
these multiples.

(2) It is a common multiple among all the multiples of 6, & 10; for numbers 6 & 10, the
common multiples are {30, 60, 90, ....}, so it is one among these multiples.

(3) It is the least common multiple (LCM) among all the common multiples; for numbers 6
& 10, the least common multiple is 30: LCM.

So, in other words, LCM of numbers is the smallest possible number which gets divided by
each number without leaving any reminder.

Did you notice that in math, we never come across a termHighest Common Multiple? It
is because it is insignificant to discuss this as the Highest Common Multiple for any set of
numbers would always be infinite; multiples of numbers are never ending.

Computation of LCM by factor method:


Example: Computation of LCM of 24, 54, and 70:

Step1: Factorise the numbers into prime factors

=> 24 = 23 .3;
=> 54 = 2.33 ;
=> 70 = 2.5..7

www.manhattanreview.com 19992016 Manhattan Review


Number Properties Guide Concepts 21

Step2: Select the term with the highest exponent for each prime factor

=> The prime factors with their highest exponents are 23 , 33 , 5, & 7.

Step3: Multiply the numbers selected in step 2; it is the LCM of numbers.

=> LCM = 23 .33 .5.7 = 7560.

Computation of LCM of algebraic expressions:

Example: Computation of LCM of 10xy 2 z5 , 15x 2 y 3 z4 , & 24x 3 y 3 z:

=> 10xy 2 z5 = 2.5.x.y 2 .z z5;


=> 15x 2 y 3 z4 = 3.5.x 2 .y
y 3 .x 4 ;
3 3 3
=> 24x y z = 2 .3.x x .y 3 .z
3

LCM of 10xy 2 z5 , 15x 2 y 3 z4 , & 24x 3 y 3 z = 23 .3.5.x 3 .y 3 .z5 = 120x


x 3y 3z 5

19992016 Manhattan Review www.manhattanreview.com


22 Number Properties Guide Concepts

2.1.16 Highest Common FactorHCF OR Greatest Common DivisorGCD


Like LCM, the HCF/GCD also has three properties:

(1) It is a factor; for numbers 18 & 24, the factors of 6 are {1, 2, 3, 6, 9, 18}, and the factors
of 24 are {1, 2, 3, 4, 6, 8, 12, 24}, so HCF is one among these factors.

(2) It is a common factor among these factors; for numbers 18 & 24, the common factors
are {1, 2, 3, 6}, so it is one among these factors.

(3) It is the highest common factor (HCF) among all the factors; for numbers 18 & 24, the
highest common factor is 6: HCF.

So, in other words, HCF of numbers is the largest possible number which divides each number
without leaving any reminder.

Did you notice that in math, we never come across a termLowest Common Factor? It is be-
cause it is insignificant to discuss this as the Lowest Common Factor for any set of numbers
would always be 1.

Computation of HCF by factor method:

Example: Computation of HCF of 24, 54, and 210.

Step1: Factorise the numbers into prime factors

=> 24 = 23 .3;
=> 54 = 2.33 ;
=> 210 = 2.3.5.7.

Step2: Select the term with the lowest common exponent for each common prime factor

=> All the common prime factors with their lowest exponents are 2, & 3.

Step3: Multiply the numbers selected in step 2. It is the HCF/GCD of numbers.

=> HCF = 2.3 = 6.

Computation of HCF for algebraic expressions:

Example: Computation of HCF of 10xy 2 z5 , 15x 2 y 3 z4 , & 24x 3 y 3 z:

=> 10xy 2 z5 = 2.5.x y 2 .z5 = 2.5.((x ) .((y 2 ) .((z ) .z4 ;


x .y
=> 15x 2 y 3 z4 = 3.5.x 2 .y 3 .z4 = 3.5.((x ) .x.((y 2 ) .y.((z ) .z3 ;
=> 24x 3 y 3 z = 23 .3.x 3 .y 3 .z
z = 23 .3.((x ) .x 2 .((y 2 ) .y.((z )

HCF of 10xy 2 z5 , 15x 2 y 3 z4 , & 24x 3 y 3 z = x y 2 z .

www.manhattanreview.com 19992016 Manhattan Review


Number Properties Guide Concepts 23

LCM and HCF revisited:

x 3y 3z 5
LCM of N1 , N2 , N3 = 120x HCF of N1 , N2 , N3 = x y 2 z
Numbers:
N1 , N2 , N3 LCM is completely divisible by num- Numbers are completely divisible
bers by HCF

LCM 120x 3 y 3 z5 N1 10xy 2 z5


N1 = 10xy 2 z5 = = 12x 2 y = = 10z4
N1 10xy 2 z5 HCF xy 2 z

LCM 120x 3 y 3 z5 N2 15x 2 y 3 z4


N2 = 15x 2 y 3 z4 = = 8xz = = 15xyz3
N2 15x 2 y 3 z4 HCF xy 2 z

LCM 120x 3 y 3 z5 N3 24x 3 y 3 z


N3 = 24x 3 y 3 z = = 5z4 = = 24x 2 y 2
N3 24x 3 y 3 z HCF xy 2 z

19992016 Manhattan Review www.manhattanreview.com


24 Number Properties Guide Concepts

2.2 Exponents

4
Exponents, also called indices, are powers given to bases. Expo-

2
nents tell the number of times a number is multiplied by itself.
Exponent or
Base
index or power Example: 24 = 2 2 2 2 = 16. Here 2 is the base and 4 is
the exponent or index.

Here are some basic operations of indices:

Operations Examples

am an = am+n 23 25 = 23+5 = 28

(ab)m = am bm (2 3)4 = 24 34

am an = amn 23 25 = 235 = 22

3
(am )n = amn (22 ) = 22.3 = 26

1 1
= am = 23
am 23

1/3
am = (am )1/n = am/n
3 6
n
2 = (26 ) = 26/3 = 22 = 4

a0 = 1, if a 6= 0 20 = 1

(a)Even = +(a)Even ; (2)4 = 2 2 2 2 = 16


a = positive number

(a)Odd = (a)Odd ; (2)3 = 2 2 2 = 8 or


a = positive number

(2)3 = (2 2 2) = 8
n
Note that (am )n 6= am

3 2
(22 ) = 22.3 = 26 = 64; 23 = 29 = 512 6= 64

www.manhattanreview.com 19992016 Manhattan Review


Number Properties Guide Concepts 25

2.2.1 Unit digits of numbers with exponents

You may come across a question in which you are asked to deduce the unit digit of a number
with a positive integer exponent.

Example: What is the unit digit of 24 ?

We know that 24 = 16, so the unit digit is 6, however, if the question asks: What is the unit
digit of 217 ?, then it would be a time-consuming exercise to calculate the value of 217 .

Let us see the power cycle of 2. It is 2, 4, 8, 16, 32, 64, 128, 256, 512, 1024, 2048, ...... You may
observe that the unit digit is the same for every 5th indexed number in the series. For example,
the unit digit is 2 for 1st , 5th , 9th , 13th numbers in the series; while the unit digit is 8 for 3rd ,
7th , 11th numbers in the series. So, for Base, 2, the unit digit repeats after every 4th position,
i.e. there exists a cycle of 4 positions, so the power cycle of base 2 is 4.

Table showing the power cycles of the 10 digits:

Base Series Digits of Cycle Unit digit


cycle

0 0, 0, 0, 0, ... 0 1 0 for any exponent

1 1, 1, 1, 1, ... 1 1 1 for any exponent

2 2, 4, 8, 16, 32, 64, 128, 256, 512, ... 2, 4, 8, 6 4 repeats after every 4th
number

3 3, 9, 27, 81, 243, 729, ... 3, 9, 7, 1 4 repeats after every 4th


number

4 4, 16, 64, 216.... 4, 6 2 repeats after every 2nd


number

5 5, 25, 125, 525, ... 5 1 5 for any exponent

6 6, 36, 216, 1264, ... 6 6 6 for any exponent

7 7, 49, 343, 2401, 16807, ... 7, 9, 3, 1 4 repeats after every 4th


number

8 8, 64, 512, 4096, 32768, ... 8, 4, 2, 6 4 repeats after every 4th


number

9 9, 81, 729, ... 9, 1 4 repeats after every 2nd


number

Summary:

(1) The unit digits for the bases 0, 1, 5, & 6 are the bases itself.

19992016 Manhattan Review www.manhattanreview.com


26 Number Properties Guide Concepts

(2) The unit digits for the bases 2, 3, 7, & 8 follow the cycle of 4.

(3) The unit digits for the bases 4, & 9 follow the cycle of 2.

2.2.2 Deduction of unit digit


Example: What is the unit digit of 734 ?

Step1: Identify the base, its power cycle, and the exponent.
=> We know that the power cycle of the base, 7, is 4 (7,9,3,1), and the exponent is 34.

Step2: Divide the exponent by the power cycle, and get the remainder.
=> The remainder of 34/4 is 2.

Step3: Identify the digit of the power cycle corresponding to the remainder.
=> We know that the remainder is 2, and the power cycle of 7 is (7,9,3,1), hence the 2nd digit
of the power cycle would be the unit digit of 734 , which is 9: (7,9,3,1).

What if the questions had been: What is the unit digit of 736 ?

Again, we know that the power cycle of the base, 7, is 4 (7,9,3,1), and the exponent is 36 and
dividing 36 by 4 gives a remainder 0. If the remainder is 0, we must take the last number
in the power cycles which is 1. So the unit digit of 736 is 1.

Example: What is the unit digit of 72 .93 .32 ?

Step1: Deduce the unit digit of each number with exponent


=> The unit digit of 72 is 9; similarly, the unit digit of 93 is 9; similarly, the unit digit of 32
is 9.

Step2: Multiply all the unit digits deduced


=> The unit digit of 72 .93 .32 => 9.9.9 = 93 is 9.

Note: For solving questions asking for the unit digit, keep in mind that 5 multiplied with any
odd digit always results in a number ending with 5, while 5 multiplied with any even digit
always results in a number ending with 0.

www.manhattanreview.com 19992016 Manhattan Review


Number Properties Guide Concepts 27

2.2.3 Roots and Surds

Root is a familiar word; you have heard this beforesquare root, cube root etc.

Square root:

It is opposite of square. While square of a number means multiplying a number with itself to
get a new number, Square root means to get a number such that multiplying it with itself
would give the original number.

Example:
Say a number is x = 4, then the square = x 2 = 42 = 16, and square root =

2 2 2
x = 4 = 2; 2 2 = 4. Some consider that 4 = 2 or 2. However, it is not so. Its value is
+2 only.

The confusion starts from here:

If x 2 = 16, what is x?
2 2
While taking the square root of both the sides, we get x 2 = 16 => x = 4 => x = 4.

Note that 2 x = 2, not 2.

Remember that square of a number is NOT always greater than its square root. If a number

is x = 4, then the x 2 = 16, and x = 2; 16 > 2; however if a number is x = 1/4, then the

x 2 = 1/16, and x = 1/2; 1/16 < 1/2.

Square of a negative number is possible, but the square root is not possible,
it would be an
2
imaginary number. If a number is x = 4, then x 2 = 16, and x = 2; an imaginary
number.

Cube root:

Like square root, which is opposite of square, cube root is opposite of cube. Cube of a number
means multiplying a number with itself thrice to get a new number, Cube root means to get
a number such that multiplying it with itself thrice would give the original number.


3
Example: Say a number is x = 8, then the cube = x 3 = 83 = 512, and cube root = 3 x = 8 =
2; 2 2 2 = 8.

Like square, and its square root, cube of a number is NOT always greater than its cube root. If

a number is x = 8, then the x 3 = 512, and 3 x = 2; 512 > 2; however if a number is x = 1/8,

then the x 3 = 1/512, and 3 x = 1/2; 1/512 > 1/2.

Cube root gives unique value of x:

We have seen that if x 2 = 4, => x = 2.; no unique value of x; it may be either 2 or 2.


However if x 3 = 8, => x = 2; a unique value of x; because if x = 2, x 3 = 8. So cube root
gives the unique value. This concept is widely tested in the Data Sufficiency questions.

19992016 Manhattan Review www.manhattanreview.com


28 Number Properties Guide Concepts

nth root:

Like square root, and cube root, nth root means multiplying a number with itself n times to

get the original number. Say a number is x, then its nth root would be n x.

5
Example: Say a number is x = 32, 32 = 2; 2 2 2 2 2 = 32.

2.2.3.1 Surds

If the root of a number renders an irrational number, its exponent is called Surd.

2
The square root of 2, 2 = 1.414; an irrational number, hence a surd.

You must know a few values: 2 = 1..414,, 3 = 1..732,, 5 = 2..236.

2.2.3.2 Rationalization of surds

Keeping a surd in the denominator is not a standard way of writing a number or expression in
3
mathematics. Surds should be placed in the numerator. If a number is 2 , we can get rid of

2 placed in the denominator by multiplying and dividing the fraction by 2.
 
3 3 2 3 2 3 2
=> = = 2 = .
2 2 2 ( 2) 2

2+1
Example 1: Rationalize .
21

We cannot multiply the numerator and the denominator
( 2 1)
by to remove the surd
in the
denominator; the denominator will result into ( 2 1)2 = ( 2)2 2. 2.1 + 12 = 2 2 2 + 1 =

3 2 2; again we would have a surd.

The optimum approach to do this is to make use of the formula (a b)(a + b) = a2 b2 .


2+1
Assume a = 2, and b = 1. By multiplying to the number, we will have a pair of
2+1
( 2 1)( 2 + 1) in the denominator, which will give [( 2)2 12 ] = 2 1 = 1; thus the surd
is removed.

The term ( 2 + 1) is called the conjugate of ( 2 1).

2+1 2+1 ( 2 + 1)2 ( 2 + 1)2

=> = = = ( 2 + 1)2 .
21 2+1 ( 2)2 12 1


x y
Example 2: Rationalize .
x+ y
 
x y x y ( x y)2 ( x y)2
=> = 2 2 = ; ( x y) is the conjugate of
x+ y x y ( x) + ( y) xy
( x + y).

www.manhattanreview.com 19992016 Manhattan Review


Number Properties Guide Concepts 29

Key: If the expression in the denominator has + sign, then multiply and divide the number
with its conjugate, i.e. an expression with sign and vice-versa. (x y) and (x + y) are
conjugates of each other, provided x is a rational number and y is a surd. .

2.3 Multiplication and Division of decimals with 10x

5482.75
We are often faced with multiplications such as 20.0987103 or . Follow the following
104
approach to do such operations in less time.

Scenario 1: Multiplication

Here, we either multiply a number by a number having a positive exponent of 10 or divide it


with a negative exponent of 10.

Step 1: Move the decimal to its right for the same number of times as the exponent value

Step 2: If number of digits to the right of the decimal is less than the value of the exponent,
add requisite number of 0s after the last digit of the number

Example 1: 20.0987 103

The value of the exponent is 3. By moving the decimal to the right 3 times in the number
20.0987, we will get the number 20098.7.

20.0987
20098.7
Example 2: 20.09 104

As per the process described in the example above, we move the decimal to the right 4 times
in the number 20.09. Since the number has only two digits after the decimal, we will add two
0s after 9. Thus, 20.09 104 = 200900.

Example 3: 20.0987 103

The value of the exponent is 3. Let us first make it positive. We can bring 103 in the
numerator, thus, 20.0987 103 = 20.0987 103 . Now the situation is the same as was in the
example 1. By moving the decimal to the right 3 times in the number 20.0987, we will get the
number 20098.7.

19992016 Manhattan Review www.manhattanreview.com


30 Number Properties Guide Concepts

Scenario 2: Division
Here, we either divide a number by a number having a positive exponent of 10 or multiply with
a negative exponent of 10.

Step 1: Move decimal to its left for the same number of times as the exponent value

Step 2: If number of digits to the left of the decimal is less than the value of the exponent, add
requisite number of 0s before the first digit of the number

Example 3: 20.0987 103

The value of the exponent is 3. By moving decimal to the left 3 times in the number 20.0987,
we will get number the 0.0200987.

20.0987
0.0200987
Example 4: 20.0987 103 ?

20.0987
First, convert division into multiplication. So, 20.0987 103 = 103 = 20.0987 103 . Now
the question fits into scenario 1. Here, the value of the exponent is 3. By moving decimal to
the right 3 times in the number 20.0987, we will get the number 20098.7.

In a nut shell, following numbers are equal; 20.0987 103 = 2.00987 104 = 0.200987 105 =
20.0987 200.987 2009.87
200.987 102 = 20098.7 = 200987 101 . Similarly, 100 = 1000 = 104 .

Example 5: 3, 000, 000 0.0003?

To transform the second number, 0.0003, equals to 3 times the exponent of 10, it needs an
exponent of 4 to the base 10 or the multiplication of 10,000; we can borrow four 0s from
the first number, 3,000,000, so 3, 000, 000 0.0003 becomes 3000000
 3 equalling 900.

3000000 x 0.0003
1.96[103 (6.03 3 102 )]
Example 6: =?
(1.4)2

1.96[103 (6.03 3 102 )] 1.96[10


 3 (6.03 3 100)]
= 103 (6.03 3 100)

=> =
(1.4)2 1.96
 

=> 103 (6.03 0.03) = 1000 6 = 6000.

www.manhattanreview.com 19992016 Manhattan Review


Number Properties Guide Concepts 31

2.4 Common Binomial expressions


(1) a(b + c) = ab + ac
150 51 = 150(50 + 1) = 150 50 + 150 1 = 7500 + 150 = 7650
38 19 = 38(20 1) = 38 20 38 1 = 760 38 = 722

(2) (a + b)2 = a2 + 2ab + b2


1022 = (100 + 2)2 = 1002 + 2.100.2 + 22 = 10000 + 400 + 4 = 10404

(3) (a b)2 = a2 2ab + b2


982 = (100 2)2 = 1002 2.100.2 + 22 = 10000 400 + 4 = 9604

(4) (a2 b2 ) = (a + b)(a b)


982 22 = (98 + 2)(98 2) = 100 96 = 9600

(5) (a + b + c)2 = a2 + b2 + c 2 + 2(ab + bc + ca)

(6) (a + b)3 = a3 + 3a2 b + 3ab2 + b3


1023 = (100 + 2)3 = 1003 + 3.1002 .2 + 3.100.22 + 23 = 1000000 + 60000 + 1200 + 8 =
1061208

or (a + b)3 = a3 + 3ab(a + b) + b3

513 = (50 + 1)3 = 503 + 3.50.1.(50 + 1) + 13 = 125000 + 150.51 + 1 = 132651

(7) (a b)3 = a3 3a2 b + 3ab2 b3


983 = (1002)3 = 1003 3.1002 .2+3.100.22 23 = 100000060000+12008 = 941192

or (a b)3 = a3 3ab(a b) b3

493 = (50 1)3 = 503 3.50.1.(50 1) 13 = 125000 150.49 1 = 117649

(8) (a3 + b3 ) = (a + b)(a2 ab + b2 )


113 + 93 = (11 + 9)(112 11.9 + 92 ) = 20.(121 99 + 81) = 20.103 = 2060

(9) (a3 b3 ) = (a b)(a2 + ab + b2 )


113 93 = (11 9)(112 + 11.9 + 92 ) = 2.(121 + 99 + 81) = 2.301 = 602

19992016 Manhattan Review www.manhattanreview.com


32 Number Properties Guide Concepts

2.5 Some important summation of series


n(n + 1)
(1) Sum of first positive integers = 1 + 2 + 3 + 4 + 5 + ............n =
2

Example: 12 + 13 + 14 + 15 + 16 + 17 + 18 =?

12 + 13 + 14 + 15 + 16 + 17 + 18 = (1 + 2 + 3 + 4 + .......18) (1 + 2 + 3 + 4 + ........11)

18(18 + 1) 11(11 + 1) 18.19 11.12


= = 171 66 = 105
2 2 2 2

Alternate Approach:

=> 12 + 13 + 14 + 15 + 16 + 17 + 18 = (11 + 1) + (11 + 2) + (11 + 3) + (11 + 4) + (11 +


5) + (11 + 6) + (11 + 7)

7.8
 
=> (11+11+11+11+11+11+11)+(1+2+3+4+5+6+7) = 77+ = 77+28 = 105
2
n(n + 1)(2n + 1)
(2) Sum of squares of first positive integers = 12 +22 +32 +42 +52 +............n2 =
6

n(n + 1) 2
 
3 3 3 3 3 3
(3) Sum of cubes of first positive integers = 1 + 2 + 3 + 4 + 5 + ............n =
2
(4) Sum of odd integers = 1 + 3 + 5 + ............(2n 1) = n2 ; Note that the nth term is (2n 1).

(5) Sum of even integers = 2 + 4 + 6 + ............2n = n(n + 1)

www.manhattanreview.com 19992016 Manhattan Review


Number Properties Guide Concepts 33

2.6 Fractions
Fraction is a number which is formed out of numerator and denominator. It is represented as
Numerator
. Both numerator and denominator are integers, but the denominator should not
Denominator
be 0.

3 7 9
Example: , , etc.
4 8 5
Proper fraction: For a fraction, if the absolute value of the numerator is less than the absolute
value of the denominator, it is called a proper fraction. Example: 43 , 78 , 10
9
etc.

Improper fraction: If the absolute value of the numerator of a fraction is greater than the
absolute value of the denominator, it is called improper fraction. Example: 43 , 11 9
8 , 5 etc..

Mixed fraction: If the fraction is a mix of both an integer and proper fraction, it is called mixed
3 2
fraction. Example: 2 4 , 5 3 etc.

Converting mixed fraction to improper fraction: Say, a mixed fraction is 5 32 .

Step 1: Multiply the integer part with the denominator. 5 3 = 15.


Step 2: Add the numerator to the number attained in step 1. So, 15 + 2 = 17. It is
the numerator of improper fraction.
new numerator 53+2 17
Step 3: The equivalent improper fraction is = =
denominator 3 3

Equivalent Fraction: Fractions that are equal in values are called equivalent fractions, although
the numerator and denominator of the equivalent fractions may not be the same.

Example: 1/2 = 2/4 = 3/6 = 4/8 = 5/10.

19992016 Manhattan Review www.manhattanreview.com


34 Number Properties Guide Concepts

2.6.1 Concept of fraction


Let us understand the concept of fraction through graphical representation. Fraction is a part
of a whole. For example, the fraction 3/8 means that out of 8 parts, only 3 parts are taken; it
can also be thought of as eating 3 slices of pizza of an 8-slice pizza.

3/8

Reciprocal of fraction: You get reciprocal of a fraction when you interchange numerator and
3 8
denominator. Example: Reciprocal of 8 is 3 .

8
What is the significance of ?
3
To understand this, first convert 38 into mixed fraction, which is 2 23 . It means eating 2 full
pizzas and 2 slices of pizza of a 3-slice pizza.

8/3 = Reciprocal of 3/8

Decimal Representation:

A fraction can be represented as a decimal number by dividing the numerator by the denomina-
tor. For example, the fraction 3/8 represents the numerical value 0.375; it is a decimal number.

Percent Representation:

A fraction can be represented as a percent by dividing the numerator by the denominator and
then multiplying it by 100. For example, 3/4 = 0.75 => 0.75 100% = 75% and similarly,
1/2 = 50%.

www.manhattanreview.com 19992016 Manhattan Review


Number Properties Guide Concepts 35

2.6.2 Mathematical operations on fractions

Mathematical Example
operation

1 1
Addition/ 2 + 2 = 1; two halves makes 1;

1 1 12 1 2 1 21
Subtraction 2 4 =? => 22 4 = 4 4 = 4 = 41 ; make denominators equal

1 1
2 2 = 14 ; half of a half makes it 14 ;
Multiplication
3 15
5 6 = 32 ; any numerator can cancel any denominator;

two numerators will be multiplied, and similarly, two denominators will


be multiplied

1 1 1 2
2 2 = 2 1 = 1;

will change to and the fraction to the right will become its recipro-
cal;
Division
3
5 35 = 35 53 = 1; a fraction divided by itself is always 1; product of a
fraction with its reciprocal is always 1.

Some confusing divisions:

2/3 2 3 2 2 4
= = = ;
3/2 3 2 3 3 9
2 3 2 4
=2 =2 = ;
3/2 2 3 3
2/3 2 2 1 2
whereas = 3= =
3 3 3 3 9

2
Example: =?
2
2
4
1
5
2
6
2+
7

=> Start solving from the lowermost fraction

19992016 Manhattan Review www.manhattanreview.com


36 Number Properties Guide Concepts

2 2 2
=> = = ;
2 2 2
2 2 2
4 4 4
1 1 1
5 5 5
2 2 2
6 27+6 20
2+
7 7 7

Remember that the primary division operator is between 5 and 20.


5
5 20 2 7 7 20 5
So 20 = 5 =5 = = ; Do not get it confused with 6= .
7 3 20 4 7 20
7
7
Coming back to the question:

2 2 2 2 2 2
=> = = = = =
2 2 2 2 2 2
2 2 2 2 2 2
4 4 4 144 1 16 15
1 1 1
5 75 1
2 2
20 20 4
7

2 2 2 15 15
=> = = =
2 32 30 16
2+
15 15

2.6.3 Comparing fractions

You may come across situation where you have to compare the two fractionswhich fraction
is greater or smaller?

A general rule: For positive factions, larger the numerator, greater is the fraction (with the
same denominator), and smaller the denominator, greater is the fraction (with the same nu-
merator) and vice-versa; opposite is true for negative fractions.

1 1
Example 1: Which is greater 2 or 3 ? Well, this is a simple one as the numerators of the frac-
tions are equal. For positive factions, if the numerators of the fractions are equal, the fraction
with smaller denominator is greater, so 12 > 13 ; however 12 < 31 ; 12 < 13 ; 21 > 13 .

Example 2: Which is greater 25 or 35 ? Again, this is a simple one as the denominator of the
fractions are equal. For positive factions, if the denominators of the fractions are equal, the
fraction with the larger numerator is greater, 52 < 35 ; however 25 > 35 ; 25 > 35 ; 52 > 53 .

Example 3: Which is greater 11 9


15 or 16 ? This is the easiest of all to deduce. As the numerator
of the first fraction is greater than that of the other and its denominator is smaller than that
of the other, so on both the counts, the first fraction scores over the other, hence it is greater
than the second fraction, so 11 9 11 9
15 > 16 ; however 15 < 16 .

www.manhattanreview.com 19992016 Manhattan Review


Number Properties Guide Concepts 37

Example 4: Which is greater 57 or 79 ? This is not an easier one to deduce. The numerator of
the first fraction is greater than that of the other but its denominator is also larger than that
of the other, so on one count, the first fraction scores over the second and on the other count,
the second one scores over the first. So how to deduce which fraction is greater? There are
three approaches to deduce this.

Approach 1:
Convert fractions to Decimals

Each fraction can be converted into a decimal by dividing the numerator by the denominator.
In decimal format, the value of each fraction can be compared.

5 7
=> = 0.714; = 0.777, so 0.777 > 0.714.
7 9
Since the decimal equivalent of 7/9 is larger than that of 5/7, so 7/9 > 5/7.

Approach 2:
Take 50%

The approach is to take 50% of the denominator and deduct it from the numerator, and then
compare the remaining fractions.

5 4
Which is greater 7 or 5 ?

=> 50% of 7 (denominator of thefirst fraction)is 3.5,


 and 50% of 5
 (denominator of the second
5 3.5 1.5 4 2.5 1.5
fraction) is 2.5, so we can write 7 = 7 + 7 <> 5 = 5 + 5

1.5 1.5 1.5 1.5 5


=> (50% + 7 ) <> (50% + 5 ) => It is obvious that 7 < 5 , so, 7 < 45 .

This approach may not be applicable in each case; after seeing all the approaches, you should
decide which approach works best for a specific scenario.

Approach 3:
Cross Multiplication

Another approach to compare fractions is: Cross Multiplication. Numerator of the first frac-
tion is multiplied to the denominator of the second fraction, and numerator of the second
fraction is multiplied to the denominator of the first fraction. Now compare the results, and
identify the greater of the two products; the fraction which has its numerator in the greater of
the two products is a greater fraction than the other fraction.

5 7
=> Which is greater or ?
7 9
=> (Numerator of I Denominator of II) = 5 9 = 45; (Numerator of II Denominator of I) =
7 7 = 49, since 49 > 45 and 49 includes the numerator of the second fraction, hence the
second fraction is greater than the first fraction.

19992016 Manhattan Review www.manhattanreview.com


38 Number Properties Guide Concepts

2.6.4 More on fractions


a a+c a
(1) If > 1, then < ; where a, b, & c are positive numbers
b b+c b
a 3 a+c 3+1 4 a 3
 
=> Say = , and c = 1, then = = = 1.33 < = = 1.5
b 2 b+c 2+1 3 b 2

We can understand it this way: Since the fraction a/b > 1, it means that a > b. When we
add c to each a and b, larger proportion of c is added to b i.e. denominator and
relatively a smaller proportion of c is added to a i.e. the numerator. Let us see how.

c c
   
a 1+ 1 + smaller fraction compared to " #
a+c a a b a smaller number
=   = = <
c c
 
b+c b b larger number
b 1+ 1 + larger fraction compared to
b a
a
b

a a+c a
(2) If 0 < < 1, then > ; where a, b, & c are positive numbers
b b+c b
a 2 a+c 2+1 3 a 2
 
=> Say = , and c = 1, then = = = 0.75 > = = 0.667
b 3 b+c 3+1 4 b 3

a ac a
(3) If > 1, then > ; where a, b, & c are positive numbers
b bc b
a 3 ac 31 2 a 3
 
=> Say = , and c = 1, then = = =2> = = 1.50
b 2 bc 21 1 b 2

a ac a
(4) If 0 < < 1, then < ; where a, b, & c are positive numbers
b bc b
a 2 ac 21 1 a 2
 
=> Say = , and c = 1, then = = = 0.50 < = = 0.667
b 3 bc 31 2 b 3

www.manhattanreview.com 19992016 Manhattan Review


Number Properties Guide Concepts 39

2.6.5 Fractions and their percent equivalents

Fraction Percent Fraction Percent

1 1
100% 11.11%
1 9

1 1
50% 10%
2 10

1 1
33.33% 9.09%
3 11

1 1
25% 8.33%
4 12

1 1
20% 4%
5 25

1 1
16.67% 2%
6 50

1 1
14.28% 1%
7 100

1 1
12.50% 0.50%
8 200

Application: Sometimes you may come across calculating an odd-looking percent of an odd-
looking number. The fraction to percent equivalent table may help you make the calculations
easier.

Example 1: What is 14.25% of 705?

The traditional way of calculation is time-consuming: 14.25


100 705 = 100.47. In the GMAT,
seldom you need to calculate the exact value; the options would be wide apart. GMAT does not
test your ability on how precisely you calculate the values, it rather tests your ability on how
quickly you attain the meaningful value.

Smarter approach:
1
From the table, we know that 14.25% is 14.28% which is equal to 7. So, we can rather calcu-
1
late 7 of 705; again, we can approximate 705 to 707 as there is a difference of only 2 between
705 and 707. So, 17 of 707 is easier to calculate which is equal to 101. But 101 is an approxi-
mate value, the actual value would be less than 101 as we increased 14.25% to 14.28% and 705

19992016 Manhattan Review www.manhattanreview.com


40 Number Properties Guide Concepts

to 707. The actual value already calculated as 100.47, which is quite close to the approximate
value 101.

Example 2: What is 12.61% of 802?

A. 95.68

B. 105.94

C. 101.13

Again, from the table, we know that 12.61% is 12.50% which is 18 . So, we can rather calculate 18
of 802; again, we can approximate 802 as 800 as there is a difference of only 2 between them.
So, 1/8 of 800 is easier to calculate which is equal to 100. Again, 100 is an approximate
value, the actual value must be little more than 100 as we had decreased 12.61% to 12.50%
and 802 to 800. Looking at the options, we can safely lock 101.13 as the actual value since
the other higher value is far greater than 100.

2.6.6 See-Saw Approach

How do you solve a question which asks If gasoline price increased by 10%, by what percent
should one reduce its consumption so that the expense on gasoline remains the same?

Traditional Approach:

Say, before the increase, the price of gasoline was $100/gallon, and the consumption was 100
gallons. So, the expense = 100 100 = $10000.

Now, after the increase, the price of gasoline is $110/gallon (10% up), and say the consumption
is reduced to x gallons. So, the expense = 110 x = $110x.

Since the expense needs to be the same as before, hence 10000 = 110x => x = 90.90 gallons

100 90.90
=> Percent reduction in consumption = 100% = 9.09%.
100

See-Saw Approach:

Scenario 1: When x y = C; where x is one element, y is second element, and C is a constant.

If x goes up by certain percent and y must go down with some percent such that after the
change x y = x 0 y 0 = C, follow the following short-cut steps to get the answer. Where
after the change, x becomes x 0 , and y becomes y 0 .

1
Step 1: Covert the given percent figure into fraction, say it is a ; you should take the help of
the conversion table.
1
Step 2: Add 1 to the denominator of the fraction attained; it would be a+1 .
1
Step 3: Convert a+1 fraction into percent; it is the requisite answer; you should take the help

www.manhattanreview.com 19992016 Manhattan Review


Number Properties Guide Concepts 41

of the table.

Let us get the answer to the above question with the See-Saw approach.

1 1 1
Step 1: Price goes up by 10%; equivalent fraction of 10% is 10 , so a = 10 .
1 1 1
Step 2: By adding 1 to the denominator of the fraction a, we get a+1 = 11 .
1
Step 3: With the help of the table, we get 11 = 9.09%; it is the requisite answer. The consump-
tion must go down by 9.09%.

Pl. see the pictorial representation of the See-Saw approach.

1 1
Consumption will go down by = a+ 1 = 11
= 90.09%

1 1
Price goes up by 10%= =
10 a

1 1
a a+1

Scenario 2: When x y = C; where x is one element, y is second element, and C is constant.

If x goes down by certain percent and y must go up with some percent such that after the
change x y = x 0 y 0 = C, follow the following short-cut steps to get the answer.

1
Step 1: Covert the given percent figure into fraction, say it is a;
take the help of the table.
1
Step 2: Subtract 1 from the denominator of the fraction attained; it would be
a1
1
Step 3: Convert fraction into percent; it is the requisite answer; you should take the help
a1
of the table.

Let us apply the See-Saw approach in another question: If milk price decreased by 10%, by what
percent should one increase its consumption so that the expense on milk remains the same?

1 1 1 1
Step 1: Price goes down by 10%; equivalent fraction of 10% is 10 , 10 , so a = 10 ;
1
Step 2: By subtracting 1 from the denominator of the fraction a1 , we get = 19 .
a1
1
Step 3: With the help of the table, we get 9 = 11.11%; it is the requisite answer. The consump-
tion must go up by 11.11%.

Pl. see the pictorial representation of the same.

19992016 Manhattan Review www.manhattanreview.com


42 Number Properties Guide Concepts

1 1
Price goes down by 10% = 10 = a

Consumption will go up by
1 1
= = = 11.11%
a1 9
1 1
a a1

Example 1: If the sale of a brand is reduced by 17%, by what percent should the company
increase its sales price such that the revenue earned remains the same as before?

A. 14.28%

B. 14.53%

C. 20%

D. 20.48%

First, identify whether the question fits into the category discussed above. Since Quantity
Price = Revenue, and the condition is that the revenue must remain constant, hence the
question fits into the See-Saw approach model. Out of the two scenarios, the second scenario
fits as an element goes down and the complementary element must go up.

When we look at the table, we do not find 17%, however we can safely assume that it is closer
1
to 16.67%; so the equivalent of 17% = 16.67% = 6 = a1 =>
1
= 15 = 20%. The sales price
a1
must be increased by 20%. The correct answer would be a little more than 20% as we approx-
imated 17% to 16.67%. So, the correct answer must be option Dthe only greater option than
20%. The company should increase the sales price of the brand by 20.48%.

Example 2: If the price of milk increased by 11%, by what percent should its consumption be
reduced such that the expenses on milk remains the same as before?

We identified that the question fits into the category discussed above, and it fits into the first
1 1 1 1
scenario. Now, 11% = 11.11% = 9 = a => a+1 = 10 = 10%.. The correct answer would be a
little less than 10%.

www.manhattanreview.com 19992016 Manhattan Review


Number Properties Guide Concepts 43

2.7 Absolute numbers

We know that any real number can either be 0, positive or negative. When we ignore the neg-
ative sign of a number, we seem to be interested in its absolute value and not in its directed
value.

In other words, absolute value of a number is its distance from 0. If a number is presented
between two-vertical bar (pipe ||) symbol, it means we are interested in its absolute value.

Say, a number is x, then its absolute value is |x|, as depicted in the figure below.

|x| = 3
3 3

5 4 3 2 1 0 1 2 3 4 5 +

Key properties:


(1) x 2 = |x|

(2) | x| = |x|

(3) |x| = | x|; |x| 6= | x|

(4) |x| 0

(5) |x y| = |y x|

Say x = 10 and y = 10, then


|x y| = |10 (10)| = |10 + 10| = 20;
|y x| = | 10 10| = | 20| = 20.

(6) |x + y| |x| + |y|

Say x = 10 and y = 10, then


|x + y| = |10 + 10| = 20;
|x| + |y| = |10| + |10| = 10 + 10 = 20, hence |x + y| = |x| + |y|

But, say x = 10 and y = 10, then


|x + y| = |10 + (10)| = |10 10| = 0;

19992016 Manhattan Review www.manhattanreview.com


44 Number Properties Guide Concepts

|x| + |y| = |10| + | 10| = 10 + 10 = 20, hence |x + y| < |x| + |y|

This deduction is an important finding and is tested by test-makers especially


in DS questions.

Remember that the equality holds true only when both x and y are of the same
sign and the inequality holds true when x and y are of opposite signs.

(7) |x y| |x| |y|

Here too, the equality holds true only when both x and y are of the same sign
and the inequality holds true when x and y are of opposite signs.

(8) |x y| = |y x| = |x| |y| = |y| |x|

(9) |x y| = |x| |y|

Example: If 7|2x 3| = 21, what is the value of x?

Simplify 7|2x 3| = 21 by dividing both the sides by 7; we get |2x 3| = 3.

=> 2x 3 = 3 or (2x 3) = 3.

=> 2x 3 = 3 => 2x = 6 => x = 3; or

=> (2x 3) = 3 => 2x + 3 = 3 => 2x = 0 => x = 0.

So x = 0 or 3.

www.manhattanreview.com 19992016 Manhattan Review


Number Properties Guide Concepts 45

2.7.1 Inequality
To understand Inequality, first understand Equality.

x = 5 means that the value of the variable x equals 5.

The inequality could be anything x 6= 4, x < 6 or x > 4.

When the two sides of an expression are not equal, it is called Inequality. Inequalities use
symbols <, >, , or to describe the relationship between two expressions, for example: 10 <
12, 12 > 8, x 4, x + 2y 4 etc.

Understanding inequality:

Inequality Meaning Example


x can be any value less than 5 on the number line; it
can be 0, 1, 155, or any other value up to negative
infinity.
x<5 x is less than 5 Do not assume that x is an integer. x can even be
4.999.
Always consider that x is a real number until stated
that it is an integer.
y > 10 y is greater than 10 y can be any value greater than 10 on the number
scale. It can be 10.0001, 10.56, 21, or any other
value up to infinity.
p 10 p is less than or p can be any value less than or equal to 10 on the
equal to 10 number line. It can be 10, 4, 2.3, or any other
value up to negative infinity.
q 10 q is greater than or q can be any value greater than or equal to 10 on
equal to 10 the number line. It can be 10, 14, 23.5, or any other
value up to infinity.
2 x < 4; x is greater than or x = {2, 1, 0, 1, 2, 3}
x is an integer equal to 2 but less
than 4
2 > x 4; x is less than 2 x = {3, 4}
x is an integer and greater than or
equal to 4

19992016 Manhattan Review www.manhattanreview.com


46 Number Properties Guide Concepts

Representation of inequalities on number line:

3 < x 4<x<2

5 4 3 2 1 0 1 2 3 4 5 + 5 4 3 2 1 0 1 2 3 4 5 +

Key properties of Inequalities:

(1) You can add or subtract anything from an inequality.

Say, x + 3 > 10.

You can subtract 3 from both the sides, so the inequality becomes,
x + 3 3 > 10 3 => x > 7.

Take another example; say, y 3 > 10.

You can add 3 on both the sides, so the inequality becomes,


y 3 + 3 > 10 + 3 => y > 7.

(2) You can multiply or divide an inequality by a positive number.

Say, 2x > 10.

You can divide the inequality by 2, so the inequality becomes,


2x 10
> => x > 5.
2 2
y
Say, > 10. You can multiply the inequality by 3, so the inequality becomes,
3
y
3 > 10 3 => y > 30.
3
(3) You can multiply or divide an inequality by a negative number, but doing so will ask for
reversing the sign of inequality.

We know that 7 > 5;

if we multiply an inequality by 1, it will become 7 > 5 which is wrong to


say because 7 < 5; so when we multiply or divide an inequality with a nega-
tive number, we have to flip the sign.

Say, 2x > 10.

You can divide the inequality by 2, so the inequality becomes,

www.manhattanreview.com 19992016 Manhattan Review


Number Properties Guide Concepts 47

2x 10
< => x < 5.
2 2
y
Say, > 10. You can multiply the inequality by 3, so the inequality be-
3
comes,

y
3 < 10 3 => y < 30.
3

2.7.2 Inequalities with absolute numbers

Few tough questions may be asked combining inequalities and absolute numbers.

Example 1: If |x + 1| 4, what is x?

|x + 1| 4

=> If x + 1 4 or (x + 1) 4

x + 1 4 => x 3

Or, (x + 1) 4 => x + 1 4 => x 5. (Pay attention to reversal of sign of inequality)

So, 3 x 5

It can be represented graphically as below.

3 x 5

5 4 3 2 1 0 1 2 3 4 5 +

Example 2: If |2x + 1| 3, what is x?

|2x + 1| 3 => 2x + 1 3 or (2x + 1) 3

If 2x + 1 3 => 2x 2 => x 1

Or, (2x + 1) 3 => 2x + 1 3 => 2x 4 => x 2. (Pay attention to reversal of sign


of inequality)

So 2 x or x 1

It can be represented graphically as below.

19992016 Manhattan Review www.manhattanreview.com


48 Number Properties Guide Concepts

2 x x1

5 4 3 2 1 0 1 2 3 4 5 +

Example 3: If 8 + x 2 12, what is the range of x?

8 + x 2 12

=> x 2 4;

Taking square root of both the sides, we get,

|x| 2 => x 2 or x 2.

2 x x2

5 4 3 2 1 0 1 2 3 4 5 +

2.7.3 Compound Inequality


An inequality that has more than one inequality symbol in an expression is called compound
inequality. 2 > x 4 is an example of compound inequality. GMAT inequality questions may
involve more than one inequality. To solve such questions, you may have to transform several
inequalities to a compound inequality.

Example 4: If x > 4, x 10, and x < 12, what is the range of x?

Step 1: Fix more than x or more than or equal to x inequality.

There is only one more than x inequality, so we fix x > 4.

Step 2: Fix less than x or less than or equal to x inequality.

There is one less than x and one less than or equal to x inequalities, but between
the two inequalities x 10, and x < 12, we choose the first one i.e. x 10 since
x 10 is a subset of x < 12, not vice-versa; so we fix x 10 .

www.manhattanreview.com 19992016 Manhattan Review


Number Properties Guide Concepts 49

Step 3: Combine both the inequalities derived in steps 1 and 2.

Finally, we get 4 < x 10.

4< x 10

3 4 5 6 7 8 9 10 11

Note that:

If x > y, and y > z, then we can combine the inequalities as x > y > z or x > z.

However,

If x > y, and y < z, then though we can combine the inequalities as x > y < z,
but we cannot deduce that x > z.

2.7.4 Mathematical operations on inequalities


(1) Addition and subtraction:

If x > y and a < b, we cannot deduce that x + a < y + b; it is wrong.

We must make both the inequalities in the same direction. So, we can rewrite the inequal-
ities as x > y and b > a, and now we can add these.

So, x + b > y + a .

Lets take another example.

If x 3 < y x + 6, then we cannot write the inequality as x < y x + 6 + 3 => x <


y x + 9. It is wrong!

If you subtract a number from one part of a compound inequality, you must subtract the
same number from all the parts.

The simplified and corrected inequality is: x < y + 3 x + 9.

19992016 Manhattan Review www.manhattanreview.com


50 Number Properties Guide Concepts

(2) Multiplication and division:

y
If x < 3 3x, then we can simplify the inequality by multiplying it by 3.
 
y y
x > y 9x
So, 3 x < 3 3x => 3x > ( 3 3) 3 3x => 3x x.

Always be wary of changing the sign of inequality when multiplying or dividing it by a


negative number.

x < y 9x
The inequality can also be written as 3x x.

Similarly, if 3x > 6y 3 > 6z, we can divide the inequality by 3, so we get x > 2y
y 1 >
z . Always remember that each part of the compound inequality should be divided.
2z

www.manhattanreview.com 19992016 Manhattan Review


Number Properties Guide Concepts 51

Problem Solving
Problem solving (PS) questions may not be new to you. You must have seen these types of
questions in your school or college days. The format is as follows: There is a question stem
and is followed by options, out of which, only one option is correct or is the best option that
answers the question correctly.

PS questions measure your skill to solve numerical problems, interpret graphical data, and
assess information. These questions present to you five options and no option is phrased as
None of these. Mostly the numeric options, unlike algebraic expressions, are presented in an
ascending order from option A through E, occasionally in a descending order until there is a
specific purpose not to do so.

Data Sufficiency
For most of you, Data Sufficiency (DS) may be a new format. The DS format is very unique to the
GMAT exam. The format is as follows: There is a question stem followed by two statements,
labeled statement (1) and statement (2). These statements contain additional information.

Your task is to use the additional information from each statement alone to answer the ques-
tion. If none of the statements alone helps you answer the question, you must use the infor-
mation from both the statements together. There may be questions which cannot be answered
even after combining the additional information given in both the statements. Based on this,
the question always follows standard five options which are always in a fixed order.

A. Statement (1) ALONE is sufficient, but statement (2) ALONE is not sufficient to answer the
question asked.

B. Statement (2) ALONE is sufficient, but statement (1) ALONE is not sufficient to answer the
question asked.

C. BOTH statements (1) and (2) TOGETHER are sufficient to answer the question asked, but
NEITHER statement ALONE is sufficient to answer the question asked.

D. EACH statement ALONE is sufficient to answer the question asked.

E. Statements (1) and (2) TOGETHER are NOT sufficient to answer the question asked, and
additional data specific to the problem are needed.

The book contains 200 PS & DS questions. These have been picked wisely so that you apply PS
& DS techniques and approaches in an effective and efficient way.

There is a dedicated book on Data Sufficiency; it discusses various DS strategies, techniques


and approaches.

In the next chapters, you will find 200 GMAT-like quants questions. Best of luck!

19992016 Manhattan Review www.manhattanreview.com


52 Number Properties Guide Concepts

www.manhattanreview.com 19992016 Manhattan Review


Chapter 3

Practice Questions

53
54 Number Properties Guide Questions

3.1 Problem Solving

3.1.1 Numbers & Digits

1. If x and y are integers, and 5x + 3y = 15, which of the following could NOT be the value
of y?

(A) 5
(B) 0
(C) 3
(D) 5
(E) 1005

Solve yourself:

m
2. If is an integer, which of the following statements can be true EXCEPT?
n
(A) Both m and n are integers
(B) m is an integer
(C) Either m or n is negative
n
(D) is an integer
m
(E) m = kn; where k is non-integer

Solve yourself:

nm
3. If m # n = , then what is the value of (4 # 3) # 1?
2
(A) 1

(B) 6
(C) 3
(D) 6
(E) 9

www.manhattanreview.com 19992016 Manhattan Review


Number Properties Guide Questions 55

Solve yourself:

4. There are 10 integers; out of these, at least two integers are negative. If the product of
these integers is negative, at most how many of the integers can be negative?

(A) Two
(B) Three
(C) Five
(D) Nine
(E) Ten

Solve yourself:

5. If abc = 1, cde = 0, and ebc = 0, which of the following must be 0?

(A) a
(B) b
(C) c
(D) d
(E) e

Solve yourself:

6. If (5x + 11) is a positive integer, for how many integer values of x will the number
(5x + 11) be less than 300?

(A) 56
(B) 57
(C) 58

19992016 Manhattan Review www.manhattanreview.com


56 Number Properties Guide Questions

(D) 59
(E) 60
Solve yourself:

7. Which of the following numbers does not have a factor (other than 1) which is a perfect
square?
(A) 48
(B) 117
(C) 245
(D) 297
(E) 345
Solve yourself:

8. How many numbers between 10 and 10000 have the digit 4 in the units place and the
digit 2 in the tens place?
(A) 90
(B) 99
(C) 100
(D) 190
(E) 1000
Solve yourself:

9. A students scrapbook had a large collection of stamps which he wanted to number


starting from 1. If he used up a total of 855 digits in the process, how many stamps did
he have in his scrapbook?

www.manhattanreview.com 19992016 Manhattan Review


Number Properties Guide Questions 57

(A) 285
(B) 321
(C) 420
(D) 823
(E) 855

Solve yourself:

10. How many five-digit numbers divisible by 3 can be formed using the digits 0, 1, 2, 3, 4
and 5 without repeating any digit?

(A) 96
(B) 120
(C) 192
(D) 216
(E) 240

Solve yourself:

11. a12a is a four-digit number with the digit a in both the thousands and units places.
What is the value of a, if the number is divisible by 6?

(A) 2
(B) 3
(C) 4
(D) 6
(E) 8

Solve yourself:

19992016 Manhattan Review www.manhattanreview.com


58 Number Properties Guide Questions

12. What is the units digit of 1322 1222 + 1727 2525 ?

(A) 1
(B) 3
(C) 5
(D) 7
(E) 9

Solve yourself:


13. What is the value of 7 percent of 7 7?

(A) 0.049
(B) 0.49
(C) 0.14
(D) 0.50
(E) 4.90

Solve yourself:

14. What is the value of (5 106 ) + (9 104 ) + (2 102 )?

(A) 500000
(B) 509200
(C) 590200
(D) 5090200
(E) 5900200

Solve yourself:

www.manhattanreview.com 19992016 Manhattan Review


Number Properties Guide Questions 59

2 2
15. If x 2 yz = 5, y 2 xz = 1 and z2 xy = 7, find the value of x y + y z +
(z x)2 .

(A) 3
(B) 6
(C) 8
(D) 12
(E) 24

Solve yourself:

19992016 Manhattan Review www.manhattanreview.com


60 Number Properties Guide Questions

3.1.2 Even/Odd/Consecutive/Prime numbers

16. What is the largest possible positive integer value of a that satisfies a2 + 5b = 315; where
b is a prime number?

(A) 4
(B) 5
(C) 10
(D) 15
(E) 17

Solve yourself:

17. If m and n are positive integers, and n is odd, which of the following must be even?

(A) mn + 2n
(B) mn + 2m
(C) (n + 3)(m 3)
(D) (n + 2)(m 3)
(E) (n + 2)(m 2)

Solve yourself:

18. If n is even, which of the following cannot be odd?

I. 2n + 7
II. 3n 2
III. 5n2 + 1

(A) I only
(B) II only
(C) III only
(D) I and II only

www.manhattanreview.com 19992016 Manhattan Review


Number Properties Guide Questions 61

(E) I, II and III

Solve yourself:

19. What is the value of p, if the sum of consecutive odd numbers from 1 to p equals 225 (p
being a term of the series)?
(A) 15
(B) 21
(C) 29
(D) 30
(E) 31
Solve yourself:

20. How many even numbers between 1 and 300 are divisible by 13 but not by 7?
(A) 9
(B) 10
(C) 11
(D) 21
(E) 22
Solve yourself:

21. How many numbers of the form a b, where a and b are prime numbers, have sum of
factors, excluding the factor as the number itself, equal to 31?
(A) One

19992016 Manhattan Review www.manhattanreview.com


62 Number Properties Guide Questions

(B) Two
(C) Three
(D) Four
(E) Six
Solve yourself:

22. How many numbers, which are squares of prime numbers have sum of factors equal to
57?
(A) One
(B) Two
(C) Three
(D) Seven
(E) Nine
Solve yourself:

23. A perfect number is one whose sum of factors other than the number itself is equal to
the number. Which of the following numbers is a perfect number?
(A) 12
(B) 18
(C) 20
(D) 21
(E) 28
Solve yourself:

24. How many even numbers, from 10 to 100 (both inclusive), are not multiples of 7?

www.manhattanreview.com 19992016 Manhattan Review


Number Properties Guide Questions 63

(A) 32
(B) 38
(C) 39
(D) 45
(E) 46

Solve yourself:

19992016 Manhattan Review www.manhattanreview.com


64 Number Properties Guide Questions

3.1.3 Divisibility

25. If p 2 , a perfect square number, is divisible by 5, and q, an integer, is not divisible by 5,


which of the following must be divisible by 10?

(A) pq + 5
(B) p+q
(C) 2(p + 5q)

(D) 2 5p + q
(E) p 2 + 5q

Solve yourself:

26. How many integers between 150 and 250, both inclusive, are divisible neither by 6 nor
by 5?

(A) 34
(B) 60
(C) 67
(D) 68
(E) 71

Solve yourself:

27. A positive integer X, divided by 7 has a remainder 2. Which of the following must be
true?

I. X is odd
II. X cannot be a prime number
III. (X + 5) divided by 7 has remainder 0.

(A) I only
(B) I and II only
(C) III only

www.manhattanreview.com 19992016 Manhattan Review


Number Properties Guide Questions 65

(D) I and III only


(E) I, II and III

Solve yourself:

28. If N = 1! + 2! + 3! + 4! + +
 8!, what is the remainder when N is divided by 10?
p! = 1 2 3 p 1 p.

(A) 0
(B) 1
(C) 2
(D) 3
(E) 4

Solve yourself:

29. How many numbers between 1 and 250, excluding, are divisible by either 5 or 7 but not
both?

(A) 69
(B) 70
(C) 71
(D) 77
(E) 78

Solve yourself:

30. What is the highest index of 2 that can divide (10! + 11! + 12!) without leaving any
remainder?

19992016 Manhattan Review www.manhattanreview.com


66 Number Properties Guide Questions

(A) 8
(B) 9
(C) 12
(D) 16
(E) 25

Solve yourself:

 
31. For how many positive integer values of p other than 1, is p + 4 p + 6 divisible by
p?

(A) Four
(B) Seven
(C) Eight
(D) Twelve
(E) Twenty four

Solve yourself:

www.manhattanreview.com 19992016 Manhattan Review


Number Properties Guide Questions 67

3.1.4 Exponents

32. If 220 = 215 x+y, where x and y are non-negative integers, what is the minimum possible
value of |x y|?

(A) 0
(B) 1
(C) 25
(D) 210
(E) 215

Solve yourself:

33. What is the digit in the units place of 7123 + 342 ?

(A) 0
(B) 2
(C) 4
(D) 6
(E) 8

Solve yourself:

34. A perfect cube is a number whose cube root is an integer. For example, 8, 27, and 64 are
perfect cubes. If m and n are perfect cubes, which of the following is NOT necessarily a
perfect cube?

(A) m 6 n3
(B) mn
(C) mn + 125
(D) m
(E) (m n)9

19992016 Manhattan Review www.manhattanreview.com


68 Number Properties Guide Questions

Solve yourself:

35. If a 6= 0, which of the following must be greater than a?

I. 10a
II. 10a2
III. 10 a

(A) I only
(B) II only
(C) III only
(D) II and III only
(E) None

Solve yourself:

36. What is the rightmost non-zero digit in the product 626 1443 1569 ?

(A) 2
(B) 4
(C) 5
(D) 6
(E) 8

Solve yourself:

www.manhattanreview.com 19992016 Manhattan Review


Number Properties Guide Questions 69

37. How many trailing zeros does 30! have?; n! = 1 2 3 . . . (n 1) n.

(A) 3
(B) 6
(C) 7
(D) 15
(E) 26

Solve yourself:

38. Which of the following expressions has the highest value?

(A) 1550
(B) 3340
(C) 2200
(D) 3100
(E) 1150

Solve yourself:

19992016 Manhattan Review www.manhattanreview.com


70 Number Properties Guide Questions

3.1.5 Factors/Multiplicands/LCM/HCF

39. A number of coins are to be distributed among a number of children. If the coins were
distributed equally among three, four or five children, there were one, two or three coins
left over, respectively. What is the least number of coins?

(A) 55
(B) 56
(C) 57
(D) 58
(E) 62

Solve yourself:

40. Two numbers are such that their highest common factor is 18 and their sum is 216. How
many such pairs of numbers exist?

(A) Two
(B) Four
(C) Six
(D) Ten
(E) Eleven

Solve yourself:

41. Two numbers are such that their highest common factor is 12 and the least common
multiple is 360. How many such pairs of the numbers exist?

(A) One
(B) Two
(C) Four
(D) Seven
(E) Nine

www.manhattanreview.com 19992016 Manhattan Review


Number Properties Guide Questions 71

Solve yourself:

42. If A = 12!+313!+24, which of the following is a factor of A? n! = 12. . . (n 1)n.

(A) 5
(B) 10
(C) 15
(D) 20
(E) 24

Solve yourself:

43. A store has a stock of 18 pens, 45 pencils and 63 erasers. The owner wants to put them
in boxes so that each box has an equal number of items of the same type. What is the
minimum number of boxes required?

(A) 5
(B) 6
(C) 10
(D) 11
(E) 14

Solve yourself:

44. Which of the following must be true, if p and q are positive integers and p is a factor of
q?

(A) q = p k, where k > 1


(B) Number of factors of p is less than that of q

19992016 Manhattan Review www.manhattanreview.com


72 Number Properties Guide Questions

(C) The highest common factor of p and q is more than 1


(D) The least common multiple of p and q is equal to q
(E) p 2 is greater than q

Solve yourself:

www.manhattanreview.com 19992016 Manhattan Review


Number Properties Guide Questions 73

3.1.6 Inequality

45. If 0 < mn < 1, which of the following can be true?

(A) m < 1 and n < 1


(B) m < 1 and n > 0
(C) m > 1 and n < 1
(D) m > 1 and n < 1
(E) m > 1 and n > 1

Solve yourself:

46. If p and q are non-negative integers, p + q < 11 and p q > 8, which of the following
must be true for all the qualified values of p?

(A) q<3
(B) q>2
(C) 2 < q < 10
(D) q = 0 or 1
(E) q<1

Solve yourself:


x
47. If < 1, which of the following must be greater than 1?

y

y
(A)
x
!2
x
(B)
y
x
(C)
y

y
(D)
x

19992016 Manhattan Review www.manhattanreview.com


74 Number Properties Guide Questions

s
y
(E)
x

Solve yourself:

48. If x 2 and 5x + 3y 13, which of the following could be the value of y?


(A) 2
(B) 1
(C) 0
1
(D)
5
(E) 2
Solve yourself:

49. If m > 0, which of the following must be true?


I. m3 > 1
II. m m2 < 0
III. 9m 1 > 0
IV. m3 > 0

(A) I only
(B) II only
(C) III and IV only
(D) I and II only
(E) IV only
Solve yourself:

www.manhattanreview.com 19992016 Manhattan Review


Number Properties Guide Questions 75

50. How many integer values of a are possible, if a is an integer such that a2 4a < 221?

(A) 27
(B) 28
(C) 29
(D) 30
(E) 31

Solve yourself:

19992016 Manhattan Review www.manhattanreview.com


76 Number Properties Guide Questions

3.2 Data Sufficiency

Data sufficiency questions have five standard options. They are listed below and will not
be repeated for each question.

A. Statement (1) ALONE is sufficient, but statement (2) ALONE is not sufficient to an-
swer the question asked.
B. Statement (2) ALONE is sufficient, but statement (1) ALONE is not sufficient to an-
swer the question asked.
C. BOTH statements (1) and (2) TOGETHER are sufficient to answer the question asked,
but NEITHER statement ALONE is sufficient to answer the question asked.
D. EACH statement ALONE is sufficient to answer the question asked.
E. Statements (1) and (2) TOGETHER are NOT sufficient to answer the question asked,
and additional data specific to the problem are needed.

www.manhattanreview.com 19992016 Manhattan Review


Number Properties Guide Questions 77

3.2.1 Numbers & Digits

51. Are there three distinct integers?

(1) The product of the integers is 0.


(2) The sum of the integers is 0.

Solve yourself:

52. We have a two digit number ab where a and b are the tens and units digits respectively,
both being non-zero. Is the number ab divisible by 15?

(1) The sum of the digits is 5.


(2) The product of the digits is 4.

Solve yourself:

53. Is x = 1?

(1) (2x 3)2 = x 2


(2) (2x 3)2 = x

Solve yourself:

54. Is the value of x distinct?



(1) 3x 2 8 = x

(2) x2 + 5 = x + 1

19992016 Manhattan Review www.manhattanreview.com


78 Number Properties Guide Questions

Solve yourself:

www.manhattanreview.com 19992016 Manhattan Review


Number Properties Guide Questions 79

3.2.2 Even/Odd/Consecutive/Prime numbers

55. Is the product of 10 integers negative?

(1) There are at the most 4 negative integers.


(2) There are at the most 5 positive integers.

Solve yourself:

56. Is the product of 12 integers negative?

(1) There are at the most 6 negative integers.


(2) There are at least 6 positive integers.

Solve yourself:

57. If m and n are positive integers, is (n + 3)(m 3) even?

(1) n is odd
(2) m is even

Solve yourself:

58. Is 5n2 + 1 even?

(1) 2n2 3 is odd


(2) 3n2 2 is odd

19992016 Manhattan Review www.manhattanreview.com


80 Number Properties Guide Questions

Solve yourself:

59. If a, b and c are positive integers, is the product abc odd?

(1) a + 1 is even.
(2) (b c) (a c) is odd.

Solve yourself:

60. Is q p (ab) even, if a, b, p, q are positive integers with a > b?


q
(1) p= + 4.
7
(2) a b is even.

Solve yourself:

61. Is 15 the arithmetic mean of p, q, r and 15?

(1) p + q + r = 45
(2) pq =qr =5

Solve yourself:

www.manhattanreview.com 19992016 Manhattan Review


Number Properties Guide Questions 81

3.2.3 Divisibility

62. If abc = 1, is d = 0?

(1) cde = 0
(2) ebc = 0

Solve yourself:

63. Does a positive integer X, divided by 7 has a remainder 2?

(1) (X + 5) divided by 7 has remainder 0.


(2) (X 2) divided by 7 has remainder 0.

Solve yourself:

a2
64. If the expression is not necessarily in its reduced form, is it a terminating decimal?
b

(1) b = 110
(2) a is a positive integer divisible by 44.

Solve yourself:

65. If a and b are positive integers more than 1, is 2(a3 a) divisible by b?

(1) b is an even multiple of 3 and is less than 15.


(2) b is a multiple of 4.

19992016 Manhattan Review www.manhattanreview.com


82 Number Properties Guide Questions

Solve yourself:

66. If a and b are the digits in the hundreds and units positions respectively of the number
3a1b, is (a b) positive?

(1) n is divisible by 4
(2) n is divisible by 9

Solve yourself:

67. What is the remainder when a positive number p is divided by 4?

(1) p is the square of an odd number.


(2) p is a multiple of 5

Solve yourself:

68. Is the three-digit number ABB divisible by 3, if A and B are the non-zero digits of the
number?

(1) The two-digit number BB is divisible by 3.


(2) The three-digit number A0B is divisible by 101, where A and B are the hundreds
and units digits respectively.

Solve yourself:

www.manhattanreview.com 19992016 Manhattan Review


Number Properties Guide Questions 83

p
69. If p is a positive integer, is odd?
3
(1) p is divisible by 3.
(2) p 2 is divisible by 9.

Solve yourself:

19992016 Manhattan Review www.manhattanreview.com


84 Number Properties Guide Questions

3.2.4 Exponents

70. A perfect cube is a number whose cube root is an integer. For example, 8, 27, and 64 are
perfect cubes. Is (m n)9 a perfect cube?

(1) (m n)12 is perfect cube


(2) (m n)3 is perfect cube

Solve yourself:

71. What is the minimum value of (x + y)?

(1) 7x is perfect square


(2) 147xy is perfect cube

Solve yourself:

72. Is p an integer?

(1) 3p is an integer.
(2) p 3 is an integer.

Solve yourself:

a3
73. If a is an integer, is a positive integer?
27
a2
(1) is a positive integer.
9
a
(2) is a positive integer.
9

www.manhattanreview.com 19992016 Manhattan Review


Number Properties Guide Questions 85

Solve yourself:

(2 3x 2 ) (10 x) + (x 10)
74. What is the value of ?
(x 10)
(1) x 2 = 144
(2) x < 10

Solve yourself:

75. Is x 3 = 1?

(1) x 2 + 5x + 4 = 0
(2) x 3 + 3x 2 = (1 + 3x)

Solve yourself:

76. Is p = q?

(1) p 3 q3 = 0
(2) p 2 + q2 = 0

Solve yourself:

77. If x is a positive integer, what is the value of x?

19992016 Manhattan Review www.manhattanreview.com


86 Number Properties Guide Questions

(1) x 3 is a perfect square (a number that has an integer as its square root).
(2) x 2 12x + 32 = 0

Solve yourself:

78. If p and q are positive integers, is p 2 q2 + 10q = 25?

(1) p+q =5
(2) p = 15

Solve yourself:

www.manhattanreview.com 19992016 Manhattan Review


Number Properties Guide Questions 87

3.2.5 Factors/Multiplicands/LCM/HCF

79. If s is a positive integer, does s have more than two distinct factors?
35
(1) is an integer.
s
21
(2) is an integer.
s
Solve yourself:

80. If a is an integer, is 15 a factor of a?

(1) a is divisible by both 5 and 10.


(2) a is divisible by 3.

Solve yourself:

81. A is a set of integers which contains the element 7. If n be any element of the set A, are
all the elements of A multiples of 7?

(1) n + 7 is an element of the set A.


(2) n 7 is an element of the set A.

Solve yourself:

19992016 Manhattan Review www.manhattanreview.com


88 Number Properties Guide Questions

3.2.6 Inequality

82. Is x < x 2 < x 3 ?

(1) x < x3
(2) x3 > x2

Solve yourself:

83. If a is a non-zero number, is a |a| < |a|a + 1 ?

(1) a<1
(2) a>0

Solve yourself:

m
84. If m and n are integers, is is an integer?
n

(1) n = m2
(2) |m| > 1

Solve yourself:

85. If 5x + 3y 13, does y 1?

(1) x 2
(2) x > 2

www.manhattanreview.com 19992016 Manhattan Review


Number Properties Guide Questions 89

Solve yourself:

86. Is m > 0?
(1) m m2 < 0
(2) m3 > 0
Solve yourself:


4
87. Is a. b > 1?

(1) b = a
(2) b2 > b
Solve yourself:

88. If x is an integer, is x < x 2 ?


(1) x<1
(2) x<0
Solve yourself:

89. Is 0 < jk < 1?


(1) j < 1 and k < 1

19992016 Manhattan Review www.manhattanreview.com


90 Number Properties Guide Questions

(2) j > 1 and k < 1

Solve yourself:

90. If x and y are non-negative integers, is y = 1?

(1) x + y < 11
(2) x y > 8

Solve yourself:

91. Is |xy| > 1; if x and y are integer?

(1) |x | + |y| > 1


(2) xy > 1

Solve yourself:

92. Is a7 b4 c 6 < 0?

(1) a|b|2 < 0


(2) ac 2 < 0

Solve yourself:

www.manhattanreview.com 19992016 Manhattan Review


Number Properties Guide Questions 91

93. If x and y are integers, what is the minimum possible value of (x y)?
(1) x + y < 23
(2) x > 11
Solve yourself:

94. Is 0 < mn < 1?


(1) |m| > 1 and |n| < 1
(2) |m| < 2 and |n| > 0
Solve yourself:

95. Is 0 < ab < 1?


(1) 0 < |a| < 1 and 0 < |b| < 1
(2) 0 < |ab| < 1
Solve yourself:

96. Is p > q?
(1) p > q2
(2) p>1
Solve yourself:

19992016 Manhattan Review www.manhattanreview.com


92 Number Properties Guide Questions

97. Is x > 4?
(1) (x 4) (x + 3) > 0
(2) x > 4
Solve yourself:

98. What is the value of a if a2 = 120 + b2 ; where a and b are positive integers?
(1) (a + b) > (a b).
(2) Sum of digits of the number a is 8.
Solve yourself:

99. Is x negative?
(1) x > 2q
(2) x<q
Solve yourself:

100. If p q > 0, is p + q > 0?


(1) q > p4
(2) q = 4p
Solve yourself:

www.manhattanreview.com 19992016 Manhattan Review


Number Properties Guide Questions 93

3.3 Assorted Questions

3.3.1 Problem Solving

101. If a2 + b2 + c 2 = ab + bc + ca and a = 5, what is the value of (a + b + c)?

(A) 5
(B) 8
(C) 12
(D) 15
(E) 20

Solve yourself:

30 + 2n
 
102. For how many integer values of n, is a positive integer?
n
(A) Five
(B) Six
(C) Seven
(D) Eight
(E) Nine

Solve yourself:

s 2
0.000375
q
3
103. Which of the following is the value of (0.0004)3 +
24
(A) 0.000063
(B) 0.000633
(C) 0.000705
(D) 0.006330
(E) 0.007050

19992016 Manhattan Review www.manhattanreview.com


94 Number Properties Guide Questions

Solve yourself:

104. If n leaves a remainder 5 when divided by d, and a remainder 11 when divided by 2d,
what is the value of d?

(A) 6
(B) 7
(C) 8
(D) 9
(E) 12

Solve yourself:

105. What is the smallest three digit number that when divided by any of the integers from 2
to 6, leaves a remainder 1 in each case?

(A) 111
(B) 121
(C) 122
(D) 241
(E) 721

Solve yourself:

If x 2 2y 2 = 4, what is the minimum value of |x| y ?



106.

(A) 2
(B) 0
(C) 1

www.manhattanreview.com 19992016 Manhattan Review


Number Properties Guide Questions 95

(D) 2

(E) 2 2

Solve yourself:

!
x y
107. If 2 x 8 and 2 y 5, what is the maximum value of ?
y
(A) 1
(B) 2
(C) 3
(D) 3.5
(E) 4

Solve yourself:


3

4
108. If x = 2, y = 4 and z = 10, which of the following is the correct order of x, y and
z?

(A) x<y <z


(B) x<z<y
(C) z>x>y
(D) z>y >x
(E) y >z>x

Solve yourself:

109. What is the smallest value of (m + n) where m and n are positive integers, such that
2n > 56 > 2m ?

19992016 Manhattan Review www.manhattanreview.com


96 Number Properties Guide Questions

(A) 12
(B) 14
(C) 15
(D) 26
(E) 27
Solve yourself:

n
110. If n is divided by 18, the non-zero remainder is r . If is a terminating decimal, how
18
many numbers of values of r are possible?
(A) 1
(B) 2
(C) 8
(D) 9
(E) 17
Solve yourself:

111. If n is the product of integers from 21 to 30, what is the highest exponent of 3 that can
divide n perfectly?
(A) 1
(B) 2
(C) 6
(D) 8
(E) 14
Solve yourself:

www.manhattanreview.com 19992016 Manhattan Review


Number Properties Guide Questions 97

112. If 11! = 2a 3b k; where a and b are the highest possible exponents of 2 and 3,
respectively which are factors of 8!, what is the value of k?
(A) 15 77
(B) 45 154
(C) 45 770
(D) 90 1540
(E) 90 7700
Solve yourself:

1 1 1
 
113. What is the value of + + ?
1+ 2 2+ 3 3+2
(A) 1

(B) 2

(C) 3
(D) 2
(E) 3
Solve yourself:


114.  that |p| |q| = |r | and q 3, what is the minimum
If p, q and r are integers such
non-negative value of p + r ?
(A) 2
(B) 3
(C) 5
(D) 6
(E) 8
Solve yourself:

19992016 Manhattan Review www.manhattanreview.com


98 Number Properties Guide Questions

7
115. If p and q are positive integers, what is the minimum value of (p + q) given that <
10
p 3
< ?
q 4

(A) 3
(B) 5
(C) 7
(D) 12
(E) 13

Solve yourself:

45
 
116. If k is a positive integer, how many values of k exist so that is an integer?
4k 1
(A) 0
(B) 1
(C) 2
(D) 3
(E) 5

Solve yourself:

!
q2 + 24
117. If p = , what is the minimum possible integer value of |p| given that q is a
q
negative integer?

(A) 6
(B) 8
(C) 10
(D) 11
(E) 25

www.manhattanreview.com 19992016 Manhattan Review


Number Properties Guide Questions 99

Solve yourself:


37 1
118. What is the value of (a + b + c) if a + 
=  , where a, b and c are positive
16 1
b+
c
integers.

(A) 3
(B) 5
(C) 6
(D) 7
(E) 9

Solve yourself:

119. If the Least Common Multiple (LCM) of two integers a and b is 120 and their Greatest
Common Divisor (GCD) is 6, how many values exist for (a + b)?

(A) 1
(B) 2
(C) 3
(D) 4
(E) 5

Solve yourself:

120. If [x] denotes the greatest integer


  less than or equal to x, for how many positive integer
x x

values of x less than 10, is = +1 ?
3 10

19992016 Manhattan Review www.manhattanreview.com


100 Number Properties Guide Questions

(A) 1
(B) 2
(C) 3
(D) 4
(E) 5
Solve yourself:

12
 
121. What is the minimum value of , if 3 x 8?
|6 2x|
(A) 1
(B) 0
1
(C)
2
(D) 1
(E) 2
Solve yourself:

4
122. If x and y are numbers satisfying (x 2)2 = 25 and x y 3 = 256, what is the
maximum value of |xy|?
(A) 24
(B) 36
(C) 42
(D) 56
(E) 70
Solve yourself:

www.manhattanreview.com 19992016 Manhattan Review


Number Properties Guide Questions 101

Which of the following is NOT a factor of 212 + 213 + 214 + 215 + 216 + 217 ?

123.

(A) 12
(B) 56
(C) 72
(D) 132
(E) 144

Solve yourself:

124. How many three digit numbers aba exist, where a, b and a are non-zero digits in the
hundreds, tens and units places, respectively, so that the number aba is divisible by
4?

(A) 16
(B) 18
(C) 22
(D) 24
(E) 25

Solve yourself:

125. There are three distinct numbers such that the Greatest Common Divisor (GCD) of each
pair of numbers is 3. If the product of the numbers is 540, how many such sets of three
numbers are there?

(A) 1
(B) 2
(C) 3
(D) 6
(E) 9

19992016 Manhattan Review www.manhattanreview.com


102 Number Properties Guide Questions

Solve yourself:

126. The Least Common Multiple (LCM) of the first 25 positive integers is L. What is the LCM
of the first 27 positive integers when expressed in terms of L?

(A) L
(B) 3L
(C) 6L
(D) 39L
(E) 78L

Solve yourself:

127. What is the units digit of 111! + 212! + 313! ?

(A) 0
(B) 2
(C) 4
(D) 6
(E) 8

Solve yourself:

www.manhattanreview.com 19992016 Manhattan Review


Number Properties Guide Questions 103

 
2x+28
128. What is the sum of all positive integer values of x such that x+6 is an integer?
(A) 2
(B) 6
(C) 8
(D) 12
(E) 14
Solve yourself:

129. What is the product of all possible values of x such that (2x 1)(x+4) = 1?
(A) 4
(B) 3
(C) 0
(D) 1
(E) 4
Solve yourself:

130. The sum of digits of a three digit number is 6. What is the maximum value of the product
of the three digits?
(A) 4
(B) 5
(C) 6
(D) 8
(E) 9
Solve yourself:

19992016 Manhattan Review www.manhattanreview.com


104 Number Properties Guide Questions

131. How many values of x are possible, if 2x + 2xy = y?, where x and y are integers?
(A) 0
(B) 1
(C) 2
(D) 3
(E) 4
Solve yourself:

 
1
132. If 1+ a+b = 2, what is the maximum possible integer value of a?, where a is a positive
integer and b is any positive number.
(A) 1
(B) 2
(C) 3
(D) 4
(E) 5
Solve yourself:

133. If a = bc , b = c a and c = a4 , what is the value of ac?, where each of a, b, ?? are greater
than 1.
1
(A) 4
1
(B) 3
1
(C) 2
(D) 1
(E) 4
Solve yourself:

www.manhattanreview.com 19992016 Manhattan Review


Number Properties Guide Questions 105

134. How many values of x satisfy x + [x] = 5?, where [x] denotes the greatest integer less
than or equal to x.

(A) 0
(B) 1
(C) 2
(D) 3
(E) 5

Solve yourself:

135. For how many values of n from 1 to 20, does (3n2 + 2) and (9n2 + 4) have no common
factors other than 1?

(A) 1
(B) 4
(C) 10
(D) 14
(E) 20

Solve yourself:

136. The three digit number xyz, which when


 divided by its sum of digits has the minimum
value. What is the value of x + y + z ?, where x, y and z are the digits in the hundreds,
tens and units positions?

(A) 2
(B) 3
(C) 11
(D) 18
(E) 19

19992016 Manhattan Review www.manhattanreview.com


106 Number Properties Guide Questions

Solve yourself:

137. The two digit number x9, which when divided by its sum of digits leaves a maximum
possible remainder. What is the value of x?, where x and 9 are the digits in the tens and
units positions respectively.

(A) 1
(B) 2
(C) 4
(D) 7
(E) 8

Solve yourself:

www.manhattanreview.com 19992016 Manhattan Review


Number Properties Guide Questions 107

3.3.2 Data Sufficiency

138. If 8x + y = 26, where x and y are positive integers, what is the value of y?

(1) x + y > 10
(2) x + y < 15

Solve yourself:

139. If x is a non-negative integer and y is an integers, does 5x + 3y = 36?

(1) y 2 = 36
(2) 5|x| = 3y

Solve yourself:

a
140. Is an integer?; b 6= 0.
b

(1) |a| = |b|


(2) a and b both are unequal non-integers numbers of the same magnitude.

Solve yourself:

141. Is p > q?

(1) xp + xq 0.
(2) xp + xq 0.

19992016 Manhattan Review www.manhattanreview.com


108 Number Properties Guide Questions

Solve yourself:

142. What is the value of x?


36
(1) 4 = 14
x
(2) (x 4) (x 1) = x

Solve yourself:

p
143. If p = q4 , what is the value of ?
q
(1) p = 16
(2) pq = 32

Solve yourself:

144. Is p even?
 
(1) p7 p + 2 is even.
(2) 2p is even.

Solve yourself:

www.manhattanreview.com 19992016 Manhattan Review


Number Properties Guide Questions 109

145. If a 6= 0, is a = 1?
(1) a3 = a
(2) a2 = a
Solve yourself:

146. How many prime numbers are there between 82 and p?


(1) 83 < p < 96
(2) 90 < p < 99
Solve yourself:

147. Is pq > 0?
(1) p 2 q3 < 0
(2) pq2 < 0
Solve yourself:

148. If S is a finite set of positive numbers, does the set contain a number equal to the average
of all the numbers in the set?
(1) All numbers in the set are distinct multiples of 6.
(2) The set contains an odd number of terms which are consecutive multiples of 6.
Solve yourself:

19992016 Manhattan Review www.manhattanreview.com


110 Number Properties Guide Questions

p
149. Is = 1?
q
!2 !2
p q
(1) + = 2.
q p
p q
(2) + = 2.
q p

Solve yourself:

150. What is the value of k?

(1) k(k 1)2 = 4k.


(2) k is an integer divisible by 3.

Solve yourself:

151. If the symbol represents one among addition, subtraction, multiplication or division,
and r is any non-zero number, what does represent?

(1) r 0 = r.
(2) (r 0) r = 0.

Solve yourself:

152. p, q and r denote the integer lengths of three line segments. What should be the mini-
mum value of p so that the line segments form a triangle?

(1) q = r + 6.
(2) q = 8, r = 2.

www.manhattanreview.com 19992016 Manhattan Review


Number Properties Guide Questions 111

Solve yourself:

153. What is the two-digit number A?

(1) The tens digit is twice the units digit of A.


(2) The sum of the digits of A is 6.

Solve yourself:

154. If m and n are positive integers, is m + n2 odd?

(1) mn + n2 is odd.
(2) m2 + n2 is odd.

Solve yourself:

155. If r , s and t are integers, is (r s + t) even?

(1) r is 3 times a prime number, greater than 5 and s is not divisible by 2.


(2) t leaves a remainder 3 when divided by 4.

Solve yourself:

156. If m is a multiple of an odd prime number p, is m also a multiple of p 2 ?

(1) p<8

19992016 Manhattan Review www.manhattanreview.com


112 Number Properties Guide Questions

(2) m = 60

Solve yourself:

x4y 3 x2y 5
157. What is the value of the expression ?
x2y 3 y 5
(1) x = 2.
(2) xy = 1; where both x and y are integers.

Solve yourself:

m
158. Is = 1?
n
m n
(1) =
n m
(2) (m n)2 = m2 n2

Solve yourself:

159. Is a b c d = 16?

(1) All of a, b, c and d are positive with a b c d.


(2) a + b + c + d = 8.

Solve yourself:

www.manhattanreview.com 19992016 Manhattan Review


Number Properties Guide Questions 113

160. What is the value of a b c d if a, b, c and d are positive integers?

(1) a+b+c+d=9
(2) ab>c>d

Solve yourself:

105
161. Is an integer, given that a, b and c are integers?
abc
(1) abc is prime.
(2) 1 a 5, 1 b 7 and 3 c 10.

Solve yourself:

162. If Pn = a1 a2 . . . an , where a1 > 0 and a2 < 0, is P8 > 0?

(1) All terms having an odd suffix have the same value.
(2) All terms having an even suffix have the same value.

Solve yourself:

19992016 Manhattan Review www.manhattanreview.com


114 Number Properties Guide Questions

163. If m and n are positive numbers, is m n?


m3
(1) 1
n
3
m
(2) 1
n
Solve yourself:

164. Is (a + b + c + d + e) even, where a, b, c, d and e are positive integers?


(1) (a b c d e) is odd.
(2) (a b c d e) is even.
Solve yourself:

165. Is (a b)(b c) even, where a, b and c are positive integers with a > b > c?
(1) (a b c) is odd.
(2) (a b c) is even.
Solve yourself:

166. What is the value of q, if 3p + q2 = 42?; where p and q are positive integers.
(1) p is a prime number
(2) p can be expressed as the sum of two positive integers in only one way
Solve yourself:

www.manhattanreview.com 19992016 Manhattan Review


Number Properties Guide Questions 115

167. If a, b and c are prime numbers, is c = 2?


(1) a + b + c = 56
(2) a>b>c
Solve yourself:

168. Is (n 1)(n 3) even?


(1) (n2 9) is odd.
(2) (n + 1)(n + 5) is odd.
Solve yourself:

169. What is the value of the positive integer p?


p
(1) 22 = 256
2
(2) 2p = 512
Solve yourself:

170. If ki = p i + i for all integer values of i, what is the value of p?


(1) k10 = 2k9
(2) k7 + k8 = 0
Solve yourself:

19992016 Manhattan Review www.manhattanreview.com


116 Number Properties Guide Questions

171. If p and q are positive integers, is p a factor of q?

(1) 2p1 = p 2 with p < 4


(2) 3q3 6 = 75

Solve yourself:

172. If n is a positive integer, does n have divisors other than 1 and n?

(1) n<6
(2) 5 < 4n < 13

Solve yourself:

173. Is p a positive even number?

(1) If p is divided by 5, the remainder is 2.


(2) If p is divided by 8, the remainder is 2.

Solve yourself:

www.manhattanreview.com 19992016 Manhattan Review


Number Properties Guide Questions 117

174. Is u2 > 64?

(1) u > 8

(2) u<8

Solve yourself:

175. Is 0 < p < 1?



(1) 0< 3 p<1
1
(2) p4 =
16

Solve yourself:

176. What is the greatest common divisor of the positive integers m and n?

m n
(1) The greatest common divisor of the positive integers and is 12.
4 4
m n
(2) and are even.
2 2

Solve yourself:

177. If x > 0, what fraction of y is x?

(1) x 2 2xy 3y 2 = 0
x 9y
(2) x% of =
y 100

19992016 Manhattan Review www.manhattanreview.com


118 Number Properties Guide Questions

Solve yourself:

375
178. If is an integer, how many number of values n can assume?
5n
(1) n > 5.
(2) n has more than two factors.

Solve yourself:

240 + 150n
 
179. Is an integer?; where n 6= 0.
n
(1) n is a factor of 75.
(2) n is a factor of 30.

Solve yourself:

180. If n is a prime number, what is the remainder, if n2 is divided by 6?

(1) n>2
(2) n>7

Solve yourself:

www.manhattanreview.com 19992016 Manhattan Review


Number Properties Guide Questions 119

181. What is the value of the non-zero number p?


(1) |p|3 + p 3 = 0
(2) |p|2 = 2p
Solve yourself:

182. Which one among |a|, |b|, |c| and |d| is the maximum if a < b < c < d?
(1) a = 2d
(2) d = b
Solve yourself:

183. If x, y and z are positive integers, what is the value of (|x y| + |y z|)?
(1) x + z = 2y
(2) xz = y 2
Solve yourself:

x
184. Is y an integer?; where x and y are positive integers.
(1) y(2x + 1) is an even integer.
(2) x = (2x 1)2
Solve yourself:

19992016 Manhattan Review www.manhattanreview.com


120 Number Properties Guide Questions

185. Is xy divisible by 60?


(1) x > 3 and is also a factor of 12.
(2) y is an even multiple of 15.
Solve yourself:

1 1 1
186. Can + be expressed as ?; where x, y and z are distinct positive integers
x y z
(1) x and y are both less than 7.
(2) The GCD (Greatest Common Divisor) of x and y is 1.
Solve yourself:

187. What is the value of abc?


(1) a, b and c are distinct prime numbers less than 10.
(2) ab = 15, bc = 21 and ac = 35.
Solve yourself:

188. If [x] denotes the least integer greater than or equal to x, is [x] < 0?
(1) 8x 2 + 6x + 1 = 0
(2) 2x 2 x 1 = 0
Solve yourself:

www.manhattanreview.com 19992016 Manhattan Review


Number Properties Guide Questions 121

189. Each of the three positive numbers is squared and is divided by 3. What is the sum of
the remainders obtained after their divisions?
(1) The numbers are consecutive integers.
(2) The numbers lie between 10 and 30.
Solve yourself:

190. What is the sum of the digits of a three digit number?


(1) The product of the digits is 36.
(2) All three digits are not distinct and none of the digits is 4.
Solve yourself:

191. What is the remainder when the number 2N is divided by 10?


(1) The number N when divided by 100 leaves a remainder 24.
(2) The number N when divided by 5 leaves a remainder 4.
Solve yourself:

192. If k and r are integers, what is the value of k?



(1) 16 10r = k 103
(2) 100 < 2.36 10k < 1000
Solve yourself:

19992016 Manhattan Review www.manhattanreview.com


122 Number Properties Guide Questions

193. For positive integers x, y,and n, is (x n+1 xy n ) divisible by (x 2 y 2 )? Given that


x > y.

(1) n is even
(2) n is a multiple of 3

Solve yourself:

194. If a is a positive integer, is (a3 + a) divisible by 6?

(1) For any positive integer n, [n(n + 1)(n + 2)] is divisible by a!


(2) a is an odd integer

Solve yourself:


195. If p is a positive integer, how many integer values of n are possible if 45 < n <
p
45 + p?

(1) p<6
(2) 4 < p < 10

Solve yourself:

c
196. If c < 0, is > 0?
a

b3
(1) >0
|a|
|b|
(2) >0
a+c

www.manhattanreview.com 19992016 Manhattan Review


Number Properties Guide Questions 123

Solve yourself:

1
 
197. If x is a positive number, what is the value of x + ?
x

1
 
(1) x2 + =2
x2
1
 
(2) x+ =2
x

Solve yourself:

2
1 1

+
a 2b
198. If P = , is P > 1?
4
ab

b
(1) a=
2
(2) a=4

Solve yourself:

199. The symbol represents a mathematical operation between two numbers a and b such
that a b = (ab + ba ). What is the value of x if x 2 = 17?

(1) x is a positive integer.

(2) x is a prime number.

19992016 Manhattan Review www.manhattanreview.com


124 Number Properties Guide Questions

Solve yourself:

200. If a and b are distinct integers, is (2a)b > 30?

(1) a and b are single digit primes.


(2) a+b =5

Solve yourself:

www.manhattanreview.com 19992016 Manhattan Review


Chapter 4

Answer-key

125
126 Number Properties Guide Answer Key

4.1 Problem Solving

(1) C (18) B (35) E

(2) E (19) C (36) D

(3) C (20) B (37) C

(4) D (21) C
(38) B

(5) E (22) A
(39) D

(6) E (23) E
(40) A
(7) E (24) C
(41) C
(8) C (25) C
(42) E
(9) B (26) C
(43) E
(10) D (27) C
(44) D
(11) D (28) D
(45) C
(12) A (29) B

(46) E
(13) B (30) C

(47) E
(14) D (31) B

(15) B (32) C (48) E

(16) C (33) B (49) E

(17) C (34) C (50) C

www.manhattanreview.com 19992016 Manhattan Review


Number Properties Guide Answer Key 127

4.2 Data Sufficiency

(51) E (68) B (85) A

(52) D (69) E (86) B

(53) C (70) B (87) C


(54) D (71) B (88) B
(55) C (72) C
(89) E
(56) E (73) B
(90) C
(57) A (74) A
(91) B
(58) B (75) B
(92) C
(59) B (76) D
(93) C
(60) E (77) C
(94) E
(61) A (78) A
(95) E
(62) E (79) C
(96) C
(63) B (80) C

(64) C (81) D (97) E

(65) A (82) B (98) B

(66) C (83) A (99) C

(67) A (84) C (100) D

19992016 Manhattan Review www.manhattanreview.com


128 Number Properties Guide Answer Key

4.3 Assorted Questions

4.3.1 Problem Solving

(101) D (114) B (127) E

(102) E (115) D (128) D


(103) B (116) C
(129) C
(104) A (117) C
(130) D
(105) B (118) C

(106) D (119) B (131) C

(107) C (120) C (132) B

(108) D (121) D (133) A


(109) C (122) D
(134) A
(110) A (123) D
(135) E
(111) C (124) B

(112) C (125) A (136) E

(113) A (126) B (137) D

www.manhattanreview.com 19992016 Manhattan Review


Number Properties Guide Answer Key 129

4.3.2 Data Sufficiency

(138) C (159) E (180) B

(139) C (160) C (181) B

(140) D (161) C (182) D

(141) E (162) C (183) C

(142) D (163) C (184) C

(143) B (164) A (185) C

(144) E (165) A (186) B

(145) B (166) B (187) B

(146) C (167) C (188) D

(147) C (168) D (189) A

(148) B (169) D (190) C

(149) B (170) D (191) D

(150) E (171) A (192) B

(151) C (172) B (193) A

(152) D (173) B (194) A

(153) C (174) C (195) B

(154) D (175) A (196) B

(155) C (176) A (197) D

(156) B (177) C (198) A

(157) D (178) D (199) D

(158) C (179) B (200) A

19992016 Manhattan Review www.manhattanreview.com


130 Number Properties Guide Answer Key

www.manhattanreview.com 19992016 Manhattan Review


Chapter 5

Solution

131
132 Number Properties Guide Solutions

5.1 Problem Solving

5.1.1 Numbers & Digits


15 3y 3y
1. Let us reduce the equation 5x + 3y = 15 to x = =3 .
5 5
3y 3y
must be an integer so that x must be an integer. To get as an integer, y must be
5 5
multiplicand of 5. So set values of y would be {0, 5, 10, 15, ...}.

All the options except option C qualify the criterion.

The correct answer is option C.

m
2. Say = k; where k in an integer. It means that m = kn. This negates option E, so
n
option E is the correct answer.

Alternate Approach:

We apply process of elimination. Since this question is CAN BE TRUE type of question,
we must plug in at least one value for m and n in the options that satisfy the question.

m 2
(A) Both m and n are integers Say m = 2, and n = 1. It gives = = 1, an integer.
n 1
m 2
(B) m is an integer Again, say m = 2, and n = 1. It gives = = 1, an integer.
n 1
m 2
(C) Either m or n is negative Say m = 2, and n = 1. It gives = = 1, an
n 1
integer.
n n 1
(D) is an integer Say m = 1, and n = 1. It gives = = 1, an integer, while
m m 1
m 1
= = 1 is also an integer.
n 1
m
(E) m = kn; where k is non-integer If m = kn, we can rewrite it as = k. The
n
m m
question already states that is an integer, but the option says k or is not an
n n
integer. This option cannot be true.

The correct answer is option E.

www.manhattanreview.com 19992016 Manhattan Review


Number Properties Guide Solutions 133

34
3. With the given condition, we get (4 # 3) = = 6.
2
16
Again (4 # 3) # 1 = 6 # 1 = = 3.
2

The correct answer is option C.

4. Given that the product of integers is negative means that there must be an odd number
of negative integers in the list, because even number of integers will make product
positive. So possible numbers of negative integers are 1, 3, 5, 7, and 9. The maximum
number of possible negative integers would be nine.

The correct answer is option D.

The information at least two integers are negative is redundant here.

5. abc = 1 means that none of a, b, and c is 0. Similarly, if cde = 0, it means that either d
or e or both are 0, because as deduced that c cannot be 0.

Again, ebc = 0, means that e is 0, because as deduced that b and c cannot be 0.

The correct answer is option E.

You might have observed that condition cde = 0 is redundant here. We inferred e = 0,
by only two conditions: abc = 1, and ebc = 0.

6. Given: 0 < 5x + 11 < 300.


We split this inequality into two.

(1) 0 < 5x + 11
11
=> x > = 2.20
5
=> x will have set of integers values as {2, 1, 0, . . . }

(2) 300 > 5x + 11


289
=> x < or x < 57.80
5
=>It means that x will have set of integers values as {57, 56, 55, 54 . . . }

So the total number of integer values of x would be 3 + 57 = 60.

The correct answer is option E.

19992016 Manhattan Review www.manhattanreview.com


134 Number Properties Guide Solutions

7. We need to find a number which does not have a perfect square as a factor.
A perfect square is one which has an even exponent i.e. the number is of the form x 2k ;
where x and k are integers.

Working with the options:


Option A: 48 = 16 3 = 42 3.
Thus, 42 , a perfect square is a factor of 48.

Option B: 117 = 9 13 = 32 13.


Thus, 32 , a perfect square is a factor of 117.

Option C: 245 = 49 5 = 72 5.
Thus, 72 , a perfect square is a factor of 245.

Option A: 297 = 9 33 = 32 33.


Thus, 32 , a perfect square is a factor of 297.

Hence, the correct option must be option E.

Let us verify: 345 = 3 5 23. Thus, there are no perfect squares as a factor of 345.

The correct answer is option E.

8. There is only one two-digit number having the digit 4 in the units place and the digit
2 in the tens place i.e. 24.

Among three-digit numbers, the units and tens digits are 4 and 2, respectively.
The hundreds digit may be chosen from 1 to 9 in nine ways.
Thus, there are nine such numbers.

Among four-digit numbers, the units and tens digits are 4 and 2, respectively.
The thousands digit may be chosen from 1 to 9 in nine ways and the hundreds digit
may be chosen from 0 to 9 in 10 ways.
Total number of ways of choosing the two digits = 9 10 = 90.
Thus, there are 90 such numbers.

There are no five-digit numbers having the digit 4 in the units place and the digit 2
in the tens place since the last number is 10000 which is the least possible five-digit
number.
Thus, we have 1 + 9 + 90 = 100 such numbers.

www.manhattanreview.com 19992016 Manhattan Review


Number Properties Guide Solutions 135

Alternate Approach:

There is only one two-digit number having the digit 4 in the units place and the digit
2 in the tens place i.e. 24.
Thus, the least such number is 24.
Since the last two digits should always be 2 and 4, successive numbers are obtained by
adding 100 to the least number.
The largest such number is 9924.

Thus, the numbers are 24, 124, 224, . . . 9924.


Thus, number of terms in the above series

last term 1st term 9924 24


= +1= + 1 = 100.
100 100

The correct answer is option C.

9. The first nine stamps are numbered 1, 2 . . . 9.


Thus, the number of digits used up to number the first nine stamps = 9.
From numbers 10 to 99, each number has two digits.
There are 99 10 + 1 = 90 numbers from 10 to 99.
Thus, number of digits used = 90 2 = 180.

From 100 to 999, each number has three digits.


There are 999 100 + 1 = 900 numbers.
Thus, number of digits required = 900 3 = 2700.

However, then the total number of digits would be exceeded i.e. the last number is much
less than 999.

We have already used up 9 + 180 = 189 digits.


Number of digits left to be used = 855 189 = 666.

666
Since each number has three digits, # of numbers used = = 222.
3
Thus, 222 numbers starting from 100 would end in 100 + 222 1 = 321.
Thus, the student wrote all numbers from 1 to 321 to use 855 digits.
Thus, he had 321 stamps.

The correct answer is option B.

19992016 Manhattan Review www.manhattanreview.com


136 Number Properties Guide Solutions

10. There are six digits available, of which we need to select five to make a number.
A number is divisible by 3, if the sum of its digits is divisible by 3.
Hence, we need to choose five of the six digits in such a way that their sum is divisible
by 3.

The sum of all the six digits = 0 + 1 + 2 + 3 + 4 + 5 = 15, which is a multiple of 3.

Since we need to remove a digit such that the sum remains divisible by 3, the digit to
be removed should also be divisible by 3 i.e. digits 0 or 3.

Thus, we have the following cases:

(1) Digits used: 1, 2, 3, 4, 5: Number of five digit numbers = 5! = 120.


(2) Digits used: 0, 1, 2, 4, 5: Number of five digit numbers = 4 4 3 2 1 = 96 (since
0 cannot be used in the ten-thousands position, it can be filled in four ways and
not five).

Hence, total number of five digit numbers divisible by three = 120 + 96 = 216.

The correct answer is option D.

11. A number is divisible by 6 implies that it is divisible by both 2 and 3.

Since a12a is divisible by 2, the digit in the units place i.e. a must be even.
Thus, possible values of a are: 2, 4, 6 or 8 (a cannot be 0 since the digit in the thousands
place of a four-digit number cannot be 0).
Since the number is divisible by 3, the sum of digits must be divisible by 3.
Sum of the digits of the number a12a = a + 1 + 2 + a = 2a + 3.
Thus, (2a + 3) must be divisible by 3.

Since in the number (2a + 3), 3 is divisible by 3, 2a must be divisible by 3 i.e. a is


divisible by 3.
Thus, checking with the possible values of a i.e. 2, 4, 6 or 8, we see than only a = 6
satisfies the above condition.
Hence, we have: a = 6.

The correct answer is option D.

Alternatively, you can plug in the values of a from the options, and check which number
thus formed is divisible by 6.

www.manhattanreview.com 19992016 Manhattan Review


Number Properties Guide Solutions 137

12. The units digit of 1322 is the same as that of 322 ; it is governed by the digit at the unit
place.
The same logic works for the other numbers too.
Thus, the question is equivalent to finding the units digit of 322 222 + 727 525 .

Since the exponents of 3 and 2 are the same, we can write 322 222 = (3 2)22 = 622 .

We know that the last digit of 6 raised to any positive integer exponent is always 6.

Similarly, for 727 525 , we observe that 727 is odd and 525 has units digit of 5 (since the
last digit of 5 raised to any positive integer exponent is always 5).

Thus, the last digit of 727 525 is essentially an odd number multiplied with 5 which
will always be 5 irrespective of the odd number being multiplied to.

Hence, the last digit of 1322 1222 + 1727 2525 is the last digit of 6 + 5 = 1.

The correct answer is option A.


7 7 7 49
13. 7 percent of 7 7 = 7 % of 7 7 = = = 0.49.
100 100

The correct answer is option B.

14. (5 106 ) + (9 104 ) + (2 102 ) = 5000000 + 90000 + 200 = 5090200.

The correct answer is option D.

Alternate Approach:

The value can be inferred mentally.


     
Since maximum power of the number 5 106 + 9 104 + 2 102 is 106 , which
will have six 0s.
So the correct answers could be either D or E. Second term (9 104 ) must have four
digits after 9, so the correct answer is 5090200.

2 2
15. xy + y z + (z x)2
=> x 2 2xy + y 2 + y 2 2yz + z2 + z2 2zx + x 2
  

=> 2 x 2 + y 2 + z2 xy yz zx


=> 2 x 2 yz + y 2 xz + z2 xy
   

=> 2 {(5) + 1 + 7}

19992016 Manhattan Review www.manhattanreview.com


138 Number Properties Guide Solutions

=> 6

The correct answer is option B.

www.manhattanreview.com 19992016 Manhattan Review


Number Properties Guide Solutions 139

5.1.2 Even/Odd/Consecutive/Prime numbers

16. In this problem, both a and b are positive integers.


In the equation, we have two terms which are multiples of 5 viz. 5b and 315.
Hence, a2 must be a multiple of 5.

However, since a2 is a perfect square and a multiple of 5, a2 must be a multiple of 25


implying that a is a multiple of 5.

Since we need to find the largest value of a, we choose the largest multiple of 5 such that
its square is less than 315.
Thus, we choose a = 15 (since 202 = 400 > 315)
Hence, we have: 152 + 5b = 315 => 5b = 315 225 = 90 => b = 18.

However, in this case b = 18 is not a prime number; the question states that b is a prime
number.

Taking a = 10 (the next largest possible multiple of 5)


We have: 102 + 5b = 315 => 5b = 215 => b = 43: a prime number.
Thus, the largest possible value of a = 10.

The correct answer is option C.

17. Results of multiplication of Even and Odd numbers


Even Even = Even
Even Odd = Even
Odd Odd = Odd
It must be noted that if only one even number is present, the product is even;
whereas if product of two numbers is Odd, then both the numbers must be Odd.

Let us look at the options one by one.


(A) mn + 2n : We can write this expression as
[(odd or even) odd + even odd] = [(odd or even) odd + even]. Since we do not
know the nature of m, we cannot predict whether [mn + 2n] is odd or even.

(B) mn + 2m : We can write this expression as


[(odd or even) odd + even (odd or even)] = [(odd or even) odd + even].

Since product of even or odd with any even number is even, so we can write even.
[(odd or even)] = even, but we cannot predict whether [(odd or even) odd] is odd
or even.

19992016 Manhattan Review www.manhattanreview.com


140 Number Properties Guide Solutions

(C) (n + 3) (m 3) : We can write this expression as


[(odd + odd) (even or odd odd)] = [even (even or odd odd)] . We need not
bother about the fate of (even or odd odd), because whether it is odd or even,
it is multiplied by an even number. Further, we already know that any number
multiplied by an even number will result in an even number. So (n + 3)(m 3) must
be an even number.

The correct answer is option C.

18. The question is if n is even, which of the following CANNOT be odd?

Results of Even and Odd numbers

Even Even = Even


Even Odd = Even
Odd Odd = ODD

It must be noted that if there is only one even number is present, the product is
even, whereas if product of two numbers is Odd, then both the numbers must be Odd.

Let us look at the options one by one.

I. 2n + 7: In this option, 2 and n are even, whereas 7 is odd.


So 2n + 7 = even even + odd = even + odd = odd. Thus, 2n + 7 is an odd number.

II. 3n 2: In this option, 2 and n are even, whereas 3 is odd.


So 3n 2 = odd even even = even even = even. Thus, 3n 2 is an even
number. Correct answer.

III. 5n2 + 1: In this option n is even, whereas 5 and 1 are odd.


So 5n2 + 1 = odd even even + odd = even + odd = odd. Thus, 5n2 + 1 is an odd
number.

The correct answer is option B.

19. Consecutive odd integers from one are: 1, 3, 5, 7, 9, . . .


We see that there is a constant gap of 2 between consecutive terms of the above series
and hence, it constitutes an arithmetic progression.

For an arithmetic progression, the sum of terms is given as:

first term + last term


 
(number of terms)
2
In the above arithmetic progression, the 1st term is 1 and the last term is p .

www.manhattanreview.com 19992016 Manhattan Review


Number Properties Guide Solutions 141

Since we start with 1 (an odd  number) and


 end with p, another odd number, the
p1 p+1
number of terms in the series = +1 = (for example: For the series 1,
2 2
91

3, 5, 7, 9; the number of terms = +1=5 .
2

Thus, we have:

1+p p+1
   
= 225
2 2
p+1 2
 
=> = 225
2
p+1
=> = 15
2
=> p = 29.

Alternate Approach:

Let us take a few terms of the series and see if there is any pattern apparent in the value
of the sum of the terms:

One term: 1: Sum = 1 = 12


Two terms: 1, 3: Sum = 4 = 22
Three terms: 1, 3, 5: Sum = 9 = 32

Thus, we observe that the sum is actually the square of the number of terms added.

Since 225 = 152 , it implies that we need to add 15 terms to get the sum as 225.
Thus, we need to find the 15th term of the series: 1, 3, 5, 7 . . .
The nth term of an arithmetic progression is given by: a + (n 1)d; where a is the 1st
term, n is the number of terms and d is the common difference.

Here: a = 1, n = 15 and d = 2.

Thus, the 15th term = 1 + (15 1) 2 = 29.

The correct answer is option C.

20. Since we need even numbers divisible by 13, we should look at numbers
13 2, 13 4, 13 6 . . .

The last number less than 300 divisible by 13 is 299 = 13 23.

19992016 Manhattan Review www.manhattanreview.com


142 Number Properties Guide Solutions

Since we need even numbers, the last multiple of 13 would be 13 22.

Thus, the required set of numbers is: 13 2, 13 4, 13 6, . . . 13 20, 13 22, i.e., 11


numbers.

However, we do not want numbers divisible by 7.

In the above set, there is only one multiple of 7 i.e. 13 14.


Thus, we need to exclude it.

Hence, the # of such numbers = 11 1 = 10.

The correct answer is option B.

21. The factors of a b; where a and b are prime are 1, a, b and a b.

Thus, we have:
1 + a + b = 31
=> a + b = 30
Since a and b are prime, we can have the following cases for a + b = 30:

a b
7 23
11 19
13 17

Thus, there are three possible numbers: 7 23 = 161, 11 19 = 209, and 13 17 = 221.

The correct answer is option C.

22. Let the number be a2 , where a is a prime number.

The factors of a2 are 1, a and a2 .

Thus, we have:

1 + a + a2 = 57
=> a + a2 = 56
=> a2 + a 56 = 0
=> a2 + 8a 7a 56 = 0
=> a (a + 8) 7 (a + 8) = 0

www.manhattanreview.com 19992016 Manhattan Review


Number Properties Guide Solutions 143

=> (a 7) (a + 8) = 0
=> a = 7 or 8

Since a is a prime number, we have a = 7.

Hence, we have only one such number.

The correct answer is option A.

You may cross check this. For a2 = 49, we have factors 1, 7, & 49, and their sum
= 1 + 7 + 49 = 57.

23. We need to work with the options:

Option A: Factors of 12 (other than 12) are: 1, 2, 3, 4, and 6.


Sum of the above factors = 1 + 2 + 3 + 4 + 6 = 16 6= 12.

Hence, option A is incorrect.

Option B: Factors of 18 (other than 18) are: 1, 2, 3, 6, and 9.


Sum of the above factors = 1 + 2 + 3 + 6 + 9 = 21 6= 18.

Hence, option B is incorrect.

Option C: Factors of 20 (other than 20) are: 1, 2, 4, 5, and 10.


Sum of the above factors = 1 + 2 + 4 + 5 + 10 = 22 6= 20.

Hence, option C is incorrect.

Option D: Factors of 21 (other than 21) are: 1, 3, and 7.

Sum of the above factors = 1 + 3 + 7 = 11 6= 21.

Hence, option D is incorrect.

Hence, option E must be correct. Let us verify:

Option E: Factors of 28 (other than 28) are: 1, 2, 4, 7, and 14.

Sum of the above factors = 1 + 2 + 4 + 7 + 14 = 28.

19992016 Manhattan Review www.manhattanreview.com


144 Number Properties Guide Solutions

The correct answer is option E.

Few perfect numbers are 6, 28, 496.

24. Since we need even numbers from 10 to 100, we express them as multiples of 2:

10 = 2 5 (first one)

100 = 2 50 (last one)

Thus, there are 50 5 + 1 = 46 even integers from 10 to 100.

From 2 5 to 2 50, the multiples of 7 are: 2 7, 2 14, 2 21, 2 28, 2 35, 2 42 and
2 49 i.e. there are seven numbers.

Hence, the number of even numbers not a multiple of 7 are: 46 7 = 39.

The correct answer is option C.

www.manhattanreview.com 19992016 Manhattan Review


Number Properties Guide Solutions 145

5.1.3 Divisibility

25. Since p 2 is a perfect square number and is divisible by 5, which is not a perfect square,
p 2 must be a multiple of 25 and p itself must be a multiple of 5.
Now, we do not know whether p is an even multiple of 5 (i.e. divisible by 10) or an odd
multiple of 5 (i.e. not divisible by 10).
Since the question asks us to find a term divisible by 10, among the options, we must
have 2p as one of the term.

Thus, the correct answer can only be options C or D.

 
Again, since q is not divisible by 5, option D, 2 5p + q = 10p + 2q will not be divisible
by 5 (though 10p is divisible by 5, 2q is not divisible by 5) or will not be divisible by 10
either.

Hence, the answer must be option C.

Let us see how.

Option C: 2((p + 5q
q ) = 2p
p + 10q
q

It has two terms: 2p (which is divisible by 10 as discussed earlier) and 10q (also divisible
by 10). Hence, (2p + 10q) is divisible by 10.

The correct answer is option C.

Alternate Approach:

This question can solved with a plug-in value approach.

Let us assume that p 2 = 25 (perfect square and divisible by 5) => p = 5 and q = 3 (not
divisible by 5). By plugging in the values in options, we find that only option C works.

26. Let us calculate the number of integers divisible by 6.


The first integer divisible by 6 within the given range of 150 to 250 is 150.
Every 6th integer after this will be divisible by 6.
Thus, the set of integers divisible by 6 is {150, 156, 162, . . . , 246}
246 150
So, number of integers in the series = + 1 = 17.
6

We can also calculate the same in the following manner:


The first integer divisible by 6 within the given range of 150 to 250 is 150 = 6 25.

19992016 Manhattan Review www.manhattanreview.com


146 Number Properties Guide Solutions

The last integer in the series is 246 = 6 41.


So, the number of integers in the series = 41 25 + 1 = 17.

Similarly, let us calculate the number of integers divisible by 5.


The first integer divisible by 5 within the given range of 150 to 250 is 150 = 5 30.
The last integer in the series is 250 = 5 50.
So, the number of integers in the series = 50 30 + 1 = 21.

Now, few integers would be common in both the above series. These integers would be a
multiple of the LCM of 6 and 5 i.e. 30; we must exclude them once.
The first integer divisible by 30 within the given range of 150 to 250 is 150 = 30 5.
The last integer in the series is 240 = 30 8.

So, the number of integers in the series = 8 5 + 1 = 4.


So, the number of integers which are divisible by 6 or 5 or both are: 17 + 21 4 = 34.
Total number of integers in the range from 150 to 250 = 250 150 + 1 = 101.

Thus, the number of integers neither divisible by 6 nor by 5 is: 101 34 = 67.

The correct answer is option C.

27. Say the number X is such that X = 7n + 2; where n is a non-negative integer;


n = {0, 1, 2, 3 . . . }.

Option analysis
I. X is odd: It is not must be true. If n = 2, X = 7 2 + 2 = 16, which is even.

II. X cannot be a prime number: It is not must be true. If n = 0, X = 7 0 + 2 = 2,


which is prime.

III. (X + 5) divided by 7 has remainder 0: This condition must be true.

X + 5 = 7n + 2 + 5 = 7n + 7 = 7(n + 1), which is a multiplicand of 7 and hence


completely divisible by 7 or leaves a remainder 0.

The correct answer is option C.

Alternate Approach:

If X, divided by 7 leaves a remainder 2, X would have either value from set {2, 9, 16,
23,. . . ..}.

www.manhattanreview.com 19992016 Manhattan Review


Number Properties Guide Solutions 147

Option analysis

I. X is odd: It is not must be true. Have a look at the set above. 16 is even.

II. X cannot be a prime number: It is not must be true. Have a look at the set above. 2
is prime.

III. (X + 5) divided by 7 has remainder 0: This condition must be true. For any value
from the set, if you add 5, the number would be completely divisible.

Another way of understanding this is: Since X, divisible by 7 has 2 as remainder. This
means that X is short by 5 to make it completely divisible. Hence X + 5 would be
completely divisible or leaving a reminder of 0.

The correct answer is option C.


28. We know that p! = 1 2 3 p 1 p.

Thus, we have:

1! = 1
2! = 1 2 = 2
3! = 1 2 3 = 6
4! = 1 2 3 4 = 24
5! = 1 2 3 4 5 = 120
6! = 1 2 3 4 5 6 = 5! 6
7! = 1 2 3 4 5 6 7 = 5! 42, etc.

Thus, we observe that all factorials from 5! onwards are multiples of 10 and hence
would not leave any remainder when divided by 10.

The remainder would only come from the first four terms which are not a multiple of 10.
Thus, we have: 1! + 2! + 3! + 4! = 1 + 2 + 6 + 24 = 33.
Thus, the remainder when N is divided by 10 is the same as when 33 is divided by 10
which is 3.

Lets test this.

1! + 2! + 3! + 4! + 5! = 1 + 2 + 6 + 24 + 120 = 153.

Similarly, for others. . .

19992016 Manhattan Review www.manhattanreview.com


148 Number Properties Guide Solutions

Hence, the remainder is 3.

The correct answer is option D.

250
29. Number of multiples of 5 between 1 and 250 = 1 = 50 1 = 49 (since we want
5
the numbers to be between 1 and 250, we cannot include 250 as a possible multiple of
5).

Number of multiples of 7 between 1 and 250 is the quotient on dividing 250 by 7 (we
250
do not subtract 1 since 250 is not a multiple of 7) i.e. quotient of i.e. 35.
7

However, there are some numbers which are multiples of both 5 and 7 i.e. 35 which
have been counted in both the above cases. We need to remove them since we do not
want numbers divisible by both 5 and 7.

Number of multiples of 35 between 1 and 250 is the quotient on dividing 250 by 35 i.e.
250
quotient of i.e. 7.
35

Thus, the total # of such numbers = (49 7) + (35 7) = 42 + 28 = 70.

The correct answer is option B.

It is to be noted that 49 + 35 7 = 77 is a wrong answer as 77 includes those numbers


too that are divisible by both 5 and 7.

30. Given that, 10! + 11! + 12!


= 10! + 11 10! + 12 11 10!
= 10! (1 + 11 + 12 11)
= 10! (12 + 12 11)
= 10! 12 (1 + 11)
= 10! 12 12

The highest exponent of 2 in 10! is calculated as shown below. We have seen this
process in detail in earlier questions.

10
=5
2
5
= 2.5 = 2 (Integer part)
2
2
=1
2

www.manhattanreview.com 19992016 Manhattan Review


Number Properties Guide Solutions 149

Thus, the exponent of 2 in 10! is 5 + 2 + 1 = 8.

2
Again, 12 12 = 122 = 22 3 = 24 32 .

Thus, the exponent of 2 in 12 12 is 4.

Since the terms are multiplied, the exponents of 2 will be added.

Thus, the exponent of 2 in 10! + 11! + 12! is: 8 + 4 = 12.

Hence, the highest exponent 2 in (10! + 11! + 12!) is 12.

The correct answer is option C.

 
31. p+4 p+6
=> p 2 + 10p + 24

=> p p + 10 + 24.


Thus, p p + 10 + 24 must be divisible by p.

This is possible when 24 is divisible by p.

Thus, p should be the factors of 24 (other than 1) i.e. 2, 3, 4, 6, 8, 12 and 24.

Hence, there are seven possible values of p.

The correct answer is option B.

19992016 Manhattan Review www.manhattanreview.com


150 Number Properties Guide Solutions

5.1.4 Exponents

32. 220 = 215 x + y


220 y 220 y
=> x = 15
= 15
15 .
2 2 2

Since x is an integer, y must be divisible by 215 .


Thus, y = 215 k; k is an integer.

220
Hence, 220 = 215 x + 215 k => 220 = 215 (x + k) => x + k = = 25 .
215

We need to find such values of x and k such that |x y| is minimum or the difference
between x and y is minimum.

Putting k = 0, we have x = 25 = 32.


Thus, y = 215 k = 0.

Hence, the difference between x and y is 32 0 = 32.

Putting k = 1, we get x = 25 1 = 31.


Thus, y = 215 k = 215 = a higher value.

Hence, the difference between x and y = |x y| = a higher value.

Similarly, putting k = 2, we get x = 25 2 = 30.


Thus, y = 215 .2 = 216 = a higher value.

Hence, the difference between x and y = |x y| = a higher value.

In this case, the difference between x and y is much greater than 32.
As k increases, the difference between x and y would increase.

Hence, the minimum difference is 32 or 25 .

The correct answer is option C.

www.manhattanreview.com 19992016 Manhattan Review


Number Properties Guide Solutions 151

33. The units digit of exponents of 7 and 3 follow a cycle of 4 places as shown below:

Number with exponent Unit digit Number with exponent Unit digit

71 7 31 3

72 9 32 9

73 3 33 7

74 1 34 1
30
Thus, 7123 = 74 73 => Units digit is 3.

10
342 = 34 32 => Units digit is 9.

Hence, the units digit of 7123 + 342 is the unit digit of 3 + 9 = 12 i.e. 2.

The correct answer is option B.

34. Approach 1:

Find out the cube root of options and figure out whether the value is an integer value or
not? The option does not return an integer is not a perfect cube.
 1
(A) Cube root of m6 n3 = m6 n3 3 = m2 n. This is a integer, hence option A is a


perfect cube.

(B) Cube root of mn: Since m and n are perfect cube, their products will also be
perfect cube.

(C) Cube root of mn + 125: This is the correct answer. Perfect cube + Perfect
cube will not necessarily be a perfect cube. For example, if m = n = 1, then
mn + 125 = 1 + 125 = 126; not a perfect cube.

(D) Cube root of m: Since m is perfect cube, m will also be perfect cube.

h i1
Cube root of (m n)9 = (m n)9 = (m n)3 . This is an integer; hence option
3
(E)
E is a perfect cube.

The correct answer is option C.

Approach 2:

Pick up some value. Say we take m = 1, and n = 8.

19992016 Manhattan Review www.manhattanreview.com


152 Number Properties Guide Solutions

(A) m6 n3 = 16 .83 = 1.83 = 83 . Cube root of 83 is 8, which is an integer; hence option A


is a perfect cube.

(B) mn = 1.8 = 8. Cube root of 8 is 2, which is an integer; hence option B is a perfect


cube.

(C) mn + 125 = 1.8 + 125 = 133. Cube root of 133 is not an integer; hence option C is
not a perfect cube. Correct answer.

(D) m = 1. Cube root of 1 is 1, which is an integer; hence option D is a perfect


cube.

(E) (m n)9 = (1 8)9 = (7)9 . Cube root of (7)9 is (7)3 , which is an integer;
hence option E is a perfect cube.

35. This is a MUST BE TRUE type of question, we must pick at least one value for each option
such that the option breaks. If it does holds true for all cases, it is the correct answer.
It is to be noted that a 6= 0 means that a can take any valuesnegative, positive or
fractional but not 0.

I. 10a: Say if a = 1, 10a = 10. Since 10 < 1, this option is incorrect.

1 1 1 1 1
   
II. 10a2 : Say if a = , 10a2 = 10. . = . Since a, this option is
10 10 10 10 10
also incorrect.

III. 10 a: Say if a = 10, 10 a = 10 10 = 0. Since 0 a, this option is also incorrect.

The correct answer is option E.

36. 626 1443 1561


=> (2 3)26 (2 7)43 (3 5)61
=> 226+43 326+61 561 743
=> 269 387 561 743

Every 2 would combine with a 5 to result in 10 i.e. a 0 at the right in the product.
Since there are sixty nine 2s and sixty one 5s, there would be 69 61 = eight 2s left
over.

Thus: 269 387 561 743 = 28 387 743 1061




Since we need the rightmost non-zero digit, we can ignore 1061 .

www.manhattanreview.com 19992016 Manhattan Review


Number Properties Guide Solutions 153

The units digit of exponents of 7, 3 and 2 follow a cycle of 4 places as shown below:

Number Number Number


with Unit digit with Unitdigit with Unitdigit
exponent exponent exponent
71 7 31 3 21 2

72 9 32 9 22 4

73 3 33 7 23 8

74 1 34 1 24 6
2
28 => 24 => Units digit = 6.
21
387 => 34 33 => Units digit = 1 7 = 7.


10
743 => 74 73 => Units digit = 1 3 = 3.


Thus, the units digit of 28 387 743 is the units digit of 6 7 3 = 6.

The correct answer is option D.

37. Trailing zeros refers to the zeroes at the end of a number before the appearance of any
non-zero digit. For example, the number of trailing zeroes in 10200 is two and not three.

The trailing zero appears due to 10 appearing as a multiplicand i.e. if there are k trailing
zeroes, then the number has 10k as a factor.

10 itself has two factors 2 and 5.


Thus, if a number has p as the exponent of 2 and q as the exponent of 5, then the
exponent of 10 would be the lesser of p and q.

In order to find the exponent of 2 in the factorial of a number n, i.e. n!, we need to keep
dividing n successively by 2 and then adding the integer parts of the quotients attained
in each step.

Similarly, to find the exponent of 5 in n!, we need to keep dividing n successively by 5


and then adding the integer parts of the quotients attained in each step.

It is obvious that the sum of the quotients for 5 would be less than that for 2.

Hence, the exponent of 10 would be the same as the exponent for 5, which would also
be the number of trailing zeroes.

Here, we have to find the number of trailing zeroes in 30!.

19992016 Manhattan Review www.manhattanreview.com


154 Number Properties Guide Solutions

Hence, according to the argument above, we find the exponent of 5 in 30!, the process of
which is shown below:

30
=6
5
6
= 1.2 => 1 (Integer part)
5

Hence, the exponent of 5 in 30! = 6 + 1 = 7.

Hence, the number of trailing zeroes in 30! is 7.

The correct answer is option C.

Alternate Approach:

As discussed above that the exponents of 10 will bring in 0s in a number, and we know
that 2 5 makes 10, thus between a 2 and a 5, whichever has fewer 0s will make that
many 10s.

Since 5 > 2, it is obvious that there would be less number of 5s than 2s.

So let us count 5s in 30. We see that multiples of 5 are: 5, 10, 15, 20, 25(two 5s), and 30.
Each multiplicand except 25 would yield one 5, thus, total number of 5s are 7.

38. We need to have either a common base or a common exponent in order to compare the
values. However, either method is not possible in the above example.

Let us approach in a different way:

Let us work with the options:

50
Option A: 1550 < 1650 = 24 = 2200 . . . (i)
40
Option B: 3340 > 3240 = 25 = 2200 . . . (ii)
100
Option D: 3100 < 4100 = 22 = 2200 . . . (iii)
50
Option E: 1150 < 1650 = 24 = 2200 . . . (iv)

Thus, we observe that only 3340 > 2200 and all others are less than 2200 .

Hence, 3340 is the greatest of all the options.

The correct answer is option B.

www.manhattanreview.com 19992016 Manhattan Review


Number Properties Guide Solutions 155

5.1.5 Factors/Multiplicands/LCM/HCF

39. Let the number of coins be n.


We observe that:
n when divided by 3 leaves a remainder 1 i.e. (n + 3 1) = (n + 2) would be divisi-
ble by 3
n when divided by 4 leaves a remainder 2 i.e. (n + 4 2) = (n + 2) would be divisi-
ble by 4
n when divided by 5 leaves a remainder 3 i.e. (n + 5 3) = (n + 2) would be divisi-
ble by 5
Thus, we have: (n + 2) is divisible by the LCM of 3, 4 and 5 i.e. 60.

Thus, the minimum value of n = 60 2 = 58. (LCM is the least value)


Let us verify: 58 divided by 3 leaves a remainder 1, divided by 4 leaves a remainder 2,
and divided by 5 leaves a remainder 3.

The correct answer is option D.

40. Since the numbers have 18 as their HCF, they must be multiples of 18.

Let the numbers be p = 18a and q = 18b; where a and b are numbers having no
common factor except 1, we can also call them co-primes.

Thus, we have:

18a + 18b = 216

216
=> a + b = = 12.
18

Since a and b have no common factors (except 1), the possible cases are:

a b a
p = 18a b
q = 18b

1 11 18 198

5 7 90 126

7 5 126 90

11 1 198 18

Thus, there are only two pairs of such numbers: (18, 198) and (90, 126).

The correct answer is option A.

19992016 Manhattan Review www.manhattanreview.com


156 Number Properties Guide Solutions

41. Since the numbers have 12 as their HCF, they must be multiples of 12.

Let the numbers be p = 12a and q = 12b; where a and b are numbers having no
common factor except 1.

Thus, their least common multiple = 12ab.

Thus, we have:

12ab = 360

360
=> ab = = 30.
12

Since a and b have no common factors (except 1), the possible cases are:

a b a
p = 12a b
q = 12b

1 30 12 360

2 15 24 180

3 10 36 120

5 6 60 72

6 5 72 60

10 3 120 36

15 2 180 24

30 1 360 12

Thus, there are four pairs of such numbers: (12, 360), (24, 180), (36, 120) and (60, 72).

The correct answer is option C.

42. Given that, A = 12! + 3 13! + 24


=> 12! + 3 13 12! + 24
=> 12! (1 + 39) + 24
=> 12! 40 + 24
=> 8 (12! 5 + 3)
Thus, A is definitely a multiple of 8; however there is no option as 8, so we analyze
further.

www.manhattanreview.com 19992016 Manhattan Review


Number Properties Guide Solutions 157

Again, 12! is a multiple of 3.

Thus, (12! 5 + 3) is also a multiple of 3.

Hence, A is a multiple of both 3 and 8 i.e. 24.

We can verify that A is not a multiple of 5.

Referring to the expression A = 12! + 3 13! + 24, we see that both 12! and 13! are
multiples of 5; however, 24 is not.

Hence, A is not a multiple of 5, thus, options A, B, C and D are incorrect.

The correct answer is option E.

Alternate Approach:

Given that, A = 12! + 3 13! + 24, the third term of A is 24, and we know that 12! =
1 2 3 8 11 12 and 13! = 1 2 3 8 11 12 13 are multiples
of 24, thus A must be a multiple of 24.

43. The number of boxes required would be a minimum if each box contains the maximum
number of items of a single type.

Since we need to have the same number of items in each box, the number of items should
be the highest common factor of the number of items of each category
Highest common factor of 18, 45 and 63 = 9.

18
Thus, number of boxes required for pens = = 2.
9
45
Similarly, number of boxes required for pencils = = 5.
9
63
Similarly, number of boxes required for erasers = = 7.
9

Hence, the number of boxes = 2 + 5 + 7 = 14.

The correct answer is option E.

44. We need to work with the options:

Option A: Since p is a factor of q, we can have a situation in which p = q (since q is a


factor of itself).

19992016 Manhattan Review www.manhattanreview.com


158 Number Properties Guide Solutions

Thus, q = p 1 is a possible scenario.


Hence, option A is incorrect.

Option B: Going by the above logic, we can have p = q.


Thus, number of factors of p and q can be equal.
Hence, option B is incorrect.

Option C: Since p is a factor of q, we can have p = 1.


Then, the highest common factor of p and q can be 1.
Hence, option C is incorrect.

Option D: Since p is a factor of q, p is either less than or equal to q and we can say that
q = p k, where k 1.
Thus, we can say that q is a multiple of p.
Thus, the least common multiple of p and q must be q.
Hence, option D is correct.

Option E: Since p is a factor of q, p may be at least 1 or at the most q.


If p = 1, then obviously p 2 q.

Hence, option E is incorrect.

The correct answer is option D.

www.manhattanreview.com 19992016 Manhattan Review


Number Properties Guide Solutions 159

5.1.6 Inequality

45. This is a can be true type of question.


Let us see the difference between must be true and can be true types of questions.
For must be true types of questions, under all circumstances, all the possible values of
variable(s) must satisfy the equality or the inequality.
For can be true types of questions, at least one, not necessarily all, values of variable(s)
should satisfy the equality or the inequality.
Lets draw the range of 0 < mn < 1 on number line.

mn

-5 -4 -3 -2 -1 0 1 2 3 4 5
It is evident that m n is a positive number.
Since the product of two variables is positive, both are either positive or negative.
Let us analyze the options one by one.
(A) m < 1 and n < 1
m < 1 & n < 1 mean that both m & n are negative numbers. m and n both
being negative satisfies above criterion. But this is not the correct answer, since
product of m & n, if each equals 1, makes mn exactly equal to 1. But their actual
values are less than 1.

On a number line, when a negative number decreases in value, its absolute value
increases.

For the given condition, m & n have values whose absolute values are greater than
1. This makes the product of m & n greater than 1.
(B) m < 1 and n > 0
Here, m < 1 and n > 0 means that m is negative, and n is positive. It does not
satisfy our criterion of either both the variables being positive or both the variables
being negative, so this is not the correct answer.
(C) m > 1 and n < 1
Here, n < 1 means that n is negative, whereas m > 1 means that m can be
negative, positive, or even 0. This may or may not satisfy our criterion. It will
depend on what value m assumes. Since the question is not a must be true kind,
we must analyze further.

As stated earlier that for can be true type of questions, get at least one value that
satisfies the criterion. Our criterion is that the product of m & n should lie between

19992016 Manhattan Review www.manhattanreview.com


160 Number Properties Guide Solutions

0 & 1. Can you pick any suitable values for m and n?

These may be,


3
Say, m = 3/4, 1/2, 1/4, 0, 1/4, 1/2, 1, 2, 3 . . . . . . and n = , 2, 3, 4 . . .
2
1
If we pick m = 1/4 and n = 2, mn = , lying in the range of mn.
2

So option C is the correct answer.

There is no need to analyze other options as we are satisfied with option C, however
for the sake of your understanding, we discuss other options too.
(D) m > 1 and n < 1
Here, m > 1 means that m is positive, and n is negative. It does not satisfy our
criterion of either both being positive or both being negative. It is not the correct
answer.
(E) m > 1 and n > 1
Here, m > 1 & n > 1 mean that both m & n are positive. It satisfies our criterion,
but this is not the correct answer. Since both m & n will have values greater than 1,
which makes their product greater than 1. So mn would lie on the right hand side
of the actual range of 0 < mn < 1. It is not the correct answer.
The correct answer is option C.

46. p + q < 11, and p q > 8 are two linear inequalities. Looking at the options, we find
that we have to find the range of q. This calls for solving the inequalities. However the
challenge with the given inequalities is that their directions are opposite. We must make
their directions same and then add them to get the range of q.
Also, given that p and q are non-negative integers.

=> p and q can be 0 or positive integers.

Approach 1:

Given that, p + q < 11, and p q > 8.

Say, we decide to change the direction of the second inequality. To get that, we multiply
it with 1.
=> 1 (p q > 8)
=> p + q < 8. (Note that the sign of inequality is reversed)

Now we can add both the inequalities.


p + q < 11 - - - - -(1)
p + q < 8 - - - - -(2)

www.manhattanreview.com 19992016 Manhattan Review


Number Properties Guide Solutions 161

We get, 2q < 3
3
=> q < => 0 or 1
2
=> Correct option seems to be D, but we must check the corresponding values of p for
q.

If q = 0, x is either 9 or 10: both are qualified values of p.

But for q = 1, from the first inequality p + q < 11, we get p < 10. And from the second
inequality p q > 8, we get p > 9. This is not possible, it vanishes p altogether as there
is no integer lying between 9 and 10; hence this option is incorrect. So only possible
value left to be considered is q = 0.

Option E suggests q < 1, which means that q = 0, and gives p = 9 or 10.

The correct answer is option E.

Approach 2:

Another approach could be Plug in value approach.

Let us analyze the options one by one.

(A) q<3
This gives 0, 1, and 2 as three possible values of q. Let us try q = 2 in the
inequalities. p + q < 11 gives p < 9, but p q > 8 gives p > 10. This is not possible.
This option is incorrect.

(B) q>2
When these kinds of options are there, we must plug in an unusual yet qualified
values such as q = 100. q = 100 renders negative value for p, which is not possible
as p is a non-negative integer. This option is incorrect.

(C) 2 < q < 10


From option A, we already know that q cannot be greater than 2, hence this option
is also incorrect.

(D) q = 0 or 1
q = 0 gives p = 9 or 10: qualified values for x. But for q = 1, from the first
inequality p + q < 11, we get p < 10. And from the second inequality p q > 8, we
get p > 9. As discussed in approach 1, it is not possible; hence this option is also
incorrect. So only possible value left is q = 0.

(E) This is the correct answer. q < 1 means that q = 0 and p = 9 or 10.

19992016 Manhattan Review www.manhattanreview.com


162 Number Properties Guide Solutions


x
47. Given that, < 1

y
=> |x| < |y|

The above result is possible in three scenarios:

(1) If both x & y being +ive, then say x = 10, then y > 10;
(2) If both x & y being ive, then say x = 10, then y < 10;
(3) If any one of x & y being +ive and other being ive, then
(a) say x = +ive = 10, then y < 10;
(b) say x = ive = 10, then y > 10

Based on these findings, we analyze each option one by one. Note that this is a must
be true kind of question, which means that in each circumstance, the correct option
should satisfy the constraints.

An optimum approach with must be true kind of questions is that you must try to
prove an option WRONG. If you are satisfied that in each circumstance, the option
satisfies the constraints, it is the correct answer.

Let us analyze the options one by one.

y
(A)
x
It would fail if the scenario is # 3 i.e. any one of x & y is +ive and other is ive.

y
(Negative) < 1.
x
!2
x
(B)
y
!2
x x
It would fail in all the scenarios. Given that < 1 => < 1.

y y
x
(C)
y
It would fail if the scenario is # 3 i.e. any one of x & y is +ive and the other is ive.

x
(Negative) < 1.
y

y
(D)
x
y
It would fail if the scenario is # 3b i.e. x is ive. (Negative) < 1.
x
s
y
(E)
x
Since |x| < |y|

www.manhattanreview.com 19992016 Manhattan Review


Number Properties Guide Solutions 163

y

=> 1 <
x ; note that we can multiply or divide an inequality by a number provided we

are sure of its sign. |x| being +ive will not affect the sign of the inequality.
s
y
=> 1 < x ; taking square root of both the sides.

s
y
=> 1 <
x ; greater than 1.

The correct answer is option E.

13 3y
48. The inequality 5x + 3y 13 can be reduced to x .
5

Since this is a Could be true type of question, any option that is true for even a single
value for y would be the answer.

13 3y
Since x 2, we can deduce that 2
5
=> 10 13 3y
=> 3 3y
=> 1 y; notice the sign change because of multiplication of a negative number.

Only one option qualifies and that is option E.

The correct answer is option E.

Alternatively, you can plug in the value of y in the inequality, and see whether x 2.
Only option E, i.e., y = 2 would qualify.

19992016 Manhattan Review www.manhattanreview.com


164 Number Properties Guide Solutions

1 1
 
49. If m > 0, the set of possible values of m would be . . . , , , 1, 2, . . . .
3 2

This is a MUST BE TRUE type of question. It means that for all possible values of m, the
option values must be correct. We must approach the problem such that we find out at
least one value that is incorrect.

I. Incorrect. Say if m = 1, m3 = 1 1

II. It can be reduced to m < m2 . Let us try a random value of m in 0 to 1 interval.


Always remember that for 1 > m > 0, the indexed values of m would always be
1 1
less than m. Say m = , then m2 = , which is less than m. This option is not
2 4
must true.

1 1
III. 9m 1 > 0 can be reduced to m > . Incorrect. m may also lie between 0 and .
9 9

IV. m3 > 0. Correct. If m > 0, m3 would always be greater than 0.

The correct answer is option E.

50. a2 4a < 221


=> a2 4a + 4 < 225; adding 4 on each side.
=> (a 2)2 < 225
=> (a 2)2 < 152
=> a 2 < |15|
=> 15 < (a 2) < 15
=> 15 + 2 < a < 15 + 2
=> 13 < a < 17.

Thus, the possible integer values that a can assume are: 12, 11,
1, 0, 1, 2, . . . 15, 16.

Thus, there are 12 integers from 12 to 1 (both inclusive), 0, and 16 integers from 1
to 16 (both inclusive); i.e. a total of 12 + 1 + 16 = 29 integers.

The correct answer is option C.

Alternate Approach:

a2 4a < 221
a2 4a 221 < 0

www.manhattanreview.com 19992016 Manhattan Review


Number Properties Guide Solutions 165

a2 17a + 13a 221 < 0


a(a 17) + 13(a 17) < 0
(a 17)(a + 13) < 0
=> 13 < a < 17.

As explained above, there are 29 integers for a within 13 < a < 17.

19992016 Manhattan Review www.manhattanreview.com


166 Number Properties Guide Solutions

5.2 Data Sufficiency

Data sufficiency questions have five standard options. They are listed below and will not
be repeated for each question.

A. Statement (1) ALONE is sufficient, but statement (2) ALONE is not sufficient to an-
swer the question asked.
B. Statement (2) ALONE is sufficient, but statement (1) ALONE is not sufficient to an-
swer the question asked.
C. BOTH statements (1) and (2) TOGETHER are sufficient to answer the question asked,
but NEITHER statement ALONE is sufficient to answer the question asked.
D. EACH statement ALONE is sufficient to answer the question asked.
E. Statements (1) and (2) TOGETHER are NOT sufficient to answer the question asked,
and additional data specific to the problem are needed.

www.manhattanreview.com 19992016 Manhattan Review


Number Properties Guide Solutions 167

5.2.1 Numbers & Digits

51. Distinct integers means that no two integers should be same.


Let us look at the statements one by one.

Statement 1:
Their product is 0.
There can be many scenarios for this.

(1) Three integers are 0, a, and b: Integers are distinct.


(2) Three integers are 0, a, and a: Integers are not distinct.
(3) Three integers are 0, 0, and b: Integers are not distinct.
(4) Three integers are 0, 0, and 0: Integers are not distinct.

So statement 1 alone is not sufficient.

Statement 2:
Their sum is 0.
There can be many scenarios for this too.

(1) Three integers are 0, a, and a: Integers are distinct.


(2) Three integers are 2, 3, and 5: Integers are distinct.
(3) Three integers are 0, 0, and 0: Integers are not distinct.

So statement 2 alone is also not sufficient.

Statement 1 and 2 together:


There can be two scenarios for this too.

(1) Three integers are 0, a, and a: Sum and product both are 0. Integers are distinct.
(2) Three integers are 0, 0, and 0: Sum and product both are 0. Integers are not distinct.

The correct answer is option E.

52. A number is divisible by 15 only when it is divisible by both 5 and 3.

Statement 1:
Since the sum of digits is 5, we have the following cases:

19992016 Manhattan Review www.manhattanreview.com


168 Number Properties Guide Solutions

a b ab

1 4 14

2 3 23

3 2 32

4 1 41

Thus, the possible numbers are 14, 23, 32 and 41; none of which are divisible by 15.

Hence, statement 1 is sufficient to answer the question, the answer is Noa unique
answer.

The correct answer would be between A/D.

Statement II:
Since the product of the digits is 4, we have the following cases:

a b ab

1 4 14

4 1 41

2 2 22

Thus, the possible numbers are 14, 41 and 22; none of which are divisible by 15.

Hence, statement 2 is sufficient to answer the question, the answer being No a unique
answer.

The correct answer is option D.

53. Statement 1:
(2x 3)2 = x 2
=> (2x 3)2 x 2 = 0
=> {(2x 3) x} {(2x 3) + x} = 0
=> (x 3) (3x 3) = 0
=> x = 1 or 3.
Thus, we obtain two values of x, 1 or 3.
Thus, statement 1 is not sufficient to answer the question since the answer maybe both
Yes or No.

www.manhattanreview.com 19992016 Manhattan Review


Number Properties Guide Solutions 169

The correct answer would be among B/C/E.

Statement 2:
(2x 3)2 = x
=> 4x 2 12x + 9 = x
=> 4x 2 13x + 9 = 0
=> 4x 2 4x 9x + 9 = 0
=> 4x (x 1) 9 (x 1) = 0
9
=> x = or 1.
4
9
Thus, we obtain two values of x, 1 or .
4
Thus, statement 2 is not sufficient to answer the question since the answer maybe both
Yes or No.

The correct answer would be between C/E.

Statements 1 and 2 together:


Combining both the statements, we get x = 1.

Hence, we get the answer by combining statements 1 and 2 and the answer is Yes.

The correct answer is option C.

54. Statement 1:

We have: 3x 2 8 = x

Squaring both sides: 3x 2 8 = x 2




=> 2x 2 = 8
=> x 2 = 4
=> x = 2 or 2.

However, the left hand side of the equation: 3x 2 8 is a square root which is always
positive.

Hence, the right hand side, x must also be positive.


Thus, x 6= 2.
Thus, we obtain only one value of x.
Hence, the value of x is distinct.
Thus, statement 1 is sufficient to answer the question and the answer is Yes.

19992016 Manhattan Review www.manhattanreview.com


170 Number Properties Guide Solutions

The correct answer would be between A/D.

Statement 2:

We have: x 2 + 5 = x + 1
Squaring both sides: x 2 + 5 = (x + 1)2
=> x 2 + 5 = x 2 + 2x + 1
=> x = 2
Thus, we obtain only one value of x.
Hence, the value of x is distinct.

Thus, statement 2 is sufficient to answer the question and the answer is Yes.

The correct answer is option D.

www.manhattanreview.com 19992016 Manhattan Review


Number Properties Guide Solutions 171

5.2.2 Even/Odd/Consecutive/Prime numbers

55. For the product of 10 integers to be negative, there must not be any integer, equal to 0,
and there must be odd number of negative integers in the list, because even number of
integers make product positive.

Statement 1:
There are at the most 4 negative integers.
This statement alone is not sufficient, because we do not know whether there is an integer
0 in the list and whether there are odd or even number of negative integers.
So statement 1 alone is not sufficient.

Statement 2:
There are at the most 5 positive integers.
Again, this statement alone is not sufficient, because we do not know whether there is an
integer 0 in the list and whether based on the count of positive integers, there are odd
or even number of negative integers.
So statement 2 alone is not sufficient.

Statement 1 and 2 together:


Let us take two extreme cases to analyze.

(1) There are 4 negative, and 5 positive integers. It means that the 10th integer would
be 0. So the product would be 0. Answer is No.
(2) There are fewer than 4 negative, and fewer than 5 positive integers. It means that
there are more than one 0 integers. Again, the product would be 0. Answer is No.

So statement 1 and 2 together are sufficient.

The correct answer is option C.

56. For the product of 12 integers to be negative, there must not be any integer equal to 0,
and there must be odd number of negative integers in the list, because even number of
integers makes the product positive.

Statement 1:
There are at the most 6 negative integers.
This statement alone is not sufficient, because we do not know whether there is an integer
0 in the list and whether there is an odd or even number of negative integers.
Statement 1 alone is not sufficient.

Statement 2:

19992016 Manhattan Review www.manhattanreview.com


172 Number Properties Guide Solutions

There are at least 6 positive integers.


Again, this statement alone is not sufficient, because we do not know whether there is an
integer 0 in the list and whether based on the count of positive integers, there are odd
or even number of negative integers.
Statement 2 alone is not sufficient.

Statement 1 and 2 together:


Let us take two cases to analyze.
(1) There are 6 negative, and 6 positive integers. It means the product would be posi-
tive. Answer is NO.
(2) There are 5 negative, and 7 positive integers. It means the product would be nega-
tive. Answer is YES.
Statement 1 and 2 together are not sufficient.

The correct answer is option E.

57. Results of Even and Odd numbers


Even. Even = Even
Even. Odd = Even
Odd. Odd = Odd
It must be noted that even if only one even number is there, the product is even,
whereas if product of two numbers is Odd then both the numbers must be Odd.

Statement 1:
n is odd
To know the nature of (n + 3)(m 3), we must find out the nature of m or/and n.
=> (n + 3)(m 3) = (Odd + Odd) (m - odd) = Even.(m - odd) = Even;
We need not bother about the nature of (m - odd), because irrespective of its nature, its
multiplication with an even number will make it even.
So statement 1 alone is sufficient.

Statement 2:
m is even
(n + 3)(m 3) = (n + Odd) (even - odd) = (n + Odd).Odd = CANNOT SAY ANYTHING. We
do not know the nature of (n + Odd).
If (n +Odd) is odd, its multiplication with an odd number will make it odd.
However if (n + Odd) is even, its multiplication with an odd number will make it even.
So statement 2 alone is not sufficient.

The correct answer is option A.

www.manhattanreview.com 19992016 Manhattan Review


Number Properties Guide Solutions 173

58. Statement 1:
2n2 3 is odd
To know the nature of 5n2 + 1, we must find out whether n is even or odd.
In 2n2 3, 2, an even number is multiplied to n2 . Whether n is odd or even, product of
2n2 will always be even. So this information is not conclusive about n.
So statement 1 alone is not sufficient.

Statement 2:
3n2 2 is odd
Now, we know that 3n2 2 is odd. We can write it as
Odd.n2 Even = Odd; nature on n is not yet known.
We can rewrite it as Odd.n2 = Odd + Even => Odd.n2 = Odd.

This shows that product of two numbers is odd. We know that to get product Odd, both
the numbers must be odd. It means that n2 is odd. This again means that n is odd.

If n is odd, we can surely find out whether 5n2 + 1 is even.


5n2 + 1 => Odd.Odd + Odd = Odd + Odd = Even. Answer is Yes.
So statement 2 alone is sufficient.

The correct answer is option B.

59. The product abc will be odd only if each of a, b and c are odd (if any one of them is
even, the product will be even).
Thus, we need to confirm the nature of each of the three numbers to make sure whether
the product is odd.

Statement 1:
a + 1 is even => a is (even 1) => a is odd.
However, we do not know anything about b and c. If one of them is even, the product
will be even, else odd.
Thus, the answer may be both Yes as well as No.
Hence, statement 1 is not sufficient.

Hence, the answer would be among B/C/E.

Statement 2:
(b c) (a c) is odd => (b c) is odd and (a c) is odd.
(b c) is odd => one of b and c is odd and the other is even.

19992016 Manhattan Review www.manhattanreview.com


174 Number Properties Guide Solutions

Similarly, (a c) is odd => one of a and c is odd and the other is even.

Thus, we have two cases:

(1) c is odd while a and b are even


(2) c is even while a and b are odd
In either case, the product abc is even.
Thus, the answer to the question is No, a unique answer.

Hence, statement 2 is sufficient to answer the question.

The correct answer is option B.

60. Since any integer multiplied with an even integer results in an even integer, we can say
that q p (ab) will be even under any of the following circumstances:

(1) Only q is even


(2) Only p is even
(3) Both p and q are even
It should be noted that the values of a and b are not important to decide if q p (ab)
is even or odd since a > b i.e. (a b) is positive, and an even number raised to any
positive integer exponent (be it even or odd) is always even and an odd number raised
to any positive integer exponent (be it even or odd) is always odd.

Statement 1:
q q
Since p = + 4, p and q are positive integers, + 4 must be an integer.
7 7
=> q must be a multiple of 7.
7
Say, q = 7 => p = + 4 = 5.
7
14
Again, say q = 14 => p = + 4 = 6.
7
Thus, if q is an odd multiple of 7, both p and q are odd, hence q p ab is a product of
two odd integers i.e. odd.

Similarly, if q is an even multiple of 7, both p and q are even, hence q p ab is a


product of two even integers i.e. even.
Thus, the answer can be both Yes as well as No.
Hence, statement 1 is not sufficient.

Hence, the answer would be among B/C/E.

www.manhattanreview.com 19992016 Manhattan Review


Number Properties Guide Solutions 175

Statement 2:
Statement 2 does not provide any information about p and q.
Hence, statement 2 is not sufficient.

Hence, the answer would be between C/E.

Statements 1 and 2 together:


From statement 1, we can say either both p and q are simultaneously even or they are
simultaneously odd.
From statement 2, we only have information that a b is even which is inconsequential
to answering the question since the exponent does not affect whether q p ab is even.

Hence, combining statements 1 and 2 is insufficient to answer the question.

The correct answer is option E.

61. We need to verify whether 15 is the arithmetic mean of p, q, r and 15.


p + q + r + 15
=> 15 =
4
=> 60 = p + q + r + 15
=> p + q + r = 45.

Statement 1:
The first statement mentions: p + q + r = 45, which is the same thing we want according
to the above reasoning.

Thus, statement 1 is sufficient to answer the question and the answer to the question is
Yes.

The correct answer would be between A/D.

Statement 2:
p q = q r => p + r = 2q . . . (i)

Again: p q = 5 => p = q + 5 . . . (ii)


Combining (i) and (ii), we have: q + 5 + r = 2q => q = r + 5 . . . (iii)

Thus, from (ii) and (iii), we have: p = (r + 5) + 5 = r + 10 . . . (iv)

Thus, p + q + r = (r + 10) + (r + 5) + r = 3r + 15.

19992016 Manhattan Review www.manhattanreview.com


176 Number Properties Guide Solutions

However, since the value of r is unknown, we cannot conclude whether p + q + r = 45


as required.

Thus, statement 2 is not sufficient to answer the question.

The correct answer is option A.

www.manhattanreview.com 19992016 Manhattan Review


Number Properties Guide Solutions 177

5.2.3 Divisibility

62. Since abc 6= 0, it means none of a, b, and c is 0.

Statement 1:
Since cde = 0, it means that either d or e or both are 0, because c cannot be 0. We are
not sure whether d = 0.
So statement 1 alone is not sufficient.

Statement 2:
Again, since ebc = 0, it means that e is 0, because b and c cannot be 0; however this
statement does not provide any information about d.
So statement 2 alone is not sufficient.

Statement 1 & 2 together:


Even combining both the statements cannot help as d can still be 0 or not.

The correct answer is option E.

63. If X, divided by 7 has a remainder 2, it implies that (X + 2) is completely divisible by 7.

Statement 1:
We can rewrite X + 5 as (X 2 + 7). Since X + 5 or (X 2 + 7) is completely divisible
by 7 (remainder 0), it means that X 2 will also be completely divisible by 7. So in other
words, X, when divided by 7 will leave 2 as remainder. Answer is Yes.
So statement 1 alone is sufficient.

Statement 2:
We can rewrite X 2 as (X 7 + 5). Since X 2 or (X + 5 7) is completely divisible
by 7 (remainder 0), it means that X + 5 will also be completely divisible by 7. So in other
words, X, when divided by 7 will leave 7 5 = 2 as remainder. Answer is Yes.
So statement 2 alone is sufficient.

The correct answer is option B.

19992016 Manhattan Review www.manhattanreview.com


178 Number Properties Guide Solutions

Alternate approach:

We can assume any value for X. Say from statement 1, X + 5 = 49, thus, we know that
X = 44, which when divided by 7 leaves a remainder of 2. The question statement 1
holds true.
Similarly, Say from statement 2, X 2 = 49, thus, we know that X = 51, which when
divided by 7 leaves a remainder of 2. The question statement 2 holds true.

64. A fraction is a terminating decimal if, in its reduced form, its denominator can be
expressed entirely as non-negative integral exponents of 2 and 5.

Statement 1:
We know that b = 110 which cannot be expressed entirely as non-negative integral expo-
nents of 2 and 5.
However, since the fraction is not in its reduced form, it may so happen that 11 in the
denominator may cancel with the numerator and the fraction would become terminating.

Hence, statement 1 is not sufficient.

Statement 2:
We only know that a is a multiple of 44.
However, only information regarding the numerator is not sufficient to say whether the
fraction would be terminating or not.

Hence, statement 2 is not sufficient.

Statements 1 and 2 together:


From statement 1, we know that b = 110.
From statement 2, we know that a is a multiple of 44 i.e. a = 44k where k is a positive
integer.

a2 (44k)2 (4 11 k)2 16 121 k2 16 11 k2


Hence, = = = = .
b 110 110 110 10

We can see that the denominator is 10 = 21 51 i.e. can be expressed as non-negative


integral exponents of 2 and 5.
a2
Hence, is a terminating decimal.
b

Hence, we get the answer by combining statements 1 and 2.

The correct answer is option C.

www.manhattanreview.com 19992016 Manhattan Review


Number Properties Guide Solutions 179

2 a3 a = 2a a2 1 = 2a (a 1) (a + 1) = 2 (a 1) a(a + 1).
 
65.

Thus, 2(a3 a) is twice the product of three consecutive positive integers (a 1) , a


and (a + 1).
Note that the product of three consecutive positive integers is always divisible by 2 and
3.
Since the above is twice the product of three consecutive positive integers, it is divisible
by 2 2 3 = 12.

Statement 1:
Even multiples of 3 less than 15 are 3 2 = 6 or 3 4 = 12.
Thus, we have: b = 6 or 12.
Since 2(a3 a) is divisible by 12, it is also divisible by 6.
Thus, 2 a3 a is divisible by b.


Thus, statement 1 is sufficient to answer the question.

The correct answer would be between A/D.

Statement 2:
Since b is a multiple of 4, we have b = 4, 8, 12, 16, etc.
We know that for b = 4 & 12, 2(a3 a) is divisible by b.
However, for b = 8 or 16 etc, 2(a3 a) is not necessarily divisible by b.
Thus, statement 2 is not sufficient to answer the question.

The correct answer is option A.

66. Statement 1:
For a number to be divisible by 4, the number formed by the last two digits of the given
number must be divisible by 4 as well.
Thus, the number formed by the last two digits of n i.e. 1b must be divisible by 4.
It is obvious that b = 2 or 6 (since 12 and 16 are divisible by 4).
However, we cannot comment anything about the value of a and hence, we cannot deter-
mine whether the value of (a b) is positive.
Hence, statement 1 is not sufficient.

Hence, the answer would be among B/C/E.

19992016 Manhattan Review www.manhattanreview.com


180 Number Properties Guide Solutions

Statement 2:
For a number to be divisible by 9, the number formed by the sum of the digits of the
given number must be divisible by 9 as well.
Thus, the sum of the digits of n i.e. 3 + a + 1 + b = (4 + a + b) should be divisible by 9.
Thus, we have: 4 + a + b = 9 or 18 or 27 . . .
=> a + b = 5 or 14 or 23 . . .

However, the maximum possible value of a + b can only be 9 + 9 = 18.


Hence, possible values of a + b = 5 or 14.
However, we do not know the individual values of a and b and hence, we cannot deter-
mine whether the value of (a b) is positive.
Hence, statement 2 is not sufficient.

Hence, the answer would be between C/E.

Statements 1 and 2 together:


From statement 1, we have b = 2 or 6.
From statement 2, we have a + b = 5 or 14.

We can have the following cases:

(1) b = 2, a + b = 5 => a = 3 => a b = 1 => (a b) is positive.


(2) b = 2, a + b = 14 => a = 12, which is not possible since a is a single digit.
(3) b = 6, a + b = 5 => a = 1, which is not possible since a is positive.
(4) b = 6, a + b = 14 => a = 8 => a b = 2 => (a b) is positive.

The number 3a1b would be either 3312 or 3816.

Thus, we see that there are two possible cases and in either case, the value of (a b)
is positive, i.e. the answer is Yes, a unique answer. Though we did not get the unique
value of the number, we got the unique nature of (a b), so in DS, it is important to
understand carefully what the scope of the question and data is.

Hence, we get the answer by combining statements 1 and 2.

The correct answer is option C.

www.manhattanreview.com 19992016 Manhattan Review


Number Properties Guide Solutions 181

67. When a positive number is divided by 4, the remainder can be any of 0, 1, 2 or 3.

Statement 1:
An odd number when divided by 4, can leave remainder either 1 or 3.

Thus, the square of an odd number when is divided by 4, the remainder would be either
12 = 1 or 32 = 9.

However, since 9 is greater than 4, it cannot be the remainder.

To get the actual remainder, we need to divide 9 by 4 again which leaves a remainder 1.

Thus, we see that the square of an odd number always leaves a remainder 1 when
divided by 4.

Thus, statement 1 is sufficient to answer the question and the answer to the question is
1.

The correct answer would be between A/D.

Statement 2:
We have: p is a multiple of 5.

If p is an even multiple of 5, the remainder when it is divided by 4 would also be even


i.e. 0 or 2.

However, if p is an odd multiple of 5, the remainder when it is divided by 4 would also


be odd i.e. 1 or 3.

Thus, statement 2 is not sufficient to answer the question.

The correct answer is option A.

68. A number is divisible by 3 if the sum of its digits is divisible by 3.

Statement 1:
Since BB is divisible by 3, the possible values of BB can be 33, 66 or 99.

However, since we do not anything about A, hence, we cannot answer the question.

Thus, statement 1 is not sufficient to answer the question.

19992016 Manhattan Review www.manhattanreview.com


182 Number Properties Guide Solutions

The correct answer would be among B/C/E.

Statement 2:
Since A0B is a multiple of 101, it is only possible if A = B.

Thus, the three digit number ABB can be also written as BBB.

The sum of the digits of the above number = B + B + B = 3B.

Thus, we can see that the sum of digits is divisible by 3 and hence, the number is
divisible by 3.

Thus, statement 2 is sufficient to answer the question and the answer to the question is
Yes.

The correct answer is option B.

69. Statement 1:
Since p is divisible by 3, the possible values of p are 3, 6, 9, 12 . . .

p
If p = 9, then = 3 which is odd.
3
p
However, if p = 12, then = 4 which is even.
3

Thus, statement 1 is not sufficient to answer the question.

The correct answer would be among B/C/E.

Statement 2:
Since p 2 , a perfect square, is divisible by 9, we can conclude that p must be divisible by 3.

Thus, this is the same information as contained in the first statement.

Thus, statement 2 is not sufficient to answer the question.

The correct answer would be between C/E.

www.manhattanreview.com 19992016 Manhattan Review


Number Properties Guide Solutions 183

Statements 1 and 2 together:


Even after combining the two statements, we cannot answer the question since each
statement provides us with the same information.

Thus, statements 1 and 2 together are not sufficient to answer the question.

The correct answer is option E.

19992016 Manhattan Review www.manhattanreview.com


184 Number Properties Guide Solutions

5.2.4 Exponents

h i1
If (m n)9 is a perfect cube then (m n)9 = (m n)3 must be an integer.
3
70.

Statement 1:
(m n)12 is perfect cube
h i1
It implies that (m n)12 = (m n)4 is an integer, but cannot deduce that (m n)3
3

must be an integer. It may or may not be an integer.


So statement 1 alone is not sufficient.

Statement 2:
(m n)3 is perfect cube
h i3
Since (m n)3 is a perfect cube, it implies that (m n)3 = (m n)9 must also be
a perfect cube.
So statement 2 alone is sufficient.

The correct answer is option B.

71. To get the minimum value of (x + y), we must find out least possible values of x and
y.

Statement 1:
7x is perfect square
Statement 1 alone is not sufficient as we do not know anything about y.

Statement 2:
147xy is perfect cube.

We factorize 147xy, it would be 72 .3.x.y .To make 72 .3.x.y a perfect cube, xy must
be at least 7. 32 = 63, so that we have three 7s and three 3s.

xy = 63 can be factorized as (1 63), (3 21), & (7 9). Out of three possibilities,


pair of 7 9 will yield x + y = 7 + 9 = 16 (minimum).

So statement 2 alone is sufficient.

The correct answer is option B.

www.manhattanreview.com 19992016 Manhattan Review


Number Properties Guide Solutions 185

72. Statement 1:
Since 3p is an integer, we can have two possible cases:

(1) p is an integer
An Integer
 
(2) p is of the form of , which may not an integer
3
Thus, we cannot conclude that p is an integer.
Hence, statement 1 is not sufficient.

Hence, the answer would be among B/C/E.

Statement 2:
Since p 3 is an integer, we can have two possible cases:

(1) p is an integer
3
p
(2) p is of the form of (An integer), which may not an integer

Thus, we cannot conclude that p is an integer.


Hence, statement 2 is not sufficient.

Hence, the answer would be between C/E.

Statements 1 and 2 together:


From the above arguments, we see that if both statements are to hold true, p must be an
integer (since that argument is common to both statements 1 and 2).
Thus, we can conclude that p is an integer.

Hence, we get the answer by combining statements 1 and 2.

The correct answer is option C.

a3
73. If is to be positive, we must have a as positive as well.
27
 3
a3 a
Again, = .
27 3
a3 a
Thus, if is to be an integer, must be an integer as well.
27 3

Statement 1:
a2 a 2
 
= .
9 3
a2 a
Since is an integer, we can say that is also an integer.
9 3

19992016 Manhattan Review www.manhattanreview.com


186 Number Properties Guide Solutions

a
However, may be either positive or negative since the square of either a positive or
3
negative number is always positive.
a
Hence, though we can conclude that is an integer, we cannot conclude whether it is
3
positive.
Hence, statement 1 is not sufficient.

Hence, the answer would be among B/C/E.

Statement 2:
a
Since is a positive integer, we can say that a = 9k, where k is a positive integer.
9
a3 (9k)3 729k3
Thus, = = = 27k3 .
27 27 27
It is obvious that 27k3 is a positive integer.

Hence, statement 2 is sufficient.

The correct answer is option B.

74. Statement 1:
(2 3x 2 ) (10 x) + (x 10)
Looking at , we find that (x10) is common term of the nu-
(x 10)
merator, and the denominator. If we rewrite [(23x 2 ) (10 x)] as [(2+3x 2 ) (x 10)],
the expression becomes

(2 + 3x 2 ) (x 10) + (x 10)
(x 10)

(x 10) (2 + 3x 2 + 1)
=>
(x 10)

=> 3x 2 1; after canceling (x 10)

Since given is x 2 = 144, so 3x 2 1 = 3.1441 = An unique value. We need not calculate


the exact value in most of the DS problems.
Remember that we need not be concerned about the value of x; whether x is 12 or 12.
To get the value of 3x 2 1, information about x 2 is enough.
Statement 1 alone is sufficient.

The correct answer would be between A/D.

www.manhattanreview.com 19992016 Manhattan Review


Number Properties Guide Solutions 187

Statement 2:
We cannot get value of expression from x < 10 information; there would be infinite
number of value for x.

So the correct answer is option A.

The information x < 10 was a trap laid out for you, so that you jump onto this infor-
mation to rule out x = 12, derived from previous statement.

75. If x 3 = 1, we can conclude that x = 1.

Let us see how.


We can plug in some values of x to check which values of x satisfy the given inequality.
While plugging in, we choose the values of x from the following four regions:

1 0 1
Region I: x = 2 : 23 = 1 => Region I satisfies the inequality.

1 1 3 1
Region II: x = : ( ) = + = 1 => Region II satisfies the inequality.
2 2 8
3
1 1 1

Region III: x = : = = 1 => Region III satisfies the inequality.
2 2 8

Region IV: x = 2 : (2)3 = 8  1 => Region IV does not satisfy the inequality.

Hence, the solution lies in Region I, II and III i.e. x = 1.

Statement 1:
x 2 + 5x + 4 = 0 => (x + 4) (x + 1) = 0 => x = 4 or 1.
If x = 4 => x 3 = (4)3 = 64  1.
3
However, if x = 1 => x 3 = (1) = 1 = 1.
Hence, from statement 1, we cannot definitely conclude whether x 3 = 1 as the answer
may be both Yes as well as No.
Hence, statement 1 is not sufficient.

Hence, the answer would be among B/C/E.

Statement 2:

19992016 Manhattan Review www.manhattanreview.com


188 Number Properties Guide Solutions

x 3 + 3x 2 = (1 + 3x) => x 3 + 3x 2 + 3x + 1 = 0
=> (x + 1)3 = 0 => x + 1 = 0
=> x = 1.
If x = 1 => x 3 = (1)3 = 1.
Hence, from statement 2, we can definitely conclude that x 3 = 1 and the answer is Yes.

Hence, statement 2 is sufficient.

The correct answer is option B.

76. Statement 1:
p 3 q3 = 0
=> p q p 2 + pq + q2 = 0
 

=> p q = 0 or p 2 + pq + q2 = 0
Thus, we can conclude: p = q (since the roots from the other equation: p 2 + pq + q2 = 0
are imaginary).
Alternately, we can say: p 3 q3 = 0 => p 3 = q3 .
Taking cube roots on both sides: p = q.
Thus, the answer to the question is Yes, a unique answer.
Hence, statement 1 is sufficient to answer the question.

Hence, the answer would be between A/D.

Statement 2:
p 2 + q2 = 0 :
We know that the square of any number is always non-negative.
Thus: p 2 0 and q2 0.
Since p 2 + q2 = 0 => p = q = 0.
Thus, the answer to the question is Yes, a unique answer.

Hence, statement 2 is sufficient to answer the question.

The correct answer is option D.

77. Statement 1:
We know that x 3 is a perfect cube as it is given in the question narration that x is a
positive integer. However, it is also a perfect square.
This is possible if x is itself a perfect square like 1, 4, 9, etc.

www.manhattanreview.com 19992016 Manhattan Review


Number Properties Guide Solutions 189

For example, x = 1 => x 3 = 13 = 1: a perfect square


x = 4 => x 3 = 43 = 64 = 82 : a perfect square
x = 9 => x 3 = 93 = 729 = 272 : a perfect square
Thus, there are many possible values of x.
Hence, statement 1 is not sufficient to answer the question.

Hence, the answer would be among B/C/E.

Statement 2:
x 2 12x + 32 = 0
=> x 2 4x 8x + 32 = 0
=> (x 4) (x 8) = 0
=> x = 4 or 8.
Thus, there are two possible values of x.
Hence, statement 2 is not sufficient to answer the question.

Hence, the answer would be between C/E.

Statements 1 and 2 together:


Combining both statements, we see that x = 44 (since 4 is a solution from both
statements), a unique answer.

Hence, we get the answer by combining statements 1 and 2.

The correct answer is option C.

78. p 2 q2 + 10q = 25
=> p 2 q2 + 10q 25 = 0
=> p 2 q2 10q + 25 = 0

2
=> p 2 q 5 = 0
  
=> p (q 5) p + q 5 = 0
 
=> p q + 5 p + q 5 = 0
=> p q + 5 = 0 OR p + q 5 = 0
=> p q = 5 OR p + q = 5.

19992016 Manhattan Review www.manhattanreview.com


190 Number Properties Guide Solutions

Statement 1:
We have seen that p 2 q2 + 10q = 25 is true when either p q = 5 or p + q = 5.
Since statement 1 satisfies on the conditions i.e. p + q = 5, we can definitely conclude
that p 2 q2 + 10q = 25 is true and thus, the answer is Yes.
Hence, statement 1 is sufficient.

Hence, the answer would be between A/D.

Statement 2:
Statement 2 does not give us any information about q.
Hence, statement 2 is not sufficient.
(Note: A careless mistake would be to jump to a conclusion that combining both state-
ment, one would get the values of both a and b and hence the answer would be option C.)

The correct answer is option A.

Alternatively,
2
p2 q 5 = 0
2
=> p 2 = q 5

=> p = q 5
=> p = q 5 OR p = (q 5)
=> p q = 5 OR p + q = 5.

www.manhattanreview.com 19992016 Manhattan Review


Number Properties Guide Solutions 191

5.2.5 Factors/Multiplicands/LCM/HCF

79. A prime number has exactly two factors, namely, 1 and the number itself.
Any number which has more than two factors cannot be prime.

Statement 1:
35
Since is an integer, s must be a number one among the factors of 35 i.e. 1, 5, 7 or 35.
s
Here, 5 and 7 are prime numbers having exactly two factors.
1 has only one factor i.e. 1 itself.
35 has more than two factors.
Thus, we cannot conclude whether s has more than two distinct factors.
Hence, statement 1 is insufficient.

Hence, the correct answer would be among B/C/E.

Statement 2:
21
Since is an integer, s must be a number one among the factors of 21 i.e. 1, 3, 7 or 21.
s
Here, 3 and 7 are prime numbers having exactly two factors.
1 has only one factor i.e. 1 itself.
21 has more than two factors.
Thus, we cannot conclude whether s has more than two distinct factors.
Hence, statement 2 is insufficient.

Hence, the correct answer would be between C/E.

Statements 1 and 2 together:


From the above arguments, we can conclude that s must be 1 or 7 (since 1 and 7 are
common to both statements 1 and 2).
However, 1 has only one factor and 7 has exactly two factors.
Thus, neither 1 nor 7 has more than two factors.
Hence, we can conclude that s does not have more than two factors and the answer is No.

Hence, we get the answer by combining statements 1 and 2.

The correct answer is option C.

80. If 15 is a factor of a, then a must be divisible by 15.

19992016 Manhattan Review www.manhattanreview.com


192 Number Properties Guide Solutions

Statement 1:
Since a is divisible by both 5 and 10, it is divisible by the LCM of 5 and 10 i.e. 10.
Thus, possible values of a are 10, 20, 30, . . .
Thus, we can see that if a = 10 or 20, it is not divisible by 15; whereas if a = 30, it is
divisible by 15.
Thus, the answer to the question may be both No as well as Yes.
Hence, statement 1 is not sufficient.

Hence, the answer would be among B/C/E.

Statement 2:
Since a is divisible by 3, possible values of a are 3, 6, 9, 12, 15, . . .
Thus, we can see that if a =3 or 6 or 9 or 12, it is not divisible by 15; whereas if a = 15,
it is divisible by 15.
Thus, the answer to the question may be both No as well as Yes.
Hence, statement 2 is not sufficient.

Hence, the answer would be between C/E.

Statements 1 and 2 together:


From statement 1, we can say that a is divisible by 10 and from statement 2, we can say
that a is divisible by 3.
Hence, we can say that a is divisible by the LCM of 10 and 3 i.e. 30.
Thus, possible values of a are: 30, 60, 90 . . .
Thus, we can see that a is definitely divisible by 15 and the answer is Yes.

Hence, we get the answer by combining statements 1 and 2.

The correct answer is option C.

81. Statement 1:
Since n is any element of the set, we can have a situation where n = 7.
In that case, n + 7 = 7 + 7 = 14 is an element in the set.

Again, we may have a situation where n = 14.


In that case, n + 7 = 14 + 7 = 21 is an element in the set.

Continuing in the above manner, we get 7, 14, 21, 28 . . . (i.e. all multiples of 7) as the
elements of the set.

www.manhattanreview.com 19992016 Manhattan Review


Number Properties Guide Solutions 193

Thus, statement 1 is sufficient to answer the question and the answer is Yes.

The correct answer would be between A/D.

Statement 2:
Since n is any element of the set, we can have a situation where n = 7.
In that case, n 7 = 7 7 = 0 is an element in the set, (which is a multiple of 7).

Again, we may have a situation where n = 0.


In that case, n 7 = 0 7 = 7 is an element in the set.

Continuing in the above manner, we get 7, 0, 7, 14, 21 . . . (i.e. all multiples of 7) as


the elements of the set.

Thus, statement 2 is sufficient to answer the question and the answer is Yes.

The correct answer is option D.

19992016 Manhattan Review www.manhattanreview.com


194 Number Properties Guide Solutions

5.2.6 Inequality

82. Analysis of the values of a , a 2 and a 3


If a = 0 or 1, a = a2 = a3
If a > 1, a < a2 < a3 . Say a = 2. Then 2 < 4 < 4.
If a > 1, a3 < a < a2 . Say a = 2. Then 8 < 2 < 4.
If 1 > a > 0, a > a2 > a3 . Say a = 1/2. Then 1/2 < 1/4 < 1/8.
If 0 > a > 1, a2 < a3 < a. Say a = 1/2. Then 1/4 < 1/8 < 1/2.

We have to find out whether x < x 2 < x 3 .


If x < x 2 < x 3 fits into case II cited above, the question is true, else false. Let us
analyze each statement one by one.

Statement 1:
Given that, x < x 3
Looking at the analysis of values, given above, we find that x < x 3 may be true in two
conditions:

(1) If 0 > x > 1, x 2 > x 3 > x . Say x = 1/2. Then 1/4 > 1/8 > 1/2.
(2) If x > 1, x < x 2 < x 3 . Say x = 2. Then 2 < 4 < 8.

In the first condition, x 2 > x 3 , but in the other condition x 3 > x 2 .

Statement 1 alone is not sufficient.

Statement 2:
Given that, x 3 > x 2
Looking at the analysis of values, given above, we find that x 3 > x 2 can be true in ONLY
one condition, when x > 1, x < x 2 < x 3 . Say x = 2. Then 2 < 4 < 8.

Statement 2 alone is sufficient.

The correct answer is option B.

83. Given that a |a| < |a|a + 1

Let us reduce the inequality to a convenient form; dividing both the sides by |a|.

=> a < |a|a ; since |a| is a positive quantity, we can divide both the sides of the inequality
by |a| without changing the sign of the inequality.

www.manhattanreview.com 19992016 Manhattan Review


Number Properties Guide Solutions 195

Note, we cannot divide both the sides by a , as we are not sure of its sign, if a being a
negative number, the sign of inequality would reverse.

a|a convey?
What does a < |a

This inequality is such that if a is negative, left hand side is negative, whereas, right
hand side always remains positive, because |a| is independent of the sign of its index a.

Note that the sign of the exponent does not affect the sign of a number.

So for all negative values of a, the inequality holds true.

What if a is positive?

Well, it will depend on what positive value a assumes. If a = 1, the inequality becomes
equality or becomes false, but when a > 1, it is quite obvious that inequality holds true.

What if a is 0?

We need not discuss this as it is given in the question stem that a is a non-zero number.

Statement 1:
a<1
=> a is either a negative number or a positive number lying between 0 & 1; remember
that a 6= 0.
=> If a is negative: We already concluded that inequality holds true. So the answer is
YES.

If 0 < a < 1

Lets test this. Say we take a convenient value for a = 1 2.

 1
1 1 2
=> So a < |a|a => < . It might be difficult for some of you to infer the result.
2 2
We can simplify this by squaring both sides.

1 1
So, we get < . So answer is YES.
4 2

=> Should you wish to try one more value of a, you may. Say we try a = 0.10.

a < |a|a => 0.1 < (0.1)0.1

19992016 Manhattan Review www.manhattanreview.com


196 Number Properties Guide Solutions

Again, it might be difficult for some of you to infer the result. We can simplify this by
raising both sides by the power of 10. So this becomes (0.1)10 < 0.1 or (1/10)10 < 1/10.
 10
1
Since the base of is less than 1, with its index being more than 1, the value of left
10
1
hand side isgoing to beless than . So answer is YES.
10

Statement 1 alone is sufficient.

Statement 2:
a>0

=> a is positive

We already concluded that the inequality is true for all positive values of a except @
a = 1. So statement 2 has duality in nature, so the statement 2 alone is not sufficient.

The correct answer is option A.

84. Let us look at the statements one by one.

Statement 1:
n = m2
m m 1
If n = m2 , = 2 = .
n m m
Since we do not know anything about m, we cannot comment on it. For m = 1 or for
1 1
m = 1, = 1 or m = 1, integers, but for m = 0 or any other value is not an
m m
integer.

Hence, the statement 1 alone is not sufficient.

Statement 2:
|m| > 1
We do not know any information about n, so statement 2 alone is not sufficient.

Statement 1 and 2 together:


m
From statement 1, we know that for m = 1 or m = 1, is an integer. From statement
n
2, |m| > 1, we know that m < 1 or m > 1. This inequality rule out m = 0, an
m
indeterminable value for . For each value falling in the range of m < 1 or m > 1,the
n
m 1
expression in question = is not an integer.
n m

The correct answer is option C.

www.manhattanreview.com 19992016 Manhattan Review


Number Properties Guide Solutions 197

85. We rearrange inequality.


13 5x
5x + 3y 13 => y
3
To make sure that y is more than 1, we must plug in the maximum possible value of x
in the rearranged inequality so that we get minimum possible value of y. Reason for this
is that in the numerator 13 5x, x is negative, and the maximum possible value of x
will reduce the value of 13 5x, and in turn y would be minimum.
Let us look at the statements one by one.

Statement 1:
x 2
13 5 2
At xmax = 2, ymin = = 1
3
=> y m in = 1, or y 1,

So statement 1 alone is sufficient.

Statement 2:
x > 2
We know that
13 5x
y
3
We can only plug in the minimum possible value of x, because as per inequality x 2,
we can know the minimum value and not the maximum value of x. Therefore, we will be
able to get the maximum possible value of y. Reason for this is that in the numerator
13 5x, x is negative, minimum possible value of x will reduce the value of 13 5x
to the minimum, and in turn y would be maximum.
13 + 5 2 23
So at xmin = 2, ymax = =
3 3
23 23
=> ymax = , or y
3 3
We do not know whether y will indeed take a value equal to 1.

So statement 2 alone is not sufficient.

The correct answer is option A.

86. Let us look at the statements one by one.

Statement 1:
m m2 < 0
The inequality can be reduced to m < m2 . The inequality m < m2 holds true if m > 1
or m < 0.

19992016 Manhattan Review www.manhattanreview.com


198 Number Properties Guide Solutions

This implies that for 1 m 0, it does not hold true.

So statement 1 alone is not sufficient.

Statement 2:
m3 > 0
m3 would be greater than 0 only and only if m > 0,

So statement 2 alone is sufficient.

The correct answer is option B.


87. By definition, all even roots of any number n, like square root ( n), fourth root ( 4 n),
etc. are positive.

4
For example, 4 = 2 and not 2; similarly,81 = 3 and not 3; etc.
4
Thus, irrespective of the value of b, the value of b > 0.

Thus the question comes down to determine the nature of a.

Statement 1:

4
Since: b = a => b = 8 a.

4 9
Thus, a. b = a. 8 a = a 8 .
However, we do not know the value of a (except that a must be positive since b is the
9
square root of a) and hence, cannot comment whether a 8 > 1.

Hence, statement 1 is not sufficient.

Statement 2:
Since b2 > b => b2 b > 0 => b (b 1) > 0.

This implies:

(1) b > 0 and b 1 > 0 => b > 1

OR

(2) b < 0 and b 1 < 0 => b < 0

Following the analysis done before, we can conclude that b cannot be negative, thus
b > 1.

www.manhattanreview.com 19992016 Manhattan Review


Number Properties Guide Solutions 199

However,
we do not know the value of a and hence, cannot comment on the value of
4
a. b.

Hence, statement 2 is not sufficient.

Statements 1 and 2 together:



4
From statement 2, we know that: b > 1 => b > 1.

From statement 1, we know that: b = a => a = b2 .
Since b > 1 => a = b2 > 1.
4

4
Thus, we have a > 1 and b > 1 => a. b > 1.


4
Hence, combining both statements, we can say that a. b > 1.

The correct answer is option C.

Alternate approach:

Obviously, the answer cannot be obtained from individual statements.


From statement 1, we know that b is positive.
From statement 2, therefore, we assume a value of b which is positive.
If b = 0.5 => b2 = (0.5)2 = 0.25 < b. Hence, this value of b is inadmissible as b2 > b.
If b = 2 => b2 = 4 > b. Hence, we can assume this value of b.

Thus, from statement 1: b = a => a = b2 = 4.

4

4
Hence, a. b = 4 2 > 1.

Hence, we get the answer by combining statements 1 and 2.

88. We have: x < x 2 => x x 2 < 0 => x 2 x > 0 => x (x 1) > 0.

This is possible under two circumstances:


1. x > 0, and x 1 > 0 i.e. x > 1.
Thus, from both we have: x > 1 . . . (i)

2. x < 0, and x 1 < 0 i.e. x < 1.


Thus, from both we have: x < 0 . . . (ii)

Combining (i) and (ii) we have: x < 0 or x > 1.

Statement 1:
We know that the correct solution for x should be x < 0 or x > 1.

19992016 Manhattan Review www.manhattanreview.com


200 Number Properties Guide Solutions

Since this statement mentions x < 1, we can have values where 0 < x < 1 which would
not be a solution.
At the same time, x < 0 is also valid if x < 1 which would be a solution; thus we get Yes
as well as No for the statement.
Thus, statement 1 is not sufficient to answer the question.

The correct answer would be among B/C/E.

Statement 2:
Since x < 0 is a correct solution for the given inequality, statement 2 is sufficient to
answer the question.

Hence, we get the solution from statement 2 alone.

Alternate approach:

We can plug in some values of x to check which values of x satisfy the given inequality.
While plugging in, we choose the values of x from the following four regions:

1 0 1

Region I: x = 2 : 2 < 22 => Region I satisfies the inequality.


Region II: x = 0.5 : 0.5 > 0.52 => Region II does not satisfy the inequality.
Region III: x = 0.5 : 0.5 < (0.5)2 => Region III satisfies the inequality.
Region IV: x = 2 : 2 < (2)2 => Region IV satisfies the inequality.
Hence, the solution lies in Region I, III and IV i.e. x > 1 or x < 0.

The correct answer is option B.

89. Lets draw the domain of j k on a number line.

5 4 3 2 1 0 1 2 3 4 5

www.manhattanreview.com 19992016 Manhattan Review


Number Properties Guide Solutions 201

If the inequality is true then it is clear that j k is positive.

The product of two variables can be positive if both of them are either positive or
both of them are negative.

Let us look at the statements one by one.

Statement 1:
j < 1 and k < 1

Say j = 0 and k = 2, then jk = 0. Result is No.


Say j = 1/4 and k = 2, then jk = 1/2. Result is Yes.

Statement 1 alone is not sufficient.


While solving DS problems, you must think how to break questions equilibrium. If after
applying all possible attempts, you are not able to break it. That statement alone is
sufficient.

Statement 2:
j > 1 and k < 1

Say j = 0 and k = 2, then jk = 0. Result is No.


1 1
Say j = and k = 2, then jk = . Result is Yes.
4 2
Statement 2 alone is not sufficient.

Statement 1 and 2 together:


Since both the set of given values satisfy both the statements, while resulting into No
and YES answers. Statement 1 and 2 together are not sufficient.

Say j = 0 and k = 2, then jk = 0. Result is No.


1 1
Say j = and k = 2, then jk = . Result is Yes.
4 2

The correct answer is option E.

90. It is obvious that statement 1 and 2 alone are not sufficient, because each presents
an equality with two variable and we cannot conclusively deduce anything about y;
however combining both the statements may work.

Statement 1 and 2 together:

Approach 1:

Given,

19992016 Manhattan Review www.manhattanreview.com


202 Number Properties Guide Solutions

x + y < 11, and x y > 8.


The inequality of statement 2 can be reduced to x + y < 8; notice the sign change
of inequality.
Now we can add both the inequalities.
x + y < 11 (1)
x + y < 8 (2),
3
We get, 2y < 3 => y <
2
=> y could be 0 or 1 (as y is non-negative integer).

If y = 0,

From statement 1 inequality, we get x < 11 and


From statement 2 inequality, we get x > 8
x is either 9 or 10

But for y = 1,

From statement 1 inequality: x + y < 11, we get x < 10 and


From statement 2 inequality: x y > 8, we get x > 9.

This is not possible, as it vanishes x altogether. So only possible value left is y = 0.


Thus the answer to the question: Is y = 1? is No.

The correct answer is option C.

Approach 2:

Another approach could be plug in value approach.


We must try the inequalities for y = 0, 1, 2, . . . .

At y = 0, statement 1: x + y < 11 gives x < 11 and statement 2: x y > 8


gives x > 8. This means that x is either 9 or 10.
At y = 1, statement 1: x + y < 11 gives x < 10 and statement 2: x y > 8
gives x > 9. This is not possible, it vanishes x altogether.
At y = 2, statement 1: x + y < 11 gives x < 9 and statement 2: x y > 8
gives x > 10. This is inconsistent.

This gives only one possible value of y i.e. 0. Or the answer to the question: Is y = 1?
is No.

The correct answer is option C.

91. From |xy| > 1, we can conclude two things.

(1) The product of xy is greater than 1.

www.manhattanreview.com 19992016 Manhattan Review


Number Properties Guide Solutions 203

(2) x as well as y can have any sign +/ .

Statement 1:
If |x | + |y| > 1, minimum value of |x | + |y| would be 2, and this follows that the
minimum values of x & y would be 1 each. Thus the minimum value for |xy| would
1. So for this set of values |xy| 1. Result is No. |x | + |y| > 1 will also yield very
large values for x & y, which will make |xy|>>1. Result is Yes.

So statement 1 alone is not sufficient.

Statement 2:
xy > 1 implies that xy is positive and greater than 1, this follows that |xy| > 1.

So statement 2 alone is sufficient.

The correct answer is option B.

92. For a7 b4 c 6 < 0


First, none of a, b, and c should be 0.
Second, the sign of only a will determine whether a7 b4 c 6 < 0, we need not infer anything
about the sign of b and c, because if a number is indexed (power) with an odd number,
the sign of number will depend on the sign of its base; unlike this, if a number is indexed
(power) with an even number, the sign of number will not depend on the sign of its base,
the number will always be positive. So if a7 b4 c 6 is negative, it is because a is negative.

19992016 Manhattan Review www.manhattanreview.com


204 Number Properties Guide Solutions

Statement 1:
a|b|2 < 0
This inequality means that none of a and b is 0. Further, b2 will always be positive
whether it is positive or negative. It means that a is negative.
Thus, we can conclude that a7 b4 c 6 < 0. Answer is Yes.
So statement 1 alone is sufficient.

The correct answer would be between A/D.

Statement 2:
ac 2 < 0
Like statement 1, this inequality means that none of a and c is 0 and c 2 will always be
positive whether it is positive or negative. It means that a is negative.
Thus, we can conclude that a7 b4 c 6 < 0. Answer is Yes.

So statement 2 alone is sufficient.

The correct answer is option C.

93. It is obvious that statement 1 and 2 alone are not sufficient, because each presents an
equality with two variable and we cannot conclusively deduce anything about x & y;
however combining both the statements may work.

Statement 1 and 2 together:

Approach 1:

We want the minimum possible value of (x y).

The minimum possible value of (x y) can be achieved when x is smallest and y is


largest.

Given, x > 11, the smallest possible value of x would be 12.

By plugging in x = 12 in statement 1 inequality: x + y < 23, we get y < 11, so


maximum possible value of y would be 10.

So, minimum possible value of (x y) = (12 10) = 2.

The correct answer is option C.

www.manhattanreview.com 19992016 Manhattan Review


Number Properties Guide Solutions 205

94. Lets draw the domain of m n on a number line.

mn

-5 -4 -3 -2 -1 0 1 2 3 4 5

If the inequality is true, then it is clear that m n is a positive value.

The product of two variables can be positive if both of them are either positive or
both of them are negative.
Let us look at the statements one by one.

Statement 1:
|m| > 1 and |n| < 1
Say m = 10 and n = 0, then mn = 0. Result is No.
1 1
Say m = 2 and n = , then mn = . Result is Yes.
4 2

Statement 1 alone is not sufficient.

While solving DS problems, you must think how to break the question equilibrium. If
after applying all possible attempts, you are not able to break it. That statement is alone
is sufficient.

Statement 2:
|m| < 2 and |n| > 0
3
Say m = and n = 2, then mn = 3. Result is No.
2
3 1 3
Say m = and n = , then mn = . Result is Yes.
2 8 16

Statement 2 alone is not sufficient.

Statement 1 and 2 together:


1 < |m| < 2 and 1 > |n| > 0
3 3 9
Say m = and n = , then mn = . Result is No.
2 4 8
3 1 3
Say m = and n = , then mn = . Result is Yes.
2 8 16

19992016 Manhattan Review www.manhattanreview.com


206 Number Properties Guide Solutions

Statement 1 and 2 together are not sufficient.

The correct answer is option E.

95. Let us look at the statements one by one.

Statement 1:
0 < |a| < 1 and 0 < |b| < 1
Inequalities 0 < |a| < 1 and 0 < |b| < 1 mean that a and b both have their absolute
values greater than 0 and less than 1. But either a or b or both may be negative too.
If both are negative or both are positive, the product of a and b would be positive and
0 < ab < 1. Answer is Yes.
However if one between a and b is positive and other is negative, the product of a and
b would be negative and 0 > ab > 1. Answer is No.

Statement 1 alone is not sufficient.

Statement 2:
0 < |ab| < 1
With the same reasoning as cited in statement 1, we cannot conclude.

So the statement 2 alone is also not sufficient.

Statement 1 & 2 together:


Even after combining both the statements, we cannot conclude as both the statements
in fact are the same.

The correct answer is option E.

96. Statement 1:
Since p > q2 and q2 0 (any number raised to an even power can never be negative)
=> p > 0.
There are two possible cases: 0 < p < 1 or p > 1.

Case (i): 0 < p < 1


1
Say, p = .
4
1
Thus, q2 <
4
1
=> q < | |
2

www.manhattanreview.com 19992016 Manhattan Review


Number Properties Guide Solutions 207

1 1
=> <q<
2 2
1
=> Since p = lies in the range of q, thus a part p > q and for the other part p < q. No
4
conclusive answer.

Case (ii): p > 1


Say, p = 4.
Thus, q2 < 4
=> q < |2|
=> 2 < q < 2
=> p > q (whatever be the value of q we choose, we always find that p > q).
Thus, the answer may be both Yes as well as No considering both cases (i) and (ii).
Hence, statement 1 is not sufficient.

Hence, the answer would be among B/C/E.

Statement 2:
Statement 2 does not provide any information about the nature of q.
Hence, statement 2 is not sufficient.

Hence, the answer would be between C/E.

Statements 1 and 2 together:


Combining both statements, we get the same situation as case (ii) of statement 1.
From that situation, our conclusion is p > q and our answer is Yes.

Hence, we get the answer by combining statements 1 and 2.

The correct answer is option C.

97. Statement 1:
(x 4) (x + 3) > 0: There are two possibilities:
x 4 > 0 and x + 3 > 0 => x > 4 and x > 3 => x > 4 . . . (i)
OR
x 4 < 0 and x + 3 < 0 => x < 4 and x < 3 => x < 3 . . . (ii)
From (i) and (ii) we have: x < 3 or x > 4.

Thus, the answer to the question may be both Yes as well as No.

19992016 Manhattan Review www.manhattanreview.com


208 Number Properties Guide Solutions

Hence, statement 1 is not sufficient to answer the question.

Hence, the answer would be among B/C/E.

Statement 2:
x > 4 is alone not sufficient to answer the question since we can have a value of x for
example, 2 which is not more than 4 or a value for example, 6 which is more than 4.
Thus, the answer to the question may be both Yes as well as No.
Hence, statement 2 is not sufficient to answer the question.

Hence, the answer would be between C/E.

Statements 1 and 2 together:


From statement 1, we have: x < 3 or x > 4.
From statement 2, we have: x > 4.
Combining the above results, we get: 4 < x < 3 or x > 4.

Though from 4 < x < 3, we may jump into the conclusion that no integer falls
between 3 & 4, thus the only possibility is that x > 4, but this conclusion is wrong as
the question does not state that x is an integer. If nothing is mentioned in the question,
all the numbers are real.

There can be infinite real numbers lie in the range 4 < x < 3.
Thus, the answer to the question may be both Yes as well as No.

Hence, even after combining statements 1 and 2, it is not sufficient to answer the
question.

The correct answer is option E.

98. a2 = 120 + b2
=> a2 b2 = 120
=> (a + b) (a b) = 120.
Thus, (a + b) and (a b) are both factors of 120.

Since a and b are positive integers, we have: (a + b) > (a b).


Thus, we need to divide 120 in two factors whose product is 120 and one factor is
greater than the other.

Once we have the above two factors, say p and q (p > q), where p q = 120, we have:
a + b = p . . . (i)

www.manhattanreview.com 19992016 Manhattan Review


Number Properties Guide Solutions 209

a b = q . . . (ii)
p+q
Adding the above two equations, we get: 2a = p + q => a = .
2

Since a is an integer, (p + q) must be divisible by 2.


This is possible if p and q are both odd or both even.

However, since p q = 120, an even number, both of them cannot be odd.


Thus, we must have both p and q as even.
We can have the following cases:
p+q 12 + 10
(1) p = 2 2 3 = 12 and q = 2 5 = 10 => a = = = 11.
2 2
p+q 20 + 6
(2) p = 2 2 5 = 20 and q = 2 3 = 6 => a = = = 13.
2 2
p+q 30 + 4
(3) p = 2 3 5 = 30 and q = 2 2 = 4 => a = = = 17.
2 2
p+q 60 + 2
(4) p = 2 2 3 5 = 60 and q = 2 => a = = = 31.
2 2

Statement 1:
Statement 1 states that (a + b) > (a b).
However, it is obvious since a and b are given as positive integers as explained above.
The statement is redundant or is a no-statement as it does not provide any additional
information.
Hence, statement 1 is not sufficient.

Hence, the answer would be among B/C/E.

Statement 2:
We have seen that the possible values of a are 11, 13, 17 or 31.
Among these, only 17 has the sum of digits as 8.
Thus, the answer is a = 17.
Hence, statement 2 is sufficient.

The correct answer is option B.

99. Statement 1:
We have: x > 2q.

However, the value of q is unknown i.e. it can be positive or negative.

19992016 Manhattan Review www.manhattanreview.com


210 Number Properties Guide Solutions

Thus, we cannot determine whether x is negative.

Thus, statement 1 is not sufficient to answer the question.

The correct answer would be among B/C/E.

Statement 2:
We have: x < q.

Like in statement 1, the value of q is unknown i.e. it can be positive or negative.

Thus, we cannot determine whether x is negative.

Thus, statement 2 is not sufficient to answer the question.

The correct answer would be between C/E.

Statements 1 and 2 together:


Combining both statements, we have: q > x > 2q i.e. q > 2q => 0 > q.

Thus, we conclude that q is negative.

Since we know that x < q, we can conclude that x is negative as well.

Hence, we get the answer by combining statements 1 and 2.

The correct answer is option C.

100. Since p q > 0, we can have two cases:

(1) p > 0 and q > 0

OR

(1) p < 0 and q < 0

Statement 1:
We know: q > p 4 .

Since p 4 > 0 => q > 0.

Since q > 0 and p q > 0 => p > 0.

www.manhattanreview.com 19992016 Manhattan Review


Number Properties Guide Solutions 211

Thus, both p and q are positive.

Hence, p + q > 0.

Thus, statement 1 is sufficient to answer the question and the answer is Yes.

The correct answer would be between A/D.

Statement 2:
We know: q = 4p.

Thus, p q = p 4p = 4 p 2 > 0 (since p 2 > 0).




Since p 2 > 0 => p > 0.

Since p > 0 and p q > 0 => q > 0.

Thus, both p and q are positive.

Hence, p + q > 0.

Thus, statement 2 is sufficient to answer the question and the answer is Yes.

The correct answer is option D.

19992016 Manhattan Review www.manhattanreview.com


212 Number Properties Guide Solutions

5.3 Assorted Questions

5.3.1 Problem Solving

101. a2 + b2 + c 2 = ab + bc + ca
=> 2a2 + 2b2 + 2c 2 = 2ab + 2bc + 2ca
=> 2a2 + 2b2 + 2c 2 2ab 2bc 2ca = 0
=> a2 2ab + b2 + b2 2bc + c 2 + c 2 2ca + a2 = 0
  

=> (a b)2 + (b c)2 + (c a)2 = 0


The sum of three perfect squares is zero.

Since a perfect square can never be negative, the above sum is zero only when each term
is zero.
Thus: a b = 0, b c = 0, c a = 0 => a = b = c.

However, we know that a = 5.


Hence, a = b = c = 5

=> (a + b + c) = 15.

The correct answer is option D.

Cheeky Approach:

Though it is not an alternate approach, this particular question can be worked worked
out in much smarter way.

We know that a2 + b2 + c 2 = ab + bc + ca. Thus, we can write LHS as


a.a + b.b + c.c = a.b + b.c + c.a

Since there is no constraint, we can safely assume that a = b = c.


Thus, (a + b + c) = 15.

30 + 2n 30 2n 30
102. We can simplify = + = +2
n n n n
30 + 2n 30
 
Thus, is a positive integer, if is an integer.
n n

Thus, n is a factor of 30.


Thus, the possible values of n are: 1, 2, 3, 5, 6, 10, 15, and 30. Since there is no
constraint with the value of n, we can take it as a negative integer too; the question

www.manhattanreview.com 19992016 Manhattan Review


Number Properties Guide Solutions 213

30 30
does not state that n is also a positive integer. At n = 30, +2 = + 2 = +1
n 30
30
(acceptable value). We cannot take n = 15 as this will render +2 = 0, thus rejected.
15

Hence, n can have 1, 2, 3, 5, 6, 10, 15, 30, 30: Nine values.

The correct answer is option E.

q
103. Simplifying first part: (0.0004)3
r 3 p p
= 4 104 = 43 1012 = 64 1012 = 8 106 = 0.000008.
s 2
3 0.000375
Simplifying second part:
24
v !2 v
6 2
s
375 106 12
52 104
u 3 !
6
u
3
u u3 5 3 10 3 5 10
= t = t = = = 0.000625.
23 3 23 3 26 22
s
0.000375 2
q 
3 3
Thus, we have: (0.0004) + = 0.000008 + 0.000625 = 0.000633.
24

The correct answer is option B.

104. The remainder when n is divided by 2d is 11.

Thus, we have: n = 2dq + 11; where q is the quotient when n is divided by 2d.

If we divide n by d, the remainder is the same as when 11 is divided by d (since 2dq is a


multiple of d, it does not leave any reminder when divided by d).

Thus, when 11 is divided by d, the remainder is 5.

Hence, a possible value of d is 11 5 = 6 (since 11 when divided by 6 leaves a remainder


5).

Now, we need to check if any factors of 6 also give the same remainder.

However, factors of 6 (apart from 6) are 1, 2, and 3; none of these divisor can leave a
remainder of 5 (since the remainder must be less than the divisor).

Hence, d = 6.

The correct answer is option A.

19992016 Manhattan Review www.manhattanreview.com


214 Number Properties Guide Solutions

105. Let the required three digit number be n.

Thus, when n is divided by 2, 3, 4, 5, or 6 we have the same remainder of 1.

Hence, n when divided by the LCM of 2, 3, 4, 5, and 6 would also leave the same remain-
der 1.
Thus, n when divided by LCM, 60, leaves a remainder 1.

Hence, we can express n as 60m + 1; where m is an integer.

Since we need the smallest three digit value of n, we choose m = 2; which gives us:
n = 60 2 + 1 = 121.

We did not choose a smaller value of m = 1 as by choosing m = 1, would give the


number = 61, which is a two digit number!

The correct answer is option B.


106. Since we need to minimize the value of |x| y , we need to minimize |x| and
maximize |y|.

We know that the minimum value of the modulus of a number is 0.


Substituting x = 0 in x 2 2y 2 = 4, we get: y 2 = 2 which is not possible.

Thus, we need to choose a value of x such that |x| is minimum and y 2 is non-negative.

x 2 2y 2 = 4 => 2y 2 = x 2 4.
x2 4 x2
=> y 2 = = 2
2 2
x2 x2
=> 2 0 => 2
2 2
=> x 2 4 => |x| 2
The minimum value of |x| = 2 => y = 0


Hence, the minimum value of |x| y = 2 0 = 2.

The correct answer is option D.


x y x y
107. = y ; (since y > 0 => y = |y|)

y

x y x y x

= = = 1 .

y y y y

www.manhattanreview.com 19992016 Manhattan Review


Number Properties Guide Solutions 215

We know: 2 x 8 and 2 y 5

Since we need the maximum value, we can have two cases:

(1) x
is the least negative value and y is the least positive value: x = 2, y = 2 :
x
2



1 = 1 = |2| = 2.

y 2

(2) x is the most positive value and y is the least positive value: x = 8, y = 2:

x 8


1 = 1 = |3| = 3.

y 2

Hence, the maximum value is 3.

The correct answer is option C.

1 1 1
108. We need to compare 2 2 , 4 3 and 10 4 . Let us take the LCM of the denominators of the
indices (i.e. 2, 3 and 4; LCM =12) so that the comparison becomes easier. One can
compare exponents without calculating their value if either their bases are equal or
their indices are equal. Though we can make the bases of first two exponents equal, we
cannot do the same with the third exponent.

We have:
1 1

12
12
x = 2 2 = 26 26 =
 12
= 64
1 1
12 4
12
y = 4 3 = 44 12 =

4 = 256
1 1
12
p
z = 10 4 = 103 12 = 103 =
12

1000

Thus, we have: z > y > x; by comparing the bases.

The correct answer is option D.

109. Since we need the smallest integer value of (m + n), we must find the minimum values
of m and n.

Since 56 > 2m , we can chose m = 1 (since 56 > 21 ).

2 2
Again, 56 = 53 = 1252 < 1282 = 27 = 214
Thus, we have 214 > 56 => n = 14.

Hence, the minimum value of (m + n) = 1 + 14 = 15.

The correct answer is option C.

19992016 Manhattan Review www.manhattanreview.com


216 Number Properties Guide Solutions

110. The divisor must be greater than the remainder. Hence, r < 18.

n
Since is terminating (terminating decimals occur only if the divisor can be expressed
18
n
in the form of exponents of 2 and 5 only), we can say that can be reduced to a form
18
where the denominator is 2 (we need to cancel all the 3s of the denominator so that the
decimal becomes terminating).

n x
Thus, can be reduced to the form (canceling 9 from the numerator and the denom-
18 2
inator).
x
The non-zero remainder for can be only 1.
2
n
Thus, the remainder for is 1 9 = 9.
18

For example: if 27 is divided by 18, the remainder is 9. However, in the division of 27 by


18, we can cancel 9 from the numerator and denominator to make it the division of 3 by
2 which leaves a remainder 1.

Hence, there can be only one possible value of r .

The correct answer is option A.

30!
111. n = 21 22 23 24 . . . 30 = .
20!
The highest exponent of 3 in 30! can be calculated by adding the quotients (integer parts)
when 30 is successively divided by 3:
30
= 10
3
10
= 3.33 3
3
3
=1
3
Thus, the highest exponent of 3 in 30! is 10 + 3 + 1 = 14.

Similarly, the highest exponent of 3 in 20! is:

20
= 6.66 6
3
6
=2
3
Thus, the highest exponent of 3 in 20! is 6 + 2 = 8.

Thus, we have:

www.manhattanreview.com 19992016 Manhattan Review


Number Properties Guide Solutions 217

30! 314
8 = 3(148) = 36 .
20! 3

Thus, the highest exponent of 3 is 6.

The correct answer is option C.

112. The highest possible exponent of 2 in 8! is:

8
=4
2
4
=2
2
2
=1
2

So, we have: a = 4 + 2 + 1 = 7.

The highest possible exponent of 3 in 8! is:

8
= 2.67 2
3

So, we have: b = 2.

In 8! = 1 2 3 4 5 6 7 8, the prime factors are 2, 3, 5 and 7.

Thus, 8! = 27 32 5 7.

Thus, we have:

11! = 8! 9 10 11

= 27 32 5 7 32 (2 5) 11
 

= 27 32 (5 7 32 2 5 11)
= 27 32 (2 32 52 7 11).

Thus, we have: k = 2 32 52 7 11 = 32 5 (2 5 7 11) = 45 770.




The correct answer is option C.

113. We can rationalize each term as follows:

19992016 Manhattan Review www.manhattanreview.com


218 Number Properties Guide Solutions

1
Taking first term,
1+ 2

21 21 21
= n   o =  2 = = 21
1+ 2 21 2 12 21

1
Taking second term,
2+ 3

3 2 3 2 3 2
= n   o = 2 2 = = 3 2
2+ 3 3 2 3 ( 2) 32

1
Taking third term,
3+2

2 3 2 3 2 3
=   = 2
= =2 3
3+2 2 3 (2)2 ( 3) 43

    
Hence, the required sum = 21 + 3 2 + 2 3 = 2 1 = 1.

The correct answer is option A.


114. We have: |p| q = |r | => q = |p| |r |.


Also, q 3 => p |r | 3 => p 3 + |r |.

We need to minimize (p + r ) and also keep it non-negative.

Since the magnitude of p is greater than that of r , we keep p positive and make r
negative or zero so that (p + r ) becomes minimum and also remains positive.

Possible values of p and r are:


r = 0 => p 3 => Say, p = 3 (minimum) => p + r = 3 + 0 = 3

Or


r = 1 => p 4 => Say, p = 4 (minimum) => p + r = 4 1 = 3, etc.


In each case, the minimum non-negative value of p + r = 3.

The correct answer is option B.

www.manhattanreview.com 19992016 Manhattan Review


Number Properties Guide Solutions 219

7 p 3 7 1 3 4p 10p
115. < < => < < => <q< . Since p & q are positive, thus we
10 q 4 10p q 4p 3 7
can multiply or divide without changing the signs of inequality.

1 3
If p = 1 : 1 < q < 1 => not possible since there is no integer in the range.
3 7
2 6
If p = 2 : 2 < q < 2 => not possible since there is no integer in the range.
3 7
2
If p = 3 : 4 < q < 4 => not possible since there is no integer in the range.
7
1 5
If p = 4 : 5 < q < 5 => not possible since there is no integer in the range.
3 7
2 1
If p = 5 : 6 < q < 7 => q = 7
3 7

For higher values of p, the value of q would also increase.


Thus, the minimum possible value of p + q = 5 + 7 = 12.

The correct answer is option D.

116. We need to have 45 divisible by (4k 1).


Thus, (4k 1) must represent a factor of 45.

The factors of 45 are 1, 3, 5, 9, 15 and 45.

Checking with the factors one at a time, we have:

1
4k 1 = 1 => k = (not an integer)
2
4k 1 = 3 => k = 1 (an integer)
3
4k 1 = 5 => k = (not an integer)
2
5
4k 1 = 9 => k = (not an integer)
2
4k 1 = 15 => k = 4 (an integer)
23
4k 1 = 45 => k = (not an integer)
2

Thus, there are two values of k, 1 and 4.

The correct answer is option C.

q2 + 24 q2 24 24
117. p= = + =q+ .
q q q q

19992016 Manhattan Review www.manhattanreview.com


220 Number Properties Guide Solutions

We need to keep |p| as the least positive integer given that q is a negative integer.

Since p is an integer, q must be the factors of 24.

Since q is negative, the possible values of q are: 1, 2, 3, 4, 6, 8, 12, 24.

We can see that for q = 1 or 24, the value of |p| becomes large.

Thus, we need to choose some value of q in between the extreme values such as 4, 6,
etc.

24
q = 4 : p = 4 + = 10
(4)
24
q = 6 : p = 6 + = 10
(6)

Thus, the minimum integer value of p = |10| = 10.

The correct answer is option C.

37
118. We can break as shown:
16

37 5 1 1 1
=2+ =2+   =2+   =a+  .
16 16 16 1 1
3+ b+
5 5 c

Thus, we have: a = 2, b = 3, c = 1.

Hence, (a + b + c) = 2 + 3 + 1 = 6.

The correct answer is option C.

119. Since the integers a and b must be multiples of the GCD, we have:

a = 6x, b = 6y; where x and y are positive integers and co-prime to one another.

Thus, the LCM of a = 6x and b = 6y is 6xy.

Hence, we have: 6xy = 120 => xy = 20.

Possible values of x and y are:

www.manhattanreview.com 19992016 Manhattan Review


Number Properties Guide Solutions 221

x y

1 20

4 5

5 4

20 1

Thus, the corresponding values of a and b are:

a = 6x b = 6y a+b

6 120 126

24 30 54

30 24 54

120 6 126

Thus, possible values of (a + b) = 126 or 54.

Thus, there are two possible values of (a + b).

The correct answer is option B.

x x
   
120. The values and + 1 will be equal only if the integer parts of both are the same.
3 10
x
We can note that x < 10 => <1
10
x
 
=> 1 < +1 <2
10
x
 
=> + 1 = 1.
10

Thus, we have:

x x x
     
= 1; We know that = + 1 = 1.
3 3 10
x
=> 1 <2
3

=> 3 x < 6.

Thus, possible values of x are: 3, 4 and 5.

19992016 Manhattan Review www.manhattanreview.com


222 Number Properties Guide Solutions

Hence, there are three possible values of x.

The correct answer is option C.

121. Given that: 3 x 8


=> 6 2x 16
=> 16 2x 6 (Multiplying the inequality with minus sign; note the sign change of
the inequality)
=> 16 + 6 6 2x 6 + 6 (Adding 6)
=> 10 6 2x 12
=> |6 2x| 12.
12
 
Thus, the value of will be minimum if the denominator |6 2x| has the
|6 2x|
maximum value i.e. 12.
12 12
 
Hence, the minimum value of = = 1.
|6 2x| 12

The correct answer is option D.

122. (x 2)2 = 25 => x 2 = 5 => x = 2 5 = 7 or 3.

4 4
xy 3 = 256 => x y 3 = (4)4

=> x y 3 = 4 => y = x 3 4

By plugging in the values of x = 7 or3, we get,

=> y = 7 3 4 or 3 3 4 => y = 8 or 0 or 10 or 2.

www.manhattanreview.com 19992016 Manhattan Review


Number Properties Guide Solutions 223

The values of x and y are shown below:

x y xy |xy|

7 8 56 56

7 0 0 0

3 10 30 30

3 2 6 6

Thus, the maximum value of xy = 56.
(Simply selecting the values
of x and y with the highest magnitudes would have given
us x = 7, y = 10 => xy = 70; which is not correct since the value of y = 10 does
not correspond to the value of x = 7)

The correct answer is option D.

123. 212 + 213 + 214 + 215 + 216 + 217

= 212 1 + 2 + 22 + 23 + 24 + 25


= 212 (1 + 2 + 4 + 8 + 16 + 32) = 212 63 = 212 32 7.

Working with the options:

Option A: 12 = 22 3 (is a factor)


Option B: 56 = 23 7 (is a factor)
Option C: 72 = 23 32 (is a factor)
Option D: 132 = 22 3 11 (is not a factor since 11 is not a factor of the number)
Option E: 144 = 24 32 (is a factor)

The correct answer is option D.

124. Since the number aba is divisible by 4, the number formed by the last two digits i.e. ba
should be divisible by 4.

Thus, we need to find two-digit multiples of 4.

The first two-digit multiple of 4 is 12 = 4 3 and the last two-digit multiple of 4 is


96 = 4 24.

19992016 Manhattan Review www.manhattanreview.com


224 Number Properties Guide Solutions

Thus, number of such multiples = 243 + 1 = 22.

Now, since we do not want any digit to be 0, we need to remove some cases.

The digit a will be 0 only when 4 is multiplied with a number ending in 5 or 0.


The cases are: 4 5, 4 10, 4 15 and 4 20 i.e. four cases.

Thus, the required number of multiples of 4 so that no digit is 0 is 224 = 18.


(We do not need to check the hundreds digit since it is the same as the tens digit which
we have determined above)

The correct answer is option B.

125. Since the pair-wise GCD of the numbers is 3, the numbers can be represented as co-prime
multiples of 3.
Thus, we may say that the numbers are 3a, 3b and 3c.

Hence, we have: 3a 3b 3c = 540 => 27abc = 540 => abc = 20.

Possible values of a, b and c are shown below:

a b c

1 1 20

1 4 5

The order of a, b, c is not important since they result in the same number.
Also, while assigning values to a, b, c we should make sure that they are pair-wise
co-prime. Hence, a set of values like a = 1, b = 2, c = 10 is not possible since b and c
are not co-prime.

Again, in the above table, the first solution, i.e. a = 1, b = 1, c = 20 is not acceptable
since that results in the same value of a and b while it has been stated that the three
numbers are distinct.

Thus, there are only one set of values of the three numbers: 3, 12 & 15.

The correct answer is option A.

www.manhattanreview.com 19992016 Manhattan Review


Number Properties Guide Solutions 225

126. We have the LCM of the numbers 1, 2, 3, . . . 25 as L.


We need to find the LCM of the numbers 1, 2, 3, . . . 25, 26, 27 in terms of L.

In the set of the first 27 positive integers, the additional numbers are 26 and 27.
26 = 2 13
27 = 33

The factors of 26 i.e. 2 and 13 are already present in the earlier set of numbers from 1
to 25.
Thus, inclusion of 26 does not change the LCM.

However, the inclusion of 27 i.e. 33 will change the LCM since in the earlier set of num-
bers, the highest exponent of 3 present is 2 in 9 = 32 and 18 = 32 2.
Thus, 27 introduces an extra 3 in the LCM.

Thus, the inclusion of 27 would make the new LCM thrice of the earlier value.

Hence, the new LCM is 3L.

The correct answer is option B.

127. We consider each term separately:

111! = 1 (since 1 raised to any exponent is always 1)

212! : The last digit of exponents of 2 follows a cycle of four places as shown:

24k+1 2

24k+2 4

24k+3 8

24k 6

We know that 12! = 1 2 3 4 . . . 12 i.e. a multiple of 4.

Since 12! Is a multiple of 4, the units digit of 212! = 6.

313! : The last digit of exponents of 3 follows a cycle of four places as shown:

19992016 Manhattan Review www.manhattanreview.com


226 Number Properties Guide Solutions

34k+1 3

34k+2 9

34k+3 7

34k 1

We know that 13! = 1 2 3 4 . . . 13 i.e. a multiple of 4.

Since 13! Is a multiple of 4, the units digit of 313! = 1.

Hence, the last digit of 111! + 212! + 313! = 1 + 6 + 1 = 8.

The correct answer is option E.

2x + 28 2 (x + 6) + 16 16
 
128. = =2+
x+6 x+6 x+6
2x + 28 16
   
Thus, will be an integer if is an integer.
x+6 x+6
Thus, (x + 6) is a factor of 16.

However, since we need x to be a positive integer, we must have (x + 6) > 6.


Thus, (x + 6) is a factor of 16 which is greater than 6.

The factors of 16 greater than 6 are: 16 and 8.

Thus, we have:

x + 6 = 16 => x = 10.
x + 6 = 8 => x = 2.

Thus, sum of all possible values of x is 10 + 2 = 12.

The correct answer is option D.

129. ab = 1 is true under any of the following cases:

(1) a = 1 and any value of b


(2) b = 0 and any value of a
(3) a = 1 and any even value of b

www.manhattanreview.com 19992016 Manhattan Review


Number Properties Guide Solutions 227

Working according to the above cases:

(1) 2x 1 = 1 => x = 1
(2) x + 4 = 0 => x = 4
(3) 2x 1 = 1 => x = 0.

If x = 0, the value of the exponent = x + 4 = 0 + 4 = 4 (even).

Thus, the possible values of x are 1, 4 and 0.

Hence, the product of the possible values of x = 1 (4) 0 = 0.

The correct answer is option C.

130. Let the three digit number be abc, where a, b and c are the digits in the hundreds, tens
and units positions, respectively.

We have: a + b + c = 6.

Since we want to maximize the product of the digits, we should not use 0 for any digit
since then the product would become 0 as well.

The sum of 6 by adding three non-zero digits (in any order) can be obtained in the
following ways:

6 = 1 + 1 + 4 => Product of the digits = 4


6 = 1 + 2 + 3 => Product of the digits = 6
6 = 2 + 2 + 2 => Product of the digits = 8

Hence, the maximum value of the product of the digits is 8.

The correct answer is option D.

131. 2x + 2xy = y
=> 2x y + 2xy = 0
=> 2x y + 2xy 1 = 1 (We introduce 1 on both sides so that we can factorize the
LHS)

=> (2x 1) y + 1 = 1.
Since x and y are integers, (2x 1) and (y + 1) must be factors of 1.
Thus, we have the following cases:

19992016 Manhattan Review www.manhattanreview.com


228 Number Properties Guide Solutions

(1) 2x 1 = 1 => x = 1 and y + 1 = 1 => y = 2


(2) 2x 1 = 1 => x = 0 and y + 1 = 1 => y = 0

Thus, x can have two possible values.

The correct answer is option C.


132. We know that 2 > 1.96 = 1.4, and 2 < 2.25 = 1.5 (since 142 = 196 and 152 = 225)

Thus, 1.4 < 2 < 1.5.
Thus, we have:

   
1 1
1.4 < 1 + a+b < 1.5 => 0.4 < a+b < 0.5

1 1
=> 0.4 > (a + b) > 0.5 => 2.5 > (a + b) > 2.

Thus, the maximum possible integer value of a = 2.

The correct answer is option B.

Alternate approach:


1

1+ a+b = 2
 
1
=> a+b = 1.414 1 = 0.414
1
=> a+b > 0.4
1
=> a + b < 0.4
10
=> a + b < 4
=> a + b < 2.5

Thus the maximum integer value a can assume is 2.

133. We have: a = bc = (c a )c (substituting b = c a )


ac
= c ac = a4 (substituting c = a4 )
= a4ac .

Thus, we have: a = a4ac => a1 = a4ac => 1 = 4ac (comparing exponents since bases
are same) => ac = 41 .

The correct answer is option A.

www.manhattanreview.com 19992016 Manhattan Review


Number Properties Guide Solutions 229

134. We have: [x] + x = 5.

Since both 5 and [x] are integers, x must also be an integer.


Thus, [x] = x.

Hence, we have: x + x = 5 => x = 2.5.

However, it contradicts the fact that x is an integer.

Thus, there are no possible values of x.

The correct answer is option A.

Let h be the factor common to both 3n2 + 2 and (9n2 + 4).



135.

Thus, h must be a factor of the difference of the two numbers i.e.
9n2 + 4 3n2 + 2 = 6n2 + 2 .


Since h is a factor of (3n 2 2


 (6n 2+ 2), h must
 + 22) and also be a factor of the difference
of the two numbers i.e. 6n + 2 3n + 2 = 3n2 .

Thus, h is a factor of 3n2 and 3n2 + 2 .


 

possible for any value of h except h = 1 (since 3n2 is a multiple of 3 while



This is not
3n2 + 2 is not a multiple of 3)


Thus, 3n2 + 2 and 9n2 + 4 have no common factors other than 1 for any value of n.
 

The correct answer is option E.


136. The three digit number xyz can be written as: 100x + 10y + z .
The sum of digits of the above number = (x + y + z).
!
100x + 10y + z
Thus, we need to minimize the value of .
x+y +z
 
100x + 10y + z 99x + 9y + x + y + z 99x + 9y
= = + 1.
x+y +z x+y +z x+y +z

We observe that z is only present in the denominator.


Hence, we need to maximize z so that the ratio becomes minimum.
Hence, z = 9 (maximum value of a digit).

19992016 Manhattan Review www.manhattanreview.com


230 Number Properties Guide Solutions

Since x is multiplied with 99, we need to minimize the value of x so that the ratio
becomes minimum.
Hence, x = 1 (minimum value of a digit).

Thus, the ratio becomes:

 !
99 + 9y 99 + 9y 9 10 + y + 9 9 9
+1= +1= +1= 9+ + 1 = 10 + .
1+y +9 10 + y 10 + y 10 + y 10 + y

This will be minimum if we maximize the value of y.

Hence, y = 9 (maximum value of a digit).

9 9
 
Thus, the ratio finally becomes: 10 + = 10 .
10 + 9 19

Hence, we have: x = 1, y = 9, z = 9 => x + y + z = 19.

The correct answer is option E.

137. The two digit number x9 can be represented as (10x + 9).


The sum of digits of the above number = (x + 9).

Thus, we need to find the remainder when (10x + 9) is divided by (x + 9).

10x + 9 9x + (x + 9) 9x
 
= = + 1.
x+9 x+9 x+9

(Note: The 1 above is a part of the quotient and does not contribute to the remainder)
Thus, the remainder is the same as the remainder when 9x is divided by (x + 9).

Working with the options for x and substituting above, we have:

x = 1 : 9 is divided by 10 => The remainder is 9.


x = 2 : 18 is divided by 11 => The remainder is 7.
x = 3 : 27 is divided by 12 => The remainder is 3.
x = 7 : 63 is divided by 16 => The remainder is 15.
x = 8 : 72 is divided by 17 => The remainder is 4.

Thus, we see that the remainder is maximum when x = 7.

The correct answer is option D.

www.manhattanreview.com 19992016 Manhattan Review


Number Properties Guide Solutions 231

5.3.2 Data Sufficiency

138. Since x and y are positive integers, we can observe that possible values of x can be 1, 2
or 3.
The values of x and y are shown in the table below:

x y

1 18

2 10

3 2

Statement 1:
Since x + y > 10, we can see that two possible values exist:

x y

1 18

2 10

Thus, we have: y = 18 or 10.


Thus, the value of y is not unique.
Thus, statement 1 is not sufficient to answer the question.

The correct answer would be among B/C/E.

Statement 2:
Since x + y < 15, we can see that two possible values exist:

x y

2 10

3 2

Thus, we have: y = 10 or 2.
Thus, the value of y is not unique.
Thus, statement 2 is not sufficient to answer the question.

The correct answer would be between C/E.

19992016 Manhattan Review www.manhattanreview.com


232 Number Properties Guide Solutions

Statements 1 and 2 together:


From statements 1 and 2, we find that x = 2 and y = 10 satisfies both.
Hence, we have a unique value of y = 10.

Thus, statements 1 and 2 together are sufficient to answer the question and the answer
is y = 10.

The correct answer is option C.

139. Let us look at the statements one by one.

Statement 1:
y 2 = 36
We do not know anything about x,so statement 1 alone is not sufficient.

Statement 2:
5|x| = 3y
Since x is a non-negative integer, x will either be positive or 0, so this equation can be
simplified as 5x = 3y. Though we now know the ratio of x and y, we cannot find the
value of 5x + 3y.So statement 2 alone is not sufficient.

Statement 1 and 2 together:


From statement 2, we know that 5x = 3y, this follows that since x is non-negative, y
will also be non-negative. Again, from statement 1, we know that y 2 = 36 => y = +6.
So 5x = 3y = 3 6 = 18, and 5x + 3y = 18 + 18 = 36.

The correct answer is option C.

140. Let us look at the statements one by one.

Statement 1: |a| = |b|


a
If |a| = |b|, then a = b or a = b. In both scenarios, is an integer.
b

So statement 1 alone is sufficient.

Statement 2:
a and b both are unequal non-integers numbers of the same magnitude.
There are three information about a and b.

(1) a and b are non-integers


(2) a and b are of same magnitude, it means that |a| = |b|

www.manhattanreview.com 19992016 Manhattan Review


Number Properties Guide Solutions 233

(3) a and b are unequal. From condition 2 above we can infer that a = b
a 2.5
Say a = 2.5, so b must be 2.5. This means that = = 1
b 2.5

So statement 2 alone is also sufficient.

The correct answer is option D.

141. Statement 1:
xp + xq 0 => x(p + q) 0.

Thus, there are two possibilities:

(1) x 0 and (p+q) 0: Since (p+q) 0, we can have p > 0 and q < 0 satisfying p > q
(say: p = 4, q = 2) OR p < 0 and q > 0 not satisfying p > q (say: p = 2, q = 4).
(2) x 0 and (p + q) 0: Since (p + q) 0, we can have p > 0 and q < 0 satisfying
p > q (say: p = 2, q = 4) OR p < 0 and q > 0 not satisfying p > q (say:
p = 4, q = 2).

Hence, statement 1 is not sufficient to answer the question as the answer may be Yes or
No.

The correct answer would be among B/C/E.

Statement 2:
xp + xq 0 => x(p + q) 0.

Thus, there are two possibilities:

(1) x 0 and (p + q) 0
(2) x 0 and (p + q) 0

Thus, we have the same situations as discussed above.

Hence, statement 2 is not sufficient to answer the question as the answer may be Yes or
No.

The correct answer would be between C/E.

Statements 1 and 2 together:


Combining both statements, we have: xp + xq = 0 => x(p + q) = 0

19992016 Manhattan Review www.manhattanreview.com


234 Number Properties Guide Solutions

Thus, we have two possibilities:

(1) x = 0: Here, p and q can take any value either satisfying or not satisfying p > q
(2) p + q = 0 => p = q: Here, either of p or q may be negative either satisfying or
not satisfying p > q

Hence, statements 1 and 2 together are not sufficient to answer the question.

The correct answer is option E.

142. Statement 1:
36
4 = 14
x
36
=> = 18
x
=> x = 2
Thus, we get a unique value of x.
Hence, statement 1 is sufficient to answer the question.

The correct answer would be between A/D.

Statement 2:
(x 4) (x 1) = x
=> x 2 5x + 4 = x
=> x 2 4x + 4 = 0
=> (x 2)2 = 0
=> x = 2
Thus, though it is a quadratic, it gives us a unique solution for x.

Hence, statement 2 is sufficient to answer the question.

The correct answer is option D.

143. Statement 1:
p = q4 = 16
=> q = 2
p 16
Thus, we have: = = 8.
q 2
p
Thus, we cannot determine a unique value of .
q

www.manhattanreview.com 19992016 Manhattan Review


Number Properties Guide Solutions 235

Hence, statement 1 is not sufficient to answer the question.

The correct answer would be among B/C/E.

Statement 2:
pq = 32
Substituting p = q4 , we get: q4 q = 32
=> q5 = 25
=> q = 2.
Thus, p = 24 = 16.
p 16
Thus, we get: = = 8, a unique value.
q 2

Hence, statement 2 is sufficient to answer the question.

The correct answer is option B.

144. Statement 1:
   
p 7 p + 2 is even if at least one among p 7 and p + 2 is even:

Case 1: (p 7) is even => p = (7+ even) i.e. odd.

Case 2: (p + 2) is even => p = (even 2) i.e. even.

Thus, p can be odd or even.


Hence, statement 1 is not sufficient to answer the question as the answer may be Yes
and No.

The correct answer would be among B/C/E.

Statement 2:
even 8
2p is even => p = which may be even as well as odd (for example: =
2 2
6
4 (even) ; = 3 (odd)).
2
Hence, statement 2 is not sufficient to answer the question, as the answer may be Yes or
No.

The correct answer would be between C/E.

Statements 1 and 2 together:

19992016 Manhattan Review www.manhattanreview.com


236 Number Properties Guide Solutions

Since both statements lead to the same conclusion that p may be odd as well as even,
we cannot find a unique answer even after combining both statements.

Hence, statements 1 and 2 together are not sufficient to answer the question.

The correct answer is option E.

145. Statement 1:
a3 = a
=> a3 a = 0
=> a a2 1 = 0


=> a (a 1) (a + 1) = 0
=> a = 0 or 1 or 1.
However, since a 6= 0 => a = 1 or 1.

Hence, statement 1 is not sufficient to answer the question, as the answer may be Yes or
No.

The correct answer would be among B/C/E.

Statement 2:
a2 = a
=> a2 + a = 0
=> a (a + 1) = 0
=> a = 0 or 1.
However, since a 6= 0 => a = 1.

Hence, statement 2 is sufficient to answer the question and the answer is Yes.

The correct answer is option B.

146. Statement 1:
Between 83 and 96 (none inclusive), the only prime is 89.
Thus, if p = 84, the number of primes between 82 and p is 1 (only 83).
In the other extreme, if p = 95, the number of primes between 82 and p is 2 (both 83
and 89).
Thus, the number of primes between 82 and p can be either one or two.

Hence, statement 1 is not sufficient to answer the question.

www.manhattanreview.com 19992016 Manhattan Review


Number Properties Guide Solutions 237

The correct answer would be among B/C/E.

Statement 2:
Between 90 and 99 (none inclusive), the only prime is 97.
Thus, if p = 91, the number of primes between 82 and p is 2 (both 83 and 89).
In the other extreme, if p = 98, the number of primes between 82 and p is 3 (83, 89 and
97).
Thus, the number of primes between 82 and p can be either two or three.

Hence, statement 2 is not sufficient to answer the question.

The correct answer would be between C/E.

Statements 1 and 2 together:


Combining both statements, we see that 90 < p < 96, so the number of primes must be
two (83 and 89 only).

Hence, statements 1 and 2 together are sufficient to answer the question.

The correct answer is option C.

147. pq > 0 is possible under the following conditions:


(1) p > 0 and q > 0
(2) p < 0 and q < 0
Statement 1:
p 2 q3 < 0
2
=> pq q < 0
2
Since pq is a perfect square, it must be positive.
Thus, we can conclude that q < 0.
However, since we do not know anything about p, we cannot determine the answer.
Hence, statement 1 is not sufficient to answer the question.

The correct answer would be among B/C/E.

Statement 2:
pq2 < 0
Since q2 is a perfect square, it must be positive.
Thus, we can conclude that p < 0.
However, since we do not know anything about q, we cannot determine the answer.

19992016 Manhattan Review www.manhattanreview.com


238 Number Properties Guide Solutions

Hence, statement 2 is not sufficient to answer the question.

The correct answer would be between C/E.

Statements 1 and 2 together:


Combining both statements, we see that p < 0 and q < 0 => pq > 0.

Hence, statements 1 and 2 together are sufficient to answer the question and the answer
is Yes.

The correct answer is option C.

148. The question means whether the set has an element that is equal to the average of all
the elements in the set, including that element.

For example, if the set has three elements: 10, 20, & 30, to its average is 20, and one of
the elements is 20, so answer is Yes.

However if the set has three elements: 10, 17, & 30, to its average is 19, but there is no
such element in the set equals 19, so answer is No.

Statement 1:
The statement implies that the numbers in the set are any multiples of 6 such that they
are distinct.

Since the numbers can be randomly chosen, the average of all the numbers may or may
not be a part of the set.

For example, let the set be: S = {6 1, 6 5, 6 8, 6 18} i.e. {6, 30, 48, 108}.

6 + 30 + 48 + 108
The average of the above numbers = = 48, which is a number of the
4
set.

Again, let the set be: S = {6 1, 6 2, 6 12} i.e. {6, 12, 72}.

6 + 12 + 72
The average of the above numbers = = 30, which is not a number of the set.
3

Thus, there may or may not be a number which is an element of the set S.

Hence, statement 1 is not sufficient to answer the question.

www.manhattanreview.com 19992016 Manhattan Review


Number Properties Guide Solutions 239

The correct answer would be among B/C/E.

Statement 2:
Since the set contains an odd number of terms, and the terms are consecutive multiples
of 6, we can definitely say that the average of the terms will be a term of the set.
Let us see how:
Let the set have 5 terms and the consecutive multiples be: m, (m + 1), (m + 2), (m + 3)
and (m + 4).

Thus, the numbers are: 6m, 6 (m + 1) , 6 (m + 2) , 6(m + 3) and 6(m + 4).

6m + 6 (m + 1) + 6 (m + 2) + 6 (m + 3) + 6 (m + 4)
Thus, their average = = 6(m + 2).
5

Thus, we see that the average is the middle term.

In general, since there are an odd number of terms, there will always be a single middle
term and that will be the average of all the terms.

Hence, statement 2 is sufficient to answer the question.

The correct answer is option B.

149. Statement 1:
!2 !2
p q
+ =2
q p
p q 1
Let x = => = .
q p x

Thus, we have:

1 1 1
x2 + = 2 => x 2 + 2 + 2 x = 2 + 2
x2 x x
1 2 1
 
=> x + = 4 => x + = 2
x x
=> x 2 + 1 = 2x
=> x 2 2x + 1 = 0 => (x 1)2 = 0
=> x = 1
Hence, statement 1 is not sufficient to answer the question as the answer may be Yes or
No.
The correct answer would be among B/C/E.

19992016 Manhattan Review www.manhattanreview.com


240 Number Properties Guide Solutions

!2 !2
p q p
Alternatively, we see that = = 1 => p = q => = 1
q p q

Statement 2:
p q 1
Let x = => = .
q p x

Thus, we have:

1
x+ = 2 => x 2 + 1 = 2x
x
=> x 2 2x + 1 = 0 => (x 1)2 = 0
=> x = 1
Hence, statement 2 is sufficient to answer the question and the answer is Yes.

p q
Alternatively, we see that = = 1.
q p

The correct answer is option B.

150. Statement 1:
k(k 1)2 = 4k => k(k 1)2 4k = 0
n o
=> k (k 1)2 22 = 0
=> k {(k 1 2) (k 1 + 2)} = 0
=> k (k 3) (k + 1) = 0
=> k = 0 or 3 or 1.
Hence, statement 1 is not sufficient to answer the question as the answer is not unique.
The correct answer would be among B/C/E.

Statement 2:
Since k is an integer divisible by 3, the value of k may be all negative multiples of 3, all
positive multiples of 3 or 0.
Hence, statement 2 is not sufficient to answer the question as the answer is not unique.
The correct answer would be between C/E.

Statements 1 and 2 together:


Combining both statements, we have k = 0 or 3

Hence, statements 1and 2 together are not sufficient to answer the question as the
answer is not unique.

The correct answer is option E.

www.manhattanreview.com 19992016 Manhattan Review


Number Properties Guide Solutions 241

151. Statement 1:
We know that the symbol stands for one among addition, subtraction, multiplication
or division.
Let us check one by one:

Let represent addition: r + 0 = r (satisfies).

Let represent subtraction: r 0 = r (satisfies).

Let represent multiplication: r 0 = 0 6= r (does not satisfy since we know that r 6= 0).

Let represent division: r 0 6= r (does not satisfy since division by 0 is not defined).

Hence, statement 1 is not sufficient to answer the question as the answer is not unique.

The correct answer would be among B/C/E.

Statement 2:
Let us check one by one:

Let represent addition: (r + 0) + r = 2r 6= 0 (does not satisfy).

Let represent subtraction: (r 0) r = 0 (satisfies).

Let represent multiplication: (r 0) r = 0 (satisfies).

Let represent division: (r 0) r 6= 0 (does not satisfy since division by 0 is not


defined).

Hence, statement 2 is not sufficient to answer the question as the answer is not unique.

The correct answer would be between C/E.

Statements 1 and 2 together:


Combining both statement, we see that the symbol represents subtraction.

Hence, statements 1 and 2 together are sufficient to answer the question.

The correct answer is option C.

19992016 Manhattan Review www.manhattanreview.com


242 Number Properties Guide Solutions

152. Since p, q and r denote the three sides of a triangle, sum of two sides must be greater
than the third side:

p + q > r . . . (i)
p + r > q . . . (ii)
q + r > p . . . (iii)

Statement 1:
Since p + r > q and q = r + 6 => p + r > r + 6 => p > 6.

Since p, q, r are integers, the minimum value of p = 7.

Hence, statement 1 is sufficient to answer the question.

The correct answer would be between A/D.

Statement 2:
Since q = 8, r = 2 and p + r > q => p + 2 > 8 => p > 6.
Since p, q, r are integers, the minimum value of p = 7.
Hence, statement 2 is sufficient to answer the question.

The correct answer is option D.

www.manhattanreview.com 19992016 Manhattan Review


Number Properties Guide Solutions 243

153. Statement 1:
Since the tens digit is twice the units digit, the possible values of A can be 21, 42, 63, or
84.
Hence, statement 1 is not sufficient to answer the question as the answer is not unique.

The correct answer would be among B/C/E.

Statement 2:
Since the sum of the digits is 6, possible values of A can be 15, 51, 24, 42, 33 or 60.
Hence, statement 2 is not sufficient to answer the question as the answer is not unique.

The correct answer would be between C/E.

Statements 1 and 2 together:


Combining both statements, we can see that the only number possible is 42.

Hence, statements 1 and 2 together are sufficient to answer the question.

The correct answer is option C.

154. m + n2 is odd if m is even and n is odd (i.e. n2 is odd) OR if m is odd and n is even (i.e.
n2 is even).

Statement 1:
mn + n2 is odd => n(m + n) is odd.

Thus, n is odd and (m + n) is odd => m is even.

Thus, we have: m is even and n is odd.

Hence, m + n2 is odd.

Hence, statement 1 is sufficient to answer the question and the answer is Yes.

The correct answer would be between A/D.

19992016 Manhattan Review www.manhattanreview.com


244 Number Properties Guide Solutions

Statement 2:
m2 + n2 is odd if m is even (i.e. m2 is even) and n is odd (i.e. n2 is odd) OR if m is odd
(i.e. m2 is odd) and n is even (i.e. n2 is even).

This is the same condition as required in the question.

Hence, m + n2 is odd.

Hence, statement 2 is sufficient to answer the question and the answer is Yes.

The correct answer is option D.

155. Statement 1:
A prime number, greater than 5 is odd.
Thus, r is 3 times an odd number which is again odd.
Also, s is not divisible by 2, hence s is odd.

Thus, r s is a product of two odd numbers i.e. odd.


However, we do not have any information on t.

Hence, statement 1 is not sufficient to answer the question.

The correct answer would be among B/C/E.

Statement 2:
Since t leaves an odd remainder on being divided by 4 (an even number), t is odd.
However, we do not have any information on r and s.

Hence, statement 2 is not sufficient to answer the question.

The correct answer would be between C/E.

Statements 1 and 2 together:


Combining both statements, we can say that all of r , s and t are odd.
Thus, r s + t is the sum of two odd numbers and hence it is even.

Hence, statements 1 and 2 together are sufficient to answer the question and the answer
is Yes.

The correct answer is option C.

www.manhattanreview.com 19992016 Manhattan Review


Number Properties Guide Solutions 245

156. Statement 1:
Since p < 8 and is an odd prime, possible values of p are 3, 5 or 7.
However, there is no information on m.

Hence, statement 1 is not sufficient to answer the question.


The correct answer would be among B/C/E.

Statement 2:
m = 60 = 22 3 5.
Thus, m is a multiple of the following odd primes: 3 and 5.
Thus, we have: p = 3 or 5.

However, m is neither a multiple of 32 = 9 nor 52 = 25.

Hence, statement 2 is sufficient to answer the question and the answer is No.

The correct answer is option B.

x4y 3 x2y 5 x2y 3 x2 y 2



157. =  = x2.
x2y 3 y 5 y 3 x2 y 2

Statement 1:
Since x = 2, we can determine the value of the expression to be x 2 = 22 = 4.
Hence, statement 1 is sufficient to answer the question.
The correct answer would be between A/D.

Statement 2:
Since xy = 1 and both x and y are integers, possible values of x and y are:

(1) x=y =1
(2) x = y = 1

Thus, we have x = 1 or 1.

Hence, the value of the expression is x 2 = 1.

Hence, statement 2 is sufficient to answer the question.

The correct answer is option D.

19992016 Manhattan Review www.manhattanreview.com


246 Number Properties Guide Solutions

158. Statement 1:

m n
=
n m
=> m2 = n2
=> m2 n2 = 0 => (m + n) (m n) = 0
=> m = n OR m = n
m
=> = 1 OR 1.
n
Hence, statement 1 is not sufficient to answer the question, as the answer may be Yes or
No.

The correct answer would be among B/C/E.

Statement 2:
(m n)2 = m2 n2
=> m2 2mn + n2 = m2 n2
=> 2n2 2mn = 0 => 2n (n m) = 0
=> n = 0 OR m = n.
m
=> n = 0 OR = 1.
n
m m
Thus, we may have = 1 OR we may have n = 0 (when 6= 1)
n n
Hence, statement 2 is not sufficient to answer the question.

The correct answer would be between C/E.

Statements 1 and 2 together:


m
Combining both statements, we can definitely say that = 1 (since n cannot be 0 as
n
m
then the value of would be undefined).
n

Hence, statements 1 and 2 together are sufficient to answer the question.

The correct answer is option C.

159. Statement 1:
Since we do not have any information on the values of a, b, c and d, we cannot answer
the question.
Hence, statement 1 is not sufficient to answer the question.

The correct answer would be among B/C/E.

www.manhattanreview.com 19992016 Manhattan Review


Number Properties Guide Solutions 247

Statement 2:
a + b + c + d = 8 can be made possible in many ways (three such ways are shown below):

a b c d abcd

5 1 1 1 5 (No)

3 2 2 1 12 (No)

2 2 2 2 16 (Yes)

Hence, statement 2 is not sufficient to answer the question, as the answer is not unique.

The correct answer would be between C/E.

Statements 1 and 2 together:


Combing both statements and using the inequality from the first statement that
a b c d, we can see that all the situations of the values of a, b, c and d as obtained
in the second statement are still valid.

Hence, statements 1 and 2 are not sufficient to answer the question, as the answer is not
unique.

The correct answer is option E.

160. Statement 1:
a + b + c + d = 9 can be made possible in many ways (three such ways are shown below):

a b c d abcd

6 1 1 1 6

4 2 2 1 16

3 3 2 1 18

Hence, statement 1 is not sufficient to answer the question, as the answer is not unique.

The correct answer would be among B/C/E.

Statement 2:
Since we do not have any information on the values of a, b, c and d, we cannot answer
the question.
Hence, statement 1 is not sufficient to answer the question.

19992016 Manhattan Review www.manhattanreview.com


248 Number Properties Guide Solutions

The correct answer would be between C/E.

Statements 1 and 2 together:


Combing both statements and using the inequality from the second statement that
a b > c > d, we can see that only one situation is valid when a = b = 3, c = 2 and
d = 1.

Logically, we can get the values in the following manner:

The value of d must be 1 (Else, if d = 2, then mimimum values of c = 3 and b = 4. Thus,


we have: b + c + d = 9 => a = 0 which is not possible since a b).

Since d = 1, c = 2 and b = 3 (If c = 3, then minimum value of b = 4. Thus, we have:


b + c + d = 8 => a = 1 which is not possible since a b).

Since d = 1, c = 2 and b = 3, a = 9 (b + c + d) = 3 (this satisfies the condition that


a b).

Hence, we have a = b = 3, c = 2 and d = 1.

Hence, statements 1 and 2 are sufficient to answer the question.

The correct answer is option C.

161. Statement 1:
The prime factors of 105 are 3, 5 and 7.

Since abc is prime, it must be that two among a, b and c must be 1 and the remaining
one must be a prime number.

However, since we cannot determine whether the above prime number is a factor of 105,
we cannot answer the question. If the sole prime number among a, b, & c is 3, 5, or 7,
the answer is yes, however if it is a prime number other than these three, the answer is no.

Hence, statement 1 is not sufficient to answer the question.

The correct answer would be among B/C/E.

Statement 2:
Since we do not have any idea about the exact values of a, b and c, we cannot answer
the question.
Hence, statement 2 is not sufficient to answer the question.

www.manhattanreview.com 19992016 Manhattan Review


Number Properties Guide Solutions 249

The correct answer would be between C/E.

Statements 1 and 2 together:


From the first statement, we know that two among a, b and c are 1 and the remaining
is a prime.

From the second statement, we can see that only a and b can have a value 1 while c
cannot as the minimum value of c = 3.

Thus, we can conclude that a = b = 1.

Thus, c is a prime such that 3 c 10 => c = 3, 5 or 7.

Since the factors of 105 are also 3, 5 and 7; we can conclude that for any value of c
among 3, 5 or 7, 105 will be divisible by c.

105
Hence, is an integer.
abc

Hence, statements 1 and 2 together are sufficient to answer the question.

The correct answer is option C.

162. P8 = a1 a2 a3 a4 a5 a6 a7 a8

Statement 1:
There are four terms having an odd suffix and four terms having an even suffix in P8 .

Since a1 > 0, all terms having an odd suffix are positive.

However, we do not know whether the terms with an even suffix (other than a2 ) are
positive or negative.

Hence, statement 1 is not sufficient to answer the question.

The correct answer would be among B/C/E.

Statement 2:
Since a2 < 0, all terms having an even suffix are negative.

However, we do not know whether the terms with an odd suffix (other than a1 ) are
positive or negative.

19992016 Manhattan Review www.manhattanreview.com


250 Number Properties Guide Solutions

Hence, statement 2 is not sufficient to answer the question.

The correct answer would be between C/E.

Statements 1 and 2 together:


Combining both statements, we have all terms with an odd suffix as positive and all
terms with an even suffix as negative.

Since there are four terms with an even suffix, there are four negative terms and hence,
their product is positive.

Thus, P8 is positive.

Hence, statements 1 and 2 together are sufficient to answer the question and the answer
is Yes.

The correct answer is option C.

163. Statement 1:

m3
1 => m3 n (since n is positive, we can cross-multiply without changing the
n
inequality)

If m n, it would imply that: m n m3 => m m3

This is true only if 0 m 1.

However, such a condition has not been mentioned and hence, we cannot determine if it
is true.

Hence, statement 1 is not sufficient to answer the question.

The correct answer would be among B/C/E.

Statement 2:

3
m
1 => 3 m n (since n is positive, we can cross-multiply without changing the
n
inequality)


3

3
If m n, it would imply that: m n m => m m

www.manhattanreview.com 19992016 Manhattan Review


Number Properties Guide Solutions 251

This is true only if m 1.

However, such a condition has not been mentioned and hence, we cannot determine if it
is true.

Hence, statement 2 is not sufficient to answer the question.

The correct answer would be between C/E.

Statements 1 and 2 together:


We have n m3 and n 3 m.

Case (a): If 0 m 1: 3 m m m3 Hence, n 3 m => n m.


Case (b): If m 1: m3 m 3
m Hence, n m3 => n m.

Hence, statements 1 and 2 together are sufficient to answer the question and the answer
is No.

The correct answer is option C.

164. Statement 1:
Since a b c d e is odd, we can conclude each of a, b, c, d and e are odd.
Thus, a + b + c + d + e is the sum of five odd numbers which is odd.

Hence, statement 1 is sufficient to answer the question and the answer is No.

The correct answer would be between A/D.

19992016 Manhattan Review www.manhattanreview.com


252 Number Properties Guide Solutions

Statement 2:
Since a b c d e is even, we can conclude at least one among a, b, c, d and e is
even.

Thus, it may be that one of the numbers is even and the other four odd OR two of them
even and the other three odd, etc.

Thus, a + b + c + d + e can be the sum of one even and four odd numbers which is even
OR it can be the sum of two even and three odd numbers which is odd.

Hence, the answer may be Yes or No.

Hence, statement 2 is not sufficient to answer the question.

The correct answer is option A.

165. Statement 1:
Since (a b c) is odd, we can conclude that each of a, b and c are odd.
Hence, both (a b) and (b c) are difference of two odd numbers which is even.
Hence, (a b)(b c) is the product of two even numbers i.e. even.

Hence, statement 1 is sufficient to answer the question and the answer is Yes.

The correct answer would be between A/D.

Statement 2:
Since (a b c) is even, we can conclude that at least one among a, b and c is even.
If a is even and the other two are odd, then (b c) is the difference of two odd numbers
i.e. even.
Hence, (a b)(b c) is even.

However, if a is even, b is odd and c is even, then both (a b) and (b c) are odd.
Hence, (a b)(b c) is odd.

Hence, the answer can be Yes or No.

Hence, statement 2 is not sufficient to answer the question.

The correct answer is option A.

www.manhattanreview.com 19992016 Manhattan Review


Number Properties Guide Solutions 253

166. It is given that: 3p + q2 = 42 => q2 = 42 3p => q2 = 3(14 p).


Thus, q2 must be a multiple of 3.
Since q2 is a perfect square, 3(14 p) must be perfect square.
However, 3 is not a perfect square; hence q2 is a multiple of 32 = 9.

Thus, q2 = 9 or 36 (since p is positive, q2 < 42) => q = 3 or 6.

Statement 1:
We know that p is prime and q = 3 or 6. Thus, we have:
q2 = 9 => 3p + 9 = 42 => p = 11 (prime).
q2 = 36 => 3p + 36 = 42 => p = 2 (prime).

Thus, the value of q is not unique since q can be 3 or 6.

Hence, statement 1 is not sufficient to answer the question.

The correct answer would be among B/C/D.

Statement 2:
We know that for q = 3, p = 2 and for q = 6, p = 11.

Since p can be expressed as the sum of two positive integers in only one way, p = 2
(2 can be expressed as the sum of two positive integers only as 1 + 1). p = 11 can be
expressed in a multiple ways, such as 1 + 10; 2 + 9; 3 + 8, and more. Hence, q = 6.

Hence, statement 2 is sufficient to answer the question.

The correct answer is option B.

167. Statement 1:
We observe that the sum of three prime numbers is 56, an even number.
Hence, all the three primes cannot be odd (since three odd numbers add up to another
odd number).
Thus, one of the prime numbers must be even.

Since the only even prime is 2, we can say that one among a, b and c must be 2.
However, we cannot ascertain which of the three is 2.
Hence, statement 1 is not sufficient to answer the question.

19992016 Manhattan Review www.manhattanreview.com


254 Number Properties Guide Solutions

The correct answer would be among B/C/E.

Statement 2:
We do not know anything about the values of the three primes.

Hence, statement 2 is not sufficient to answer the question.

The correct answer would be between C/E.

Statements 1 and 2 together:


Combining the two statements, we can conclude that c is the smallest prime and since 2
is also the smallest prime, c = 2.
Hence, the answer is Yes.

Hence, statements 1 and 2 together are sufficient to answer the question and the answer
is Yes.

The correct answer is option C.

168. Statement 1:
n2 9 = (n 3)(n + 3).
Since (n2 9) is odd, we can conclude that both (n 3) and (n + 3) are odd.
Thus, n must be even.
Thus, (n 1) is odd.

Hence, (n 1)(n 3) is the product of two odd numbers and hence odd.
Hence, statement 1 is sufficient to answer the question and the answer is No.

The correct answer would be between A/D.

Statement 2:
Since (n + 1)(n + 5) is odd, we can conclude that both (n + 1) and (n + 5) are odd.
Thus, n must be even.
Thus, (n 1) and (n 3) are both odd.

Hence, (n 1)(n 3) is the product of two odd numbers and hence odd.

Hence, statement 1 is sufficient to answer the question and the answer is No.

The correct answer is option D.

www.manhattanreview.com 19992016 Manhattan Review


Number Properties Guide Solutions 255

169. Statement 1:
p p)
22 = 256 => 2(2 = 28
=> (2p ) = 8 => 2p = 23
=> p = 3.

c 3
[Note: abc 6= ab : For example, 223 = 26 = 64 whereas 22 = 28 = 256]

Hence, statement 1 is sufficient to answer the question.


The correct answer would be between A/D.

Statement 2:
2
2p = 512
2)
=> 2(p = 29
=> (p 2 ) = 9
=> p = 3.
However, since p is mentioned as positive, we have: p = 3.

Hence, statement 2 is sufficient to answer the question.

The correct answer is option D.

170. Statement 1:
k10 = 2k9
=> 10p + 10 = 2(9p + 9)
=> 10p + 10 = 18p + 18
=> 8p = 8
=> p = 1.
Hence, statement 1 is sufficient to answer the question.

The correct answer would be between A/D.

Statement 2:
k7 + k8 = 0
=> (7p + 7) + (8p + 8) = 0
=> 15p + 15 = 0
=> p = 1.
Hence, statement 2 is sufficient to answer the question.

The correct answer is option D.

19992016 Manhattan Review www.manhattanreview.com


256 Number Properties Guide Solutions

171. Statement 1:
Since p < 4, there are only three possible positive integer values of p i.e. 1, 2 and 3.

We know: 2p1 = p 2 .

Checking with the above values of p, we have:

p = 1 : LHS = 211 = 20 = 1 and RHS = 12 = 1. (Satisfies)


p = 2 : LHS = 221 = 21 = 2 and RHS = 22 = 4. (Does not satisfy)
p = 3 : LHS = 231 = 22 = 4 and RHS = 32 = 9. (Does not satisfy)

Thus, we have p = 1.
Since 1 is a factor of every positive integer, we can conclude that p = 1 must be a factor
of a positive integer q.
Hence, statement 1 is sufficient to answer the question.

The correct answer would be between A/D.

This is one of the tricky question even for those who are experts in quants, as by sheer
looking at the statement, one could conclude that since the statement does not give any
clue about q, it cannot be an answer.

Statement 2:
Since this statement does not provide any information about p, we cannot answer the
question.

Solving 3q3 6 = 75 => q = 3, which is does not help as p may have any value.

Hence, statement 2 is not sufficient to answer the question.

The correct answer is option A.

172. If a number n has divisors other than 1 and n, it is not prime (with the exception of 1
which is also not a prime number).

Thus, the question wants us to find whether n is prime.

Statement 1:
Since n < 6, possible values of n are 1, 2, 3, 4, or 5.
Thus, n may not have divisors other than 1 and n i.e. be prime (n = 2, 3, or 5) or may
have divisors other than 1 and n (n = 4).

www.manhattanreview.com 19992016 Manhattan Review


Number Properties Guide Solutions 257

Hence, statement 1 is not sufficient to answer the question.

The correct answer would be among B/C/E.

Statement 2:
5 13
Since 5 < 4n < 11 i.e. <n< , possible values of n are 2 or 3; both are prime.
4 4

Thus, n does not have divisors other than 1 and n.

Hence, statement 2 is sufficient to answer the question.

The correct answer is option B.

173. Statement 1:
Let the quotient when p is divided by 5 be q.

Since the remainder is 2, we have: p = 5q + 2.


If q is even, then p becomes 5 even + even = even; however, if q is odd, then p becomes
5 odd + even = odd.
Thus, p can be even or odd.

Hence, statement 1 is not sufficient to answer the question.


The correct answer would be among B/C/E.

Statement 2:
Let the quotient when p is divided by 8 be m.
Since the remainder is 2, we have: p = 8m + 2.
If m is even, then p becomes 8 even + even = even; and if m is odd, then too, p
becomes 8 odd + even = even.

Thus, p is even.

Hence, statement 2 is sufficient to answer the question.

The correct answer is option B.

19992016 Manhattan Review www.manhattanreview.com


258 Number Properties Guide Solutions

174. Statement 1:
Since u > 8, possible values of u could be negative numbers having magnitude less
than 8, or any non-negative number.

Thus, if u = 4 => u2 = 16 < 64.


However, if u = 10 => u2 = 100 > 64.
Thus, u2 may be more than 64 or less than 64.

Hence, statement 1 is not sufficient to answer the question since the answer may be Yes
or No.

The correct answer would be among B/C/E.

Statement 2:
Since u < 8, possible values of u could be positive numbers having magnitude less than
8, or any non-positive number.

Thus, if u = 4 => u2 = 16 < 64.


However, if u = 10 => u2 = 100 > 64.
Thus, u2 may be more or less than 64.

Hence, statement 2 is not sufficient to answer the question since the answer may be Yes
or No.

The correct answer would be between C/E.

Statements 1 and 2 together:


Combining both statements, we get: 8 < u < 8.
Thus, on squaring, we have: 0 u2 < 64.

Hence, we can definitely say that u2 < 64.

Hence, statements 1 and 2 together are sufficient to answer the question and the answer
is Yes.

The correct answer is option C.

175. Statement 1:

Since 3 p < 1 => p < 13 => p < 1.

Again, since 3 p > 0 => p > 0.
Thus, we have: 0 < p < 1.

www.manhattanreview.com 19992016 Manhattan Review


Number Properties Guide Solutions 259

Hence, statement 1 is sufficient to answer the question and the answer is Yes.

The correct answer would be between A/D.

Statement 2:
1 1
p4 = => p = .
16 2

Hence, statement 2 is not sufficient to answer the question since the answer may be Yes
or No.

The correct answer is option A.

176. Statement 1:
m n
Since the greatest common divisor (GCD or HCF) of and is 12, it implies that both
4 4
m n
and are co-prime multiples of 12 (since there should not be any common factor
4 4
other than 12).

Thus, both m and n are co - prime multiples of 12 4 = 48.


Thus, the greatest common divisor of m and n is 48.

Hence, statement 1 is sufficient to answer the question.

The correct answer would be between A/D.

Statement 2:
m n
The fact that and are even implies that m and n are multiples of 4 and thus, the
2 2
greatest common divisor is a multiple of 4.

However, we cannot find the value of the greatest common divisor.

Hence, statement 2 is not sufficient to answer the question.

The correct answer is option A.

x
177. The question asks what is the value of ?
y
Statement 1:
x 2 2xy 3y 2 = 0
=> x 2 3xy + xy 3y 2 = 0
=> x(x 3y) + y(x 3y) = 0

19992016 Manhattan Review www.manhattanreview.com


260 Number Properties Guide Solutions

=> (x + y)(x 3y) = 0


=> x = y or x = 3y
x x
=> = 1 or =3
y y
Though x is positive, y may be positive or negative; hence, both the above values of
x
are possible.
y
Hence, statement 1 is not sufficient to answer the question.

The correct answer would be among B/C/E.

Statement 2:

x 9y
x% of =
y 100
x x 9y
=> =
100 y 100
x2
=> =9
y2
x
=> = 3
y
Though x is positive, y may be positive or negative; hence, both the above values are
possible.

Hence, statement 2 is not sufficient to answer the question.

The correct answer would be between C/E.

Statements 1 and 2 together:


x
Combining both statements, we see that = 3 is the only solution possible.
y

Hence, statements 1 and 2 are sufficient to answer the question.

The correct answer is option C.

375 375
178. will be an integer provided 5n is a factor of 375 i.e. n is a factor of = 75.
5n 5
The factors of 75 = 3 52 are 1, 3, 5, 15, 25 and 75.

Statement 1:
Since n > 5 => possible values of n are 15, 25 and 75.
Thus, there are three possible values of n unique answer!

www.manhattanreview.com 19992016 Manhattan Review


Number Properties Guide Solutions 261

Hence, statement 1 is sufficient to answer the question.

The correct answer would be between A/D.

Statement 2:
Since n has more than two factors, n cannot be any of 1, 3 or 5 (These numbers have
only two factors).

Thus, the possible values of n are 15, 25 and 75.


Thus, there are three possible values of n.

Hence, statement 2 is sufficient to answer the question.

The correct answer is option D.

240 + 150n
 
179. will be an integer if (240 + 150n) is divisible by n.
n
Since 150n is divisible by n, 240 should also be divisible by n.
Thus, n should be a factor of 240.

Statement 1:
Since n is a factor of 75 (i.e. 1, 3, 5, 15, 25 or 75), it may or may not be a factor of 240
(since 240 is not a multiple of 75).

For example: if n = 5: Here, n is a factor of 240. So, the answer is Yes.


However, if n = 25 : Here, n is not a factor of 240. So, the answer is No.

Hence, statement 1 is not sufficient to answer the question.

The correct answer would be among B/C/E.

Statement 2:
Since n is a factor of 30, it must also be a factor of 240 (since 30 is itself a factor of 240).

Hence, statement 2 is sufficient to answer the question.

The correct answer is option B.

180. Statement 1:
Since n > 2 and prime, possible values of n could be 3, 5, 7, etc.

19992016 Manhattan Review www.manhattanreview.com


262 Number Properties Guide Solutions

If n = 3, then n2 = 9 => the remainder when 9 is divided by 6 is 3.


However, if n = 5, then n2 = 25 => the remainder when 25 is divided by 6 is 1.

Thus, the remainder obtained is not unique.

Hence, statement 1 is not sufficient to answer the question.

The correct answer would be among B/C/E.

Statement 2:
Since n > 7 and prime, possible values of n could be 11, 13, 17, etc.

We know that any prime number greater than 3 can be represented in the form (6k + 1)
or (6k + 5); where k is a positive integer.
For example, 13 = 6 2 + 1 while 17 = 6 2 + 5.

If n = 6k + 1, then n2 = (6k + 1)2 = 36k2 + 12k + 1 = 6(6k2 + 2k) + 1 => the remainder
when n2 is divided by 6 is 1.

Again, if n = 6k + 5, then n2 = (6k + 5)2 = 36k2 + 60k + 25 = 6(6k2 + 10k) + 25 => the
remainder when n2 is divided by 6 is the same as when 25 is divided by 6 which is 1.

Thus, the remainder obtained is unique.

Hence, statement 2 is sufficient to answer the question.

The correct answer is option B.

181. We know that |p| gives us the magnitude of p.


Thus, we have two possible cases:

(1) If p 0 => |p| = p


(2) If p 0 => |p| = p

Statement 1:
|p|3 + p 3 = 0 => |p|3 = p 3 .
Since |p| is always non-negative, we must have p < 0.
However, we do not get any exact value of p.
Hence, statement 1 is not sufficient to answer the question.

The correct answer would be among B/C/E.

www.manhattanreview.com 19992016 Manhattan Review


Number Properties Guide Solutions 263

Statement 2:
Since |p| = p (depending on whether p > 0 or p < 0), we have: |p|2 = (p)2 = p 2 .
Thus, |p|2 = 2p => p 2 = 2p => p 2 2p = 0 => p(p 2) = 0 => p = 0 or 2.
Since p is non-zero => p = 2.
Hence, we get a unique value of p.

Hence, statement 2 is sufficient to answer the question.

The correct answer is option B.

182. At first sight, the question may seem to be self-proved, but it is not so. If a, b, c, & d are
positive, |d| is maximum; however if they are negative, |a| is maximum as a negative
number with higher absolute value is in fact smaller.

For any number n, |n| = |n 0| refers to the distance of the point from the point 0.

Statement 1:
We do not know if a, b, c and d are positive.

If a > 0 and given that a = 2d => d < 0 => d < a


However, we know from the question that a < d.
Hence, this situation is not possible.

However, if a < 0 and a = 2d => d > 0 (this satisfies the condition that a < d).
Also, |a| = | 2d| = 2|d|.

Since b and c lie between a and d, |b| and |d| cannot be the maximum (since they are
closer to 0).

Thus, |a| has the maximum value.

Hence, statement 1 is sufficient to answer the question.

The correct answer would be between A/D.

Statement 2:
Since d = b and given that b < d => d > 0 and b < 0.

Thus, b and d are at equal distances from 0.


Since c lies in between b and d, |c| must be the minimum.

19992016 Manhattan Review www.manhattanreview.com


264 Number Properties Guide Solutions

Also, since a is further to the left of b, |a| > |b| = |d| > |c|.

Thus, |a| has the maximum value.

Hence, statement 2 is sufficient to answer the question.

The correct answer is option D.

Alternate approach:

From statement 1: = 2d , and the information given in the question: a < d, we can
conclude that a cannot be positive and d cannot be negative. Let see how.

Say, d = 2 => a = 4 => a > d, but this case will invalidate the fact given in the
question: a < d, which is not possible.

Note that in DS, the information given in the question cannot be challenged. While
choosing an assumed value for a statement, one must be wary of the fact that the
information in the question is not invalidated.

So, we can conclude that |a| > |d|. Lets see how.

Keeping in mind the constraint: a = 2d, we assume that d = 2 => a = 4. So it


validates |a| > |d| => | 4| > |2| => 4 > 2.

So the situation is: [a = 4] < b < c < [d = 2]

Whatever values b & c assume, |a| > |b| > |c| > |d|.

Similarly, you can build scenarios for statement 2.

183. Statement 1:
What does x + z = 2y convey? It implies that y is the average (arithmetic mean) of
x & y. If x, y & z are not equal, then we can conclude that y is neither the greatest nor
the smallest.

x+y+z
Adding y to both sides: x + y + z = 3y => 3 = y . . . (i)

However, since we do not know the values of x, y and z, we cannot determine the answer.

Hence, statement 1 is not sufficient to answer the question.

www.manhattanreview.com 19992016 Manhattan Review


Number Properties Guide Solutions 265

The correct answer would be among B/C/E.

Statement 2:
xz = y 2


Multiplying y to both sides: xyz = y 3 => 3 xyz = y . . . (ii)

However, since we do not know the values of x, y and z, we cannot determine the answer.

Hence, statement 2 is not sufficient to answer the question.

The correct answer would be between C/E.

Statements 1 and 2 together:


From (i), we can say that y is the arithmetic mean of x, y and z.
From (ii), we can say that y is the geometric mean of x, y and z.

Thus, we see that the arithmetic and geometric means are the same for x, y and z.
This is possible only when all the terms are the same i.e. x = y = z.

Hence, |x y| + |y z| = 0.

Hence, statements 1 and 2 are sufficient to answer the question.

The correct answer is option C.

Alternate approach:

Statement 1 is not sufficient as if x = y = z, the value of |x y| + |y z| = 0. However


if they are unequal, |x y| + |y z| 6= 0. Not sufficient!

Same goes with statement 2.

For the scenario x = y = z, both the statements are consistent and |x y| + |y z| = 0


unique answer.

If we find a scenario that is x 6= y 6= z, and xz = y 2 , value of |x y| + |y z| 6= 0.


Answer would be E.

However we cannot find such a scenario. You will either invalidate x = y = z or invali-
date xz = y 2 .

19992016 Manhattan Review www.manhattanreview.com


266 Number Properties Guide Solutions

184. Statement 1:
We have: y(2x + 1) is even.
However, 2x is always even => (2x + 1) is odd => y is even.

However, x may be even or odd:


x
If x is even: y is a ratio of two even numbers which may be an even or an odd integer or
a fraction.
x
If x is odd: y is a ratio of an odd and an even number which is always a fraction.

Hence, statement 1 is not sufficient to answer the question as the answer may be Yes or
No.

The correct answer would be among B/C/E.

Statement 2:
We have: x = (2x 1)2

(2x 1) is the difference between an even number (2x) and an odd number (1) i.e. is
odd.
Thus, the RHS is an odd number => the LHS must be odd as well => x is odd.

Alternatively: On solving the equation: x = 4x 2 4x + 1 => 4x 2 5x + 1 = 0


1
=> (4x 1)(x 1) = 0 => x = 4 or 1 => x = 1 which is odd (since x is an integer).

However, y may be even or odd leading to similar situations as with statement 1.


Hence, statement 2 is not sufficient to answer the question as the answer may be Yes or
No.

The correct answer would be between C/E.

Statements 1 and 2 together:


x
Combining both statements, we have: x is odd and y is even => y can never be an
integer.

Hence, statements 1 and 2 are sufficient to answer the question and the answer is No.

The correct answer is option C.

185. Statement 1:
The factors of 12 are: 1, 2, 3, 4, 6 and 12.

www.manhattanreview.com 19992016 Manhattan Review


Number Properties Guide Solutions 267

Since x > 3 => possible values of x are: 4, 6 or 12 i.e. x is definitely a multiple of 2.

However, we do not know the value of y.

Hence, statement 1 is not sufficient to answer the question.

The correct answer would be among B/C/E.

Statement 2:
Since y is an even multiple of 15, possible values of y are: 30, 60, 90 . . . etc. i.e. y is a
multiple of 30.

Thus, xy is a multiple of 30.


Hence, xy may or may not be divisible by 60.

Hence, statement 2 is not sufficient to answer the question as the answer may be Yes or
No.

The correct answer would be between C/E.

Statements 1 and 2 together:


Since x is a multiple of 2 and y is a multiple of 30 => xy is a multiple of 2 30 = 60.
Hence, xy is divisible by 60.

Hence, statements 1 and 2 together are sufficient to answer the question and the answer
is Yes.

The correct answer is option C.

1 1 1 x+y 1 xy
186. We need to check if: + = => = => z = .
x y z xy z x+y

Thus, z will be an integer if xy is divisible by (x + y).

Statement 1:
Possible values of x and y are 1, 2, 3, 4, 5 or 6.

1 1 1
If x be 1 => + > 1; It is not possible as it cannot be expressed as , where z is an
1 y z
integer.
Thus, we can safely say that x and y cannot be 1 => Possible values of x and y are 2,
3, 4, 5 or 6.

19992016 Manhattan Review www.manhattanreview.com


268 Number Properties Guide Solutions

1 1 5 1
For x = 2 and y = 3: + = which cannot be expressed in the form of .
2 3 6 z
1 1 1 1
Again, for x = 3 and y = 6: + = which is of the form .
3 6 2 z

Hence, statement 1 is not sufficient to answer the question as the answer may be Yes or
No.

The correct answer would be among B/C/E.

Statement 2:
Since the GCD of x and y is 1, it implies that x and y have no common factors other
than 1.

Thus, xy and (x + y) will never have any common factor other than 1.
Thus, xy is not divisible by (x + y).

1 1 1
Hence, + cannot be expressed as .
x y z

Hence, statement 2 is sufficient to answer the question and the answer is No.

The correct answer is option B.

187. Statement 1:
There are four prime numbers less than 10: 2, 3, 5 and 7.
Thus, the values of a, b and c can be any among the above primes.

Hence, the values of a, b and c are not known.

Hence, statement 1 is not sufficient to answer the question.

The correct answer would be among B/C/E.

Statement 2:
ab = 15
bc = 21
ac = 35

Multiplying the three, we get: ab bc ac = 15 21 35

=> a2 b2 c 2 = 32 52 72

www.manhattanreview.com 19992016 Manhattan Review


Number Properties Guide Solutions 269

=> abc = 3 5 7 = 105.

Hence, statement 2 is sufficient to answer the question.

The correct answer is option B.

188. Statement 1:
8x 2 + 6x + 1 = 0 => 8x 2 + 4x + 2x + 1 = 0

=> 4x(2x + 1) + 1(2x + 1) = 0


=> (4x + 1)(2x + 1) = 0
1 1
=> x = or = 0.25 or 0.5.
4 2

If x = 0.25 => [x] = [0.25] = 0 (since 0 is the least integer greater than or equal to
0.25).
If x = 0.5 => [x] = [0.5] = 0 (since 0 is the least integer greater than or equal to
0.5).

Hence, in either case, [x] = 0 => [x] 0.

Hence, statement 1 is sufficient to answer the question and the answer is No.

The correct answer would be between A/D.

Statement 2:
2x 2 x 1 = 0 => 2x 2 2x + x 1 = 0

=> 2x(x 1) + (x 1) = 0
=> (2x + 1)(x 1) = 0
1
=> x = or 1 => x = 0.5 or 1.
2

If x = 0.5 => [x] = [0.5] = 0.

If x = 1 => [x] = [1] = 1 (since 1 is the least integer greater than or equal to 1).

Hence, in either case, [x] 0.

Hence, statement 2 is sufficient to answer the question and the answer is No.

The correct answer is option D.

19992016 Manhattan Review www.manhattanreview.com


270 Number Properties Guide Solutions

189. Any number when divided by 3 leaves any of 0, 1 or 2 as remainders.


Hence, squares of numbers would leave respective remainders of 02 = 0, 12 = 1or
22 = 4 1 (since 4 is greater than 3, we divide 4 by 3 again to find the final remainder
of 1).

Thus, the square of a number would leave remainders either 0 or 1.

Thus, we can say that:

A number which is a multiple of 3, when squared and divided by 3 leaves a remainder 0.


Again, a number which is not a multiple of 3, when squared and divided by 3 leaves a
remainder 1.

Statement 1:
Since the three numbers are consecutive integers, there would be exactly one number
which is a multiple of 3 and the other two numbers would not be multiples of 3.

Thus, when the numbers are squared and divided by 3, one would leave a remainder 0
and the other two would leave a remainder of 1 each.

Hence, the remainders would be 0, 1 and 1 i.e. sum of the remainders is 0 + 1 + 1 = 2.

Hence, statement 1 is sufficient to answer the question.

The correct answer would be between A/D.

Statement 2:
Since we do not know the exact numbers (we only know the range), we cannot determine
the remainders obtained on dividing the square of the numbers by 3.

Hence, statement 2 is not sufficient to answer the question.

The correct answer is option A.

190. The digits of a number must be less than or equal to 9 and more than or equal to 0.

Statement 1:
Since the product of the digits is 36 (i.e. no digit is 0), possible values of the digits in
any order would be: (1, 4, 9); (3, 3, 4); (2, 3, 6); (2, 2, 9) or (1, 6, 6).

Since we need the sum of the digits, we are not interested to find which digit occurs in
which position.

www.manhattanreview.com 19992016 Manhattan Review


Number Properties Guide Solutions 271

The sum of the digits obtained in the above cases is: 14, 10, 11, 13 or 13 respectively.
Thus, the sum of the digits cannot be determined since the answer is not unique.

Hence, statement 1 is not sufficient to answer the question.

The correct answer would be among B/C/E.

Statement 2:
Statement 2 does not give us any value of the digits; hence, the sum cannot be deter-
mined.

Hence, statement 2 is not sufficient to answer the question.

The correct answer would be between C/E.

Statements 1 and 2 together:


Combining both statements, we have:

(1) (1, 4, 9) and (2, 3, 6) cannot be used since all digits are distinct
(2) (3, 3, 4) cannot be used since it uses the digit 4

Only (2, 2, 9) and (1, 6, 6) are possible since all digits are not distinct and the digit 4 is
not used.
In each case, the sum of the digits is 13.

Hence, statements 1 and 2 are sufficient to answer the question.

The correct answer is option C.

191. The remainder when a number is divided by 10 is the last digit or the units digit of the
number.

Statement 1:
When a number is divided by 100, the remainder obtained is the last two digits of the
number.

Since the number N leaves a remainder 24 when divided by 100, the last two digits of
the number is 24.

Thus, the number 2N would have its last digit as 2 4 = 8.

19992016 Manhattan Review www.manhattanreview.com


272 Number Properties Guide Solutions

Hence, when the number N is divided by 10, the remainder is 8.

Hence, statement 1 is sufficient to answer the question.

The correct answer would be between A/D.

Statement 2:
The remainder obtained when a number with its last digit as k is divided by 5 is either k
(if k < 5) or is k 5 (if k > 5).

For example: If 23 is divided by 5, the remainder is 3 (since 3 < 5); while if 27 is divided
by 5, the remainder is 7 5 = 2 (since 7 > 5).

Since the number N leaves a remainder 4 when divided by 5, the last digit of N is 4 or (4
+ 5) = 9.

Hence, the last digit of 2N is either 2 4 = 8 or 2 9 = 18 i.e. 8.

Hence, when the number N is divided by 10, the remainder is 8.

Hence, statement 2 is sufficient to answer the question.

The correct answer is option D.

192. Statement 1:

16 10r = k 103 Squaring both sides:
2
16 10r = (k 103 )
=> 16 10r = k2 106

Since there are two unknowns in the same equation, we cannot solve, for k.
(Note: We cannot simply conclude that 16 = k2 and 10r = 106 )

For example, if r = 4 => k2 = 1600

Hence, statement 1 is not sufficient to answer the question.

The correct answer would be among B/C/E.

Statement 2:
100 < 2.36 10k < 1000

www.manhattanreview.com 19992016 Manhattan Review


Number Properties Guide Solutions 273

Since k is an integer, we need to use positive values for k:

k = 1 : 2.36 10k = 2.36 10 = 23.6 (does not lie in the range from 100 to 1000)
k = 2 : 2.36 10k = 2.36 102 = 236 (lies in the range from 100 to 1000)
k = 3 : 2.36 10k = 2.36 103 = 2360 (exceeds the range from 100 to 1000)

Hence, k = 2.

Hence, statement 1 is sufficient to answer the question.

The correct answer is option B.

193. x n+1 xy n = x(x n y n ).

Statement 1:
Since n is even, (x n y n ) is divisible by both (x + y) and (x y).

For example:

n = 4 : x 4 y 4 = (x 2 y 2 )(x 2 + y 2 ) = (x x y )(x 2 + y 2 )
x + y )(x
n = 6 : x 6 y 6 = (x 3 y 3 )(x 3 + y 3 ) = (x
x y )(x 2 + xy + y 2 )(x
x + y )(x 2 xy + y 2 )

Hence, x n+1 xy n is divisible by (x + y) and (x y) i.e. is divisible by (x 2 y 2 ).

Hence, statement 1 is sufficient to answer the question and the answer is Yes.

The correct answer would be between A/D.

Statement 2:
If n is a multiple of 3, x n y n may be divisible by (x 2 y 2 ) or may not be divisible by
(x 2 y 2 ).

For example:

n = 3 : x 3 y 3 = (x y)(x 2 + xy + y 2 ) i.e. only divisible by (x y) and not by


(x 2 y 2 )
n = 6 : We have seen that (x 6 y 6 ) is divisible by both (x + y) and (x y) i.e. by
(x 2 y 2 )

19992016 Manhattan Review www.manhattanreview.com


274 Number Properties Guide Solutions

Hence, statement 2 is not sufficient to answer the question as the answer may be Yes or
No.

The correct answer is option A.

194. Statement 1:
For any positive integer n, the value n(n + 1)(n + 2) represents the product of three
consecutive integers.

Among any three consecutive integers, at least one number must be even.
Also, exactly one number must be a multiple of 3.
Hence, the product of three consecutive integers is a multiple of both 2 and 3 i.e. a
multiple of 6 = 3!

Thus, n(n + 1)(n + 2) is divisible by 3! => a = 3.

Thus, a3 + a = 33 + 3 = 30, which is divisible by 6.

Hence, statement 1 is sufficient to answer the question and the answer is Yes.

The correct answer would be between A/D.

Statement 2:
Let us take a few values to verify this statement.

a = 1 : a3 + a = 1 + 1 = 2, which is not divisible by 6.


a = 3 : We have already seen that (33 + 3) is divisible by 6.

Hence, statement 2 is not sufficient to answer the question as the answer may be Yes or
No.

The correct answer is option A.

p
195. 45 < n < 45 + p

Squaring both sides:

45 < n2 < 45 + p

Thus, we need to find the number of perfect squares between 45 and (45 + p).

www.manhattanreview.com 19992016 Manhattan Review


Number Properties Guide Solutions 275

Statement 1:
Possible values of p are 1, 2, 3, 4 or 5.

p = 1 : The number of perfect squares between 45 and 46 is zero.


p = 2 : The number of perfect squares between 45 and 47 is zero.
p = 3 : The number of perfect squares between 45 and 48 is zero.
p = 4 : The number of perfect squares between 45 and 49 is zero.
p = 5 : The number of perfect squares between 45 and 50 is one (i.e. 49).

Hence, the answer is either zero or one i.e. not unique.

Hence, statement 1 is not sufficient to answer the question.

The correct answer would be among B/C/E.

Statement 2:
Possible values of p are 5, 6, 7, 8 or 9.

p = 5 : The number of perfect squares between 45 and 50 is one (i.e. 49).


p = 6 : The number of perfect squares between 45 and 51 is one (i.e. 49).
p = 7 : The number of perfect squares between 45 and 52 is one (i.e. 49).
p = 8 : The number of perfect squares between 45 and 53 is one (i.e. 49).
p = 9 : The number of perfect squares between 45 and 54 is one (i.e. 49).

Hence, the answer is one.

Hence, statement 2 is sufficient to answer the question.

The correct answer is option B.

c
196. > 0 if c > 0 and a > 0 OR c < 0 and a < 0.
a
c
Since it is given that c < 0, then
> 0 => a < 0.
a
Thus, we need to check whether a < 0.

Statement 1:

b3
>0
|a|

19992016 Manhattan Review www.manhattanreview.com


276 Number Properties Guide Solutions

Since |a| > 0 (since modulus of any number only takes the positive magnitude of the
number), we can conclude that b3 > 0 => b > 0.

However, we cannot determine whether a < 0.

Hence, statement 1 is not sufficient to answer the question.

The correct answer would be among B/C/E.

Statement 2:

|b|
>0
a+c

Since |b| > 0 => a + c > 0.


However, it is given that c < 0 => a > 0.
c
Hence, < 0 (since c < 0 and a > 0).
a

Hence, statement 2 is sufficient to answer the question and the answer is No.

The correct answer is option B.

197. Statement 1:

1 1 2 1
   
x2 + 2 = x+ 2(x)
x x x
2
1 1
  
=> x 2 + 2 = x + 2
x x
1 2 1
   
=> x + = x2 + 2 + 2 = 2 + 2 = 4
x x
1
 
=> x + = 2.
x
1
 
(Note: x + 6= 2 since x is a positive number)
x

Hence, statement 1 is sufficient to answer the question.

The correct answer would be between A/D.

Statement 2:

2
1 1 1
    
x+ = x+ 2 x)(
x x x

www.manhattanreview.com 19992016 Manhattan Review


Number Properties Guide Solutions 277

1 1 2
   
=> x + = x+ 2
x x
1
 
=> x + = 22 2 = 4 2 = 2.
x

Hence, statement 2 is sufficient to answer the question.

The correct answer is option D.

198. Statement 1:

2
1 1

+
a 2b
P=
4
ab
b
Substituting a = => b = 2a, we have:
2

2 2
1 1 1 1
 
+ +
a 2b a 4a
P= =
4 4
ab 2a2

5 2
 

4a 25 a2
=> P = =
2 16a2 2
a 2

25
=> P = 1.
32

Hence, statement 1 is sufficient to answer the question and the answer is No.

The correct answer would be between A/D.

Alternatively, you could assume a convenient value for a & b; for example, say
b = 4 => a = 2 and solve for P .

Statement 2:

Knowing the value of a will not help us to calculate the value of P since the value of b is
unknown (the final result will be a function of b).

Hence, statement 2 is not sufficient to answer the question.

19992016 Manhattan Review www.manhattanreview.com


278 Number Properties Guide Solutions

The correct answer is option A.

199. From the definition of the symbol, we have: x 2 = x 2 + 2x .

Statement 1:
We know that x is a positive integer.
Thus, 2x and x 2 are both positive integers.

Keeping in sight, x 2 = 17, we can observe that 25 = 32 > 17 => x < 5.

Checking with the values of x as 1, 2, 3 or 4:

x = 1 : 12 + 21 = 3 6= 17 => x 6= 1 (does not satisfy)


x = 2 : 22 + 22 = 8 6= 17 => x 6= 2 (does not satisfy)
x = 3 : 32 + 23 = 17 => x = 3 (satisfies)
x = 4 : 42 + 24 = 32 6= 17 => x 6= 4 (does not satisfy)

Hence, we have x = 3, a unique value.

Hence, statement 1 is sufficient to answer the question.

The correct answer would be between A/D.

Statement 2:
Possible prime values of x can be only 2 or 3 (since x < 5 as discussed earlier).

We know that x = 3 is the only solution.

Hence, statement 2 is sufficient to answer the question.

The correct answer is option D.

200. Statement 1:
Possible values of a and b are 2, 3, 5 or 7.

Since we need to check whether (2a)b > 30, we should try with the minimum possible
values of a and b so that (2a)b is a minimum value to check against 30.

Since a and b are distinct, we can have either a = 2, b = 3 OR a = 3, b = 2.

If a = 2, b = 3: (2a)b = 43 = 64 > 30.

www.manhattanreview.com 19992016 Manhattan Review


Number Properties Guide Solutions 279

If a = 3, b = 2: (2a)b = 62 = 36 > 30.

Since, for the minimum values of a and b, the value of (2a)b > 30, we can conclude that
it will be the same for any values of a and b.

Hence, statement 1 is sufficient to answer the question and the answer is Yes.

The correct answer would be between A/D.

Statement 2:
Since it is not mentioned that a and b are positive, we can have a situation when
a = 2, b = 7 along with the situation when a = 2, b = 3 (as discussed in statement 1).

If a = 2, b = 7: (2a)b = (4)7 < 0 => (2a)b 30.

Hence, statement 2 is not sufficient to answer the question and the answer may be Yes
or No.

The correct answer is option A.

19992016 Manhattan Review www.manhattanreview.com


280 Number Properties Guide Talk to us

www.manhattanreview.com 19992016 Manhattan Review


Chapter 6

Talk to Us

Have a Question?
Please email your questions to info@manhattanreview.com. We will be happy to answer you.
Your questions can be related to a concept, an application of a concept, an explanation of a
question, a suggestion for an alternate approach, or anything else you wish to ask regarding
the GMAT.
Please mention the page number when quoting from the book.

GMAC Quants Resources


Official Guide 2017: It is the best resource to prepare for the GMAT. It is a complete
GMAT book. It comes with a Diagnostic test, which helps you measure your capability
beforehand. It features Verbal, Quantitative, and Integrated Reasoning questions types.
The book contains an access code to avail GMATPrep Software, Online Question Bank and
Exclusive Video.

GMATPrep Software: If you buy the OG, you get a free online resource from the GMAC
the testmaker. Apart from practice questions and explanation, it also has two genuine
Computer Adaptive tests; you can also buy four additional CATs and few practice ques-
tions upon the payment.

281

You are a unique candidate with unique experience.
We help you to sell your story to the admissions committee.

Manhattan Admissions is an educational consulting firm that


guides academic candidates through the complex process of
applying to the worlds top educational programs. We work
with applicants from around the world to ensure that they
represent their personal advantages and strength well and get
our clients admitted to the worlds best business schools, gra-
duate programs and colleges.

We will guide you through the whole admissions process:

Personal Assessment and School Selection


Definition of your Application Strategy
Help in Structuring your Application Essays
Unlimited Rounds of Improvement
Letter of Recommendation Advice
Interview Preparation and Mock Sessions
Scholarship Consulting

To schedule a free 30-minute consulting and candidacy evalua-


tion session or read more about our services, please visit or call:

www.manhattanadmissions.com +1.212.334.2500

About the Turbocharge Your GMAT Series (6th Edition)
The Turbocharge Your GMAT Series is carefully designed accessible manner. We delve deeply into the content of
to be clear, comprehensive, and content-driven. Long every single testing area and zero in on exactly what you
regarded as the gold standard in GMAT prep worldwide, need to know to raise your score. The full series is
Manhattan Reviews GMAT prep books offer professional comprised of 16 guides that cover concepts in mathemat-
GMAT instruction for dramatic score improvement. Now in ics and grammar from the most basic through the most
its updated 6th edition, the full series is designed to advanced levels, making them a great study resource for
provide GMAT test-takers with complete guidance for all stages of GMAT preparation. Students who work
highly successful outcomes. As many students have through all of our books benefit from a substantial boost
discovered, Manhattan Review's GMAT books break down to their GMAT knowledge and develop a thorough and
the different test sections in a coherent, concise, and strategic approach to taking the GMAT.

Turbocharge Your GMAT Series


About Manhattan Review About the Author


Manhattan Reviews origin can be traced directly back to Professor Dr. Joern Meissner has more than 25 years of
an Ivy League MBA classroom in 1999. While teaching teaching experience at the graduate and undergraduate
advanced quantitative subjects to MBAs at Columbia levels. He is the founder of Manhattan Review, a world-
Business School in New York City, Professor Dr. Joern wide leader in test prep services, and he created the origi-
Meissner developed a reputation for explaining compli- nal lectures for its first GMAT preparation classes. Prof.
cated concepts in an understandable way. Remembering Meissner is a graduate of Columbia Business School in
their own less-than-optimal experiences preparing for New York City, where he received a PhD in Management
the GMAT, Prof. Meissner's students challenged him to Science. He has since served on the faculties of prestigious
assist their friends, who were frustrated with conventional business schools in the United Kingdom and Germany. He
GMAT preparation options. In response, Prof. Meissner is a recognized authority in the areas of supply chain man-
created original lectures that focused on presenting GMAT agement, logistics, and pricing strategy. Prof. Meissner
content in a simplified and intelligible manner, a method thoroughly enjoys his research, but he believes that
vastly different from the voluminous memorization and grasping an idea is only half of the fun. Conveying knowl-
so-called tricks commonly offered by others. The new edge to others is even more fulfilling. This philosophy was
approach immediately proved highly popular with GMAT crucial to the establishment of Manhattan Review, and
students, inspiring the birth of Manhattan Review. remains its most cherished principle.

Since its founding, Manhattan Review has grown into a www.manhattanreview.com


multi-national educational services firm, focusing on info@manhattanreview.com
+1 (212) 316 -2000
GMAT preparation, MBA admissions consulting, and appli- +1 (800) 246-4600
cation advisory services, with thousands of highly satis-
Manhattan Review, 275 Madison Avenue, Suite 1429, New York, NY 10016.
fied students all over the world. The original lectures have
been continuously expanded and updated by the Manhat-
tan Review team, an enthusiastic group of master GMAT
professionals and senior academics. Our team ensures
that Manhattan Review offers the most time-efficient and
cost-effective preparation available for the GMAT. Please
visit www.ManhattanReview.com for further details.

Вам также может понравиться